Você está na página 1de 152

Prof.

Laerte Pereira
Turma ITA/IME
Cinemática terminando às 2 h 20 min do dia 1º de
Objetivos janeiro do ano seguinte. Quanto tempo
Ao estudar este capítulo você deve durou essa comemoração?
reconhecer conceitos básicos para a análise
de movimento, em especial o uniforme e Resolução:
representar graficamente variáveis de
movimento uniforme em função do tempo.

CONCEITOS BÁSICOS
A Cinemática é o ramo da Mecânica que
descreve os movimentos dos corpos através
dos conceitos de posição, velocidade e
aceleração. A Dinâmica estuda os fatores Posição e referencial
que modificam os movimentos.
Inicialmente, serão necessários alguns
conceitos primitivos, os quais não são
definidos, mas devem ter a mesma
significação para todos. Os primeiros
conceitos primitivos são os conceitos de
instante e de tempo.
Instante é o momento na qual se registra o Um objeto está situado no ponto O e três
tempo. Geralmente, utiliza-se cronômetros e observadores de três lugares diferentes
relógios para registrar esses tempos. O estão olhando para o mesmo objeto, então
instante ao qual se associa tempo zero (𝑡 = todos os três observadores terão
0) recebe o nome de origem dos tempos. observações diferentes sobre a posição do
Define-se tempo ou intervalo de tempo como objeto no ponto O e ninguém estará errado.
a duração de um fenômeno físico. Porque eles estão observando o objeto a
Por exemplo, podemos dizer que um trem partir de posições diferentes.
passou por uma estação no instante de O observador 'A' diz: O ponto O está a 3 m
tempo 𝑡1 e chegou à segunda estação no na direção oeste.
instante 𝑡2. O observador 'B' diz: O ponto O está a 4 m
Dessa forma, pode-se afirmar que o de distância na direção sul.
fenômeno ocorreu num intervalo de tempo, Portanto, a posição de qualquer ponto é
cuja duração foi Δ𝑡=𝑡2−𝑡1. completamente expressa por dois fatores:
Exemplo: Instante (t) pode ser dado por Sua distância em relação ao observador e
um número negativo? E intervalo de sua direção em relação ao observador.
tempo (Δt)? É por isso que a posição é caracterizada por
um vetor conhecido como vetor de posição.
Resolução: Considere um ponto P em plano xy e suas
Quando adotamos uma origem de tempo coordenadas são (x, y). Então o vetor de
(t0 = 0), atribuímos números positivos aos posição (𝑟⃗) do ponto será x𝑖̂+y𝑗̂ e se o ponto
instantes posteriores e negativos aos P está no espaço e suas coordenadas são (x,
anteriores. Assim, um instante pode ser y, z) então o vetor de posição pode ser
dado por um número negativo. O intervalo expresso como 𝑟⃗ = 𝑥𝑖̂+y𝑗̂+z𝑘̂.
de tempo (Δt = tfinal – tinicial) não pode ser Sistema de Referência: É um sistema ao
negativo, pois tfnal nunca é menor que qual um conjunto de coordenadas é anexado
tinicial. e com referência ao qual o observador
descreve qualquer evento.
Exemplo: Uma comemoração iniciou-se Exemplo: Considere um automóvel em
às 22 h 45 min do dia 31 de dezembro, movimento em relação a um referencial
Oxy solidário ao solo. Seja O’x’y’ outro Exemplo: Um automóvel parte do km 12
referencial, solidário à porta do veículo, de uma rodovia e desloca-se sempre no
como ilustra a figura a seguir: mesmo sentido até o km 90. Aí chegando,
retorna pela mesma rodovia até o km 20.

Determine se a maçaneta M está em Calcule, para esse automóvel, a variação


repouso ou em movimento: de espaço (Δs) e a distância percorrida
a) em relação a Oxy. (d):
b) em relação a O’x’y’.
a) na ida;

MOVIMENTO E REPOUSO b) na volta;


Dizemos que um dado ponto material está
em repouso em relação a certo referencial c) na ida e na volta juntas.
quando todas as suas coordenadas (𝒙, 𝒚, 𝒛),
medidas em relação ao referencial,
permanecem invariáveis com o passar do Rapidez (v, Rap.) e Velocidade (𝒗
⃗⃗)
tempo. Se uma de suas coordenadas variar,
dizemos que o ponto material está em Denomina-se rapidez a taxa de distância
movimento, em relação ao referencial percorrida com o tempo. É uma grandeza
adotado. escalar que tem o símbolo v, tem dimensão
Observe que o conceito de repouso e de física:
movimento dependem do referencial [M0 L1 T-1]
adotado. Uma partícula pode estar em
repouso em relação a um referencial e em Possui unidade de metro/segundo (S.I.),
movimento em relação a outro. cm/segundo (C.G.S.)
Rapidez uniforme: Quando uma partícula
Variação de posição e distância percorre distâncias iguais em intervalos de
percorrida. tempo iguais, então diz-se que ela está se
movendo com rapidez uniforme.
Se sinicial corresponde à posição de uma
partícula no tempo tinicial e sfinal à posição da
partícula no tempo tfinal, a variação de
Posição (Δs) entre tinicial e tfinal é determinada
por:
Na ilustração dada, o professor Laerte
Δs = sfinal - sinicial Pereira (motociclista) viaja a mesma
distância (5m) em cada segundo. Portanto,
A distância percorrida d é definida como o podemos dizer que a partícula está se
somatório dos módulos de todas as movendo com velocidade uniforme de 5 m/s.
variações de espaço ocorridas no intervalo Rapidez média: A rapidez média de uma
de tempo considerado, independentemente partícula para um determinado intervalo de
do sentido do movimento. tempo é definida como a relação entre a
distância total percorrida e o tempo gasto.
d = Σ |Δsida| + Σ |Δsvolta| Distância total percorrida
Rapidez média = tempo gasto
;
𝛥𝑠 Na segunda metade a velocidade escalar
𝑣=
Δt média é 10 km/h:

Velocidade
Denomina-se velocidade a taxa de A velocidade escalar média procurada é:
deslocamento com o tempo. É uma grandeza
vetorial com o símbolo 𝑣⃗ e tem dimensão
[M0 L1 T-1]
A unidade é metro/segundo (S.I.),
cm/segundo (C.G.S.).
Exemplo:(Moysés) Um motorista percorre
Velocidade uniforme: Diz-se que uma
10km a 40km/h, os 10km seguintes a
partícula tem velocidade uniforme, se a
80km/h mais 10km a 30km/h. Qual é a
magnitude assim como a direção de sua
velocidade escalar média do seu
velocidade permanecerem as mesmas e isto
percurso? Compare-a com a média
só é possível quando as partículas se movem
aritmética das velocidades escalares.
na mesma linha reta sem inverter o sentido
do movimento.
Resolução do Professor Laerte Pereira:
Velocidade média: A velocidade média de
Para calcular a velocidade média,
uma partícula para um determinado intervalo
precisamos antes calcular o intervalo de
de tempo é definida como a relação entre o
tempo do percurso. Para isso usaremos a
deslocamento percorrido e o tempo gasto.
equação de movimento do
deslocamento MRU, ou seja,
Velocidade média = tempo gasto
;

⃗⃗⃗⃗⃗
𝛥𝑠
𝑣⃗ =
Δt Desta maneira, o intervalo de tempo total
é dado pela soma de cada um desses
termos. Logo,
Exemplo: A velocidade escalar média de
um móvel durante a metade de um ∆t = ∆t1 + ∆t2 +∆t3.
percurso é 30 km/h e esse mesmo móvel
tem a velocidade escalar média de 10 Assim,
km/h na metade restante desse mesmo ∆t = 0,708 h.
percurso. Determine a velocidade escalar Como, ∆x =30km
média do móvel no percurso total. Então,

Resolução:

Média das velocidades. Por outro lado, a


média das velocidades é obtida
calculando-se a média aritmética das
Chamemos 2d a distância total do velocidades v1, v2 e v3. Assim,
percurso e d a metade do percurso. Seja
Δt1 o intervalo de tempo gasto pelo móvel
na primeira metade e Δt2 o intervalo na
segunda metade.
Na primeira metade a velocidade escalar
média é 30 km/h:
o que demonstra que estas duas Δφ = θ
quantidades em geral são diferentes. Δφ θ θ
ω= → ω = → Δt =
Δt Δt ω

Exemplo: (Ita 2013) Um dispositivo é Igualando as duas equações em Δt :


usado para determinar a distribuição de
velocidades de um gás. Em t = 0, com os 2R θ 2Rω
= →V=
orifícios O’ e O alinhados no eixo z, V ω θ
moléculas ejetadas de O’, após passar por
um colimador, penetram no orifício O do Considerando a primeira rotação completa
tambor de raio interno R, que gira com do tambor, para a determinação da
velocidade angular constante ω. velocidade mínima:
Considere, por simplificação, que neste
instante inicial ( t = 0) as moléculas em θ = 2πrad
movimento encontram-se agrupadas em 2Rω 2Rω Rω
V= → Vmín = → Vmín =
torno do centro do orifício O. Enquanto o θ 2π π
tambor gira, conforme mostra a figura, tais
moléculas movem-se horizontalmente no Concluindo:
interior deste ao longo da direção do eixo
z, cada qual com sua própria velocidade, 2Rω Rω
sendo paulatinamente depositadas na V − Vmín = −
θ π
superfície interna do tambor no final de ωR
seus percursos. Nestas condições, V − Vmín = ( 2π − θ)
πθ
obtenha em função do ângulo θ a
expressão para v − vmin , em que v é a
velocidade da molécula depositada
correspondente ao giro θ do tambor e
vmin é a menor velocidade possível para Exemplo: Um motorista planejou realizar
que as moléculas sejam depositadas uma viagem de 240 km em, no máximo, 3
durante a primeira volta deste. horas. Após percorrer 160 km em 1 hora e
45 minutos, teve que ficar parado por 15
minutos devido à execução de obras na
estrada. Para cumprir o planejamento
inicial, o motorista deve realizar o restante
do percurso com velocidade média de
a) 60 km/h.
b) 80 km/h.
c) 85 mk/h.
Resolução:
d) 90 mk/h.
Analisando o deslocamento da molécula,
e) 95 km/h.
pelo eixo Z, para se depositar na parede
interna do tambor:
Resolução:
ΔS = 2R
[Resposta do ponto de vista da
ΔS 2R 2R disciplina de Física]
V= →V= → Δt =
Δt Δt V Para o percurso restante, têm-se:
Analisando a velocidade angular do
tambor:
ΔS = ( 240 − 160 ) km  ΔS = 80km nessa primeira volta, foi igual a
6
v.
 1 1
5
   7 1 
Δt1 =  3 −  +   h  Δt1 = 1h
   4 4  Nessas condições, o trecho retilíneo L
dessa pista tem comprimento, em m,
A velocidade média no percurso restante igual a
deve ser: a) 50
vm =
ΔS1 80
=  vm = 80 km h
b) 100
ΔT1 1 c) 250
d) 400
[Resposta do ponto de vista da
disciplina de Matemática] Resolução:
Depois de ter percorrido 240 km em 1h e
45 minutos e mais 15 minutos parado, Tempo gasto nos trechos semicirculares:
ainda restam 80km, que deverão ser Δt1 =
2πR
percorridos em 1hora. Logo a velocidade v
média restante deverá ser:
80 Tempo gasto nos trechos retilíneos:
= 80 km h
1 2L 4L
Δt 2 = =
3v 3v
2
Exemplo: Um candidato ao Curso de
Formação de Oficiais Aviadores, após ser Do comprimento da pista, obtemos:
aprovado em todas as etapas anteriores, 2000 200 − L
2πR + 2L = R=
deverá realizar um Teste de Avaliação do 5 π
Condicionamento Físico (TACF). Uma das
provas do TACF consiste em correr 2.000 Da velocidade escalar média, obtemos:
m dentro de um intervalo de tempo 2π 
200 − L
+ 2L
2πR + 2L
máximo. Para realizá-la, tal candidato 6v
= 
6v
= π 
5 2πR 4L 5 2π 200 − L 4L
dará 5 voltas completas, numa pista v
+
3v v

π
+
3v
constituída de dois trechos retilíneos, de 6v 400 − 2L + 2L v 200v
comprimento L, e de dois trechos  =  = 
5 400 − 2L 4L 5 1200 − 2L
+
semicirculares, de raio R, mantendo-se v 3v
sempre sobre a linha pontilhada, conforme  1000v = 1200v − 2vL  2L = 200
ilustra a figura a seguir.  L = 100 m

Movimento Uniforme
Neste movimento a resultante das forças
sobre a partícula em movimente é sempre
nula.
A função da posição em função do tempo é:

Em sua primeira volta, o candidato s = s0 + v.t


percorre os trechos semicirculares com Sendo:
velocidade constante v e os trechos • s é a posição em t0 = 0, ou seja, a posição
3 inicial;
retilíneos com velocidade constante v.
• v é a velocidade escalar;
2
Além disso, sua velocidade escalar média, • s é a posição num instante t qualquer.
Gráficos A figura abaixo mostra o gráfico da
A seguir, temos as representações gráficas velocidade escalar (v) em função do tempo
dos movimentos uniformes progressivos (t) de um móvel em movimento uniforme.
para s0 < 0, s0 = 0 e s0 > 0. A região destacada na figura é um retângulo,
Movimento uniforme progressivo, com cuja base representa o intervalo de tempo Δt
espaço inicial positivo entre t1 e t2 e a altura representa a velocidade
escalar v.

A = 𝛥s
Movimento uniforme progressivo, com
espaço inicial nulo Exemplo: (Tópicos de Física) Uma
partícula em movimento uniforme sofre
uma variação de espaço Δs = 15 m num
intervalo de tempo Δt = 3 s, como mostra
o gráfico:

Movimento uniforme progressivo, com


espaço inicial negativo No triângulo retângulo destacado, Δs está
representado pelo cateto oposto ao
ângulo α, enquanto Δt está representado
pelo cateto adjacente a α. Por ser a
Δs
velocidade escalar dada por Δt , é muito
comum dizer que ela é igual à tangente
trigonométrica de α (cateto oposto a α
dividido pelo cateto adjacente a α).

a) A velocidade escalar é igual à tangente


trigonométrica de α?

b) A velocidade escalar e a tangente


trigonométrica de α têm o mesmo valor
Propriedade do gráfico da velocidade
numérico?
escalar em função do tempo
Resolução:
a) A velocidade escalar jamais poderia ser
igual à tangente trigonométrica de α, pois
a velocidade tem uma unidade física de
medida (m/s, no caso), enquanto a
tangente é um número puro, ou seja,
adimensional.
b) Também não. Observe que:

A tangente de α, no entanto, é o quociente Traçamos a representação gráfica no


do comprimento do cateto oposto a α pelo plano x e y.
comprimento do cateto adjacente a α: A partícula percorre 4 m em cada
segundo, e o vetor posição descreve uma
área de 16 m2 de área do triângulo se:
A coincidência numérica só aconteceria se
os segmentos representativos das
unidades de s e de t tivessem a mesma d=8m
medida.

Exemplo: Duas partículas A e B estão se


movendo com MRU no plano x e y
A Lei de Kepler para MRU (positivo). As trajetórias são paralelas e
(Aprofundamento para ITA/IME) ambas têm a mesma velocidade de 5 m/s.
O vetor posição de A descreve uma área
Toda partícula que tem movimento retilíneo de 15 m2 e a de B uma área de 20 m2 em
uniforme, percorre distâncias iguais em cada segundo, em relação à origem das
tempos iguais, portanto: coordenadas.
"O vetor posição descreve áreas iguais em Determine a distância de separação entre
tempos iguais". (Lei das áreas) as trajetórias.

Resolução do professor Laerte Pereira:

Exemplo: Uma partícula está em


movimento com MRU em um plano x e y
com velocidade 4 m/s. Sabendo que o Traçamos a representação gráfica no
vetor de posição descreve uma área de 16 plano x e y.
m2 em cada segundo, determine a Cada partícula viaja 5 m em cada
distância mínima até a origem do sistema segundo.
de coordenadas.
Para o móvel A:
Resolução do professor Laerte Pereira:
Logo, d1 = 6m
Para o móvel B:
Exercícios
d2 = 8m 1. (UFPI) A figura abaixo representa quatro
Então percursos ligando as cidades A e B.
d2 – d1 = 2 m

Analise a figura e indique a alternativa


correta.
a) O caminho I é menor que o II.
b) O caminho II é menor que o III.
c) O caminho III é menor que o IV.
d) O caminho II é menor que o IV.
e) Os caminhos I, II, III e IV são de igual
tamanho.

2. Uma partícula desloca-se do ponto A até o


ponto B.

Na primeira terça parte do percurso, sua


velocidade escalar média vale v1; na
segunda terça parte, vale v2, e na terceira, v3.
Determine a velocidade escalar média no
percurso total de A até B.

3. Num dia chuvoso, um vaso cilíndrico,


inicialmente vazio, ficou exposto à chuva o
dia todo. Cessada a chuva, verificou-se que
o nível da água dentro do vaso estava a 150
mm de altura em relação ao fundo, conforme
mostra a figura.

Diz-se, então, que ocorreu uma chuva de 150


mm. Essa altura seria diferente se o vaso
cilíndrico fosse mais largo, ou seja, se o
diâmetro de sua embocadura fosse maior?

4. Um avião decola de Fernando de Noronha


às 8 horas da manhã e chega a Rio Branco,
no Acre, às 8 horas da mesma manhã!
Sabendo que a distância entre essas
localidades é de aproximadamente 3 990 km
e que o Brasil tem quatro fusos horários, 8. Dois móveis percorrem trajetórias
calcule a velocidade escalar média do avião perpendiculares, seguindo os eixos Ox e Oy,
em km/h. de acordo com as equações:

5. (Vunesp-SP) Uma caixa de papelão vazia, x = 5 + 8t (SI)


transportada na carroceria de um caminhão y = –3 + 2t (SI)
que trafega a 90 km/h num trecho reto de
uma estrada, é atravessada por uma bala válidas tanto antes como depois de t = 0.
perdida. A largura da caixa é de 2,00 m, e a Determine o instante em que a distância
distância entre as retas perpendiculares às entre os móveis é mínima.
duas laterais per furadas da caixa e que
passam, respectivamente, pelos orifícios de 9. noite, numa quadra esportiva, uma pessoa
entrada e de saída da bala (ambos na de altura h caminha em movimento retilíneo
mesma altura) é de 0,20 m. e uniforme com velocidade escalar v. Apenas
uma lâmpada L, que pode ser considerada
a) Supondo que a direção do disparo seja uma fonte luminosa puntiforme e que se
perpendicular às laterais per furadas da encontra a uma altura H do piso, está acesa.
caixa e ao deslocamento do caminhão e que
o atirador estivesse parado na estrada,
determine a velocidade da bala.
b) Supondo, ainda, que o caminhão se
desloque para a direita, determine qual dos
orifícios, A ou B, é o de entrada

6. Dois trens movem-se nos mesmos trilhos,


ambos a 45 km/h, em sentidos opostos, Determine, em função de H, h e v, a
como representa a figura: velocidade escalar média vE da extremidade
E da sombra da pessoa projetada no chão.

10. (Uerj) Uma estrada recém-asfaltada


No instante t0 = 0, correspondente à situação entre duas cidades é percorrida de carro,
durante uma hora e meia, sem parada.
da figura, uma supermosca passa a voar em
A extensão do percurso entre as cidades é
linha reta entre os trens, fazendo um vaivém
de um ao outro até ser esmagada. de, aproximadamente:
Admitindo que ela voe com velocidade de a) 103 m.
b) 104 m.
módulo constante e igual a 120 km/h,
c) 105 m.
determine:
d) 106 m.
a) o instante em que os trens colidem;
11. Um fazendeiro percorre, com seu jeep, os
limites de sua fazenda, que tem o formato de
b) a distância total percorrida pela
supermosca desde t0 = 0 até ser esmagada. um losango, com os lados aproximadamente
iguais. Devido às peculiaridades do terreno,
cada lado foi percorrido com uma velocidade
7. Um automóvel, em movimento uniforme
média diferente: o primeiro a 20KM/h, o
por uma rodovia, passou pelo km AB às 4
horas, pelo km BA às 5 horas e pelo km AOB segundo a 30 Km/h, o terceiro a 40Km/h e,
às 6 horas. Determine a velocidade escalar finalmente, o último a 60Km/h.
A velocidade escalar média desenvolvida
do automóvel. (A e B são algarismos
desconhecidos e O é o zero.) pelo fazendeiro para percorrer todo o
perímetro da fazenda, em km/h, foi de:
a) 50 velocidades escalares constantes.
b) 42
c) 38
d) 36
e) 32

12. TEXTO PARA A PRÓXIMA QUESTÃO:


Geleira gigante na Antártida pode se
desintegrar rapidamente, advertem
cientistas

Os cientistas estão alertando sobre


mudanças dramáticas em uma das maiores
geleiras da Antártida, a geleira Thwaites,
potencialmente nos próximos 5 a 10 anos.
Pesquisadores dos Estados Unidos
e do Reino Unido estão atualmente Após o início desse treinamento, no instante
envolvidos em um intenso programa de em que ocorrer o terceiro encontro entre os
estudos em Thwaites por causa de sua taxa dois, Alberto e Bernardo terão percorrido,
de derretimento. respectivamente,
Ela já está despejando 50 bilhões de a) 250 m e 750 m.
toneladas de gelo no oceano a cada ano. b) 1250 m e 3750 m.
A Thwaites é um colosso. É quase c) 1000 m e 3000 m.
do tamanho da Grã-Bretanha ou da Flórida, d) 750 m e 2250 m.
e sua velocidade de derretimento dobrou nos e) 500 m e 1500 m.
últimos 30 anos.

(Disponível em: https://www.bbc.com)


14. A agricultura de precisão reúne técnicas
De acordo com o texto, a velocidade de agrícolas que consideram particularidades
derretimento da geleira Thwaites dobrou nos locais do solo ou lavoura a fim de otimizar o
últimos 30 anos. Se for mantido esse padrão, uso de recursos. Uma das formas de adquirir
em que a velocidade de derretimento dobra informações sobre essas particularidades é a
a cada 30 anos, a velocidade de fotografia aérea de baixa altitude realizada
derretimento será 64 vezes maior do que a por um veículo aéreo não tripulado (vant). Na
atual daqui a fase de aquisição é importante determinar o
a) 150 anos. nível de sobreposição entre as fotografias. A
b) 180 anos. figura ilustra como uma sequência de
c) 210 anos. imagens é coletada por um vant e como são
d) 240 anos. formadas as sobreposições frontais.
e) 270 anos.

13. Em um dia de treinamento, dois amigos,


Alberto e Bernardo, decidem dar voltas
consecutivas em um circuito de 1000 m de
comprimento, partindo simultaneamente de
um mesmo ponto, porém movendo-se em
sentidos opostos. Alberto caminha no
sentido horário e Bernardo corre no sentido
anti-horário com velocidade três vezes maior
do que a de Alberto. Os dois mantêm suas
16. (ITA) Um motorista deseja perfazer a
O operador do vant recebe uma encomenda distância de 20 km com velocidade escalar
na qual as imagens devem ter uma média de 80 km/h. Se viajar durante os
sobreposição frontal de 20% em um terreno primeiros 15 minutos com velocidade de 40
plano. Para realizar a aquisição das km/h, com que velocidade escalar média
imagens, seleciona uma altitude H fixa de deverá fazer o percurso restante?
voo de 1.000 m, a uma velocidade
−1 a) 120 km/h
constante de 50 m s . A abertura da b) 160 km/h
câmera fotográfica do vant é de 90. c) É impossível estabelecer a velocidade
Considere tg (45) = 1. média desejada nas circunstâncias
apresentadas
Natural Resources Canada. Concepts of Aerial d) Nula
Photography. Disponível em: www.nrcan.gc.ca. e) Nenhuma das afirmações acima é correta
Acesso em: 26 abr. 2019 (adaptado).
17. (Saraeva) De Moscou a Pushkino com
um intervalo 𝑡 = 10 𝑚𝑖𝑛 saíram dois trens
Com que intervalo de tempo o operador deve elétricos com velocidades 𝑣 = 30 𝑘𝑚/ℎ. Com
adquirir duas imagens consecutivas? que velocidade 𝑢 movia-se um trem de
a) 40 segundos Pushkino em direção a Moscou, uma vez que
b) 32 segundos encontrou os trens elétricos a um intervalo 𝑡1
c) 28 segundos = 4 𝑚𝑖𝑛, um depois do outro?
d) 16 segundos
e) 8 segundos 18. (ITA) No sistema de sinalização de
trânsito urbano chamado de “onda verde”, há
semáforos com dispositivos eletrônicos que
15. Um ciclista descreve uma volta completa indicam a velocidade a ser mantida pelo
em uma pista que se compõe de duas retas motorista para alcançar o próximo sinal ainda
de comprimento L e duas aberto. Considere que de início o painel
semicircunferências de raio R conforme indique uma velocidade de 45 km/h. Alguns
representado na figura a seguir. segundos depois ela passa para 50 km/h e,
finalmente, para 60 km/h. Sabendo que a
indicação de 50 km/h no painel demora 8,0 s
antes de mudar para 60 km/h, então a
distância entre os semáforos é de:
a) 1,0 × 10−1 km.
A volta dá-se de forma que a velocidade b) 2,0 × 10−1 km.
escalar média nos trechos retos seja v e nos c) 4,0 × 10−1 km.
trechos curvos seja 2v/3. O ciclista completa d) 1,0 km.
a volta com uma velocidade escalar média e) 1,2 km.
em todo o percurso igual a 4v/5. A partir
dessas informações, é correto afirmar que o 19. (Olimpíada Ibero-americana de Física)
raio dos semicírculos é dado pela expressão: O Sr. Gutiérrez viaja todos os dias, à mesma
a) 𝐿 = 𝜋𝑅 hora, de Montevidéu a Tarariras, onde
𝜋𝑅
b) 𝐿 = 2 trabalha. O trajeto Montevidéu-Colônia é
c) 𝐿 =
𝜋𝑅 feito em trem, enquanto, de Colônia a
3
𝜋𝑅 Tarariras, o Sr. Gutiérrez viaja no carro da
d) 𝐿 = empresa que sai de Tarariras e o recolhe
4
e) 𝐿 =
3𝜋𝑅 pontualmente na estação de Colônia. Os
2 trens partem de hora em hora e demoram
sempre o mesmo tempo. Um dia o Sr. Iguais comprimentos de corda são recolhidos
Gutiérrez levantou-se mais cedo e apanhou em iguais intervalos de tempo, ou seja, a
o trem uma hora antes do costume. Quando corda é puxada com velocidade escalar
chegou a Colônia, obviamente que o carro da constante. Seja V o módulo dessa
empresa ainda não chegara; então Gutiérrez velocidade. Determine o módulo da
resolve fazer um pouco de exercício e velocidade do barco no instante em que a
começa a caminhar em direção a Tarariras. corda forma um ângulo θ com a superfície da
Em determinado momento, encontra-se com água.
o carro da empresa, que para imediatamente
e o leva para o lugar de trabalho. 22. Um homem localizado entre duas
Supondo que Gutiérrez caminha a uma montanhas grita e ouve o primeiro eco após
velocidade constante de 6,0 km/h e o carro 3 segundos e o seguinte após 3,6 segundos.
viaja a uma velocidade também constante de Qual é a separação "D" entre as montanhas?
60 km/h, calcule quanto tempo, antes do (considere a velocidade do som no ar igual a
habitual, o Sr. Gutiérrez chega à empresa. 340 m/s)
20. Uma formiga vai caminhar com
velocidade de módulo constante e igual a V,
no interior de uma caixa cúbica de aresta L.
Caminhando por paredes laterais da caixa e
também pelo seu fundo, ela vai ao vértice A
até o ponto médio, E, da aresta CD.
Determine o mínimo tempo (tmín) em que
esse percurso pode ser feito.
23. (Gupta) Um ponto percorreu metade da
distância com uma velocidade vo. A parte
restante da distância foi coberta com
velocidade v1 pela metade do tempo, e com
velocidade v2 pela outra metade do tempo.
Encontre a velocidade média do ponto em
média ao longo de todo o tempo de
movimento.
21. Um pescador P está sobre um barranco à
beira de um lago, puxando seu barco por 24. (problems in physics) Dois amigos A e
meio de uma corda amarrada em um ponto B estão correndo em uma pista circular de
A do veículo, como está representado na perímetro igual a 40 m. No instante t = 0 eles
figura: estão no mesmo local correndo no mesmo
sentido. A está correndo lentamente a uma
velocidade uniforme de 4,5 km/hr enquanto
B está correndo rapidamente a uma
velocidade de 18 km/hr. Em que instante t0
os dois amigos se encontrarão novamente?

25. (B. BUKHOVTSEV) Um veículo a motor


percorreu o primeiro terço de uma distância
s em uma velocidade v = 10 km/h, o segundo
terço a uma velocidade v = 20 km/h e o último
terço a uma velocidade v = 60 km/h.
Determinar a velocidade média do veículo ao
longo de toda a distância s.
26. Considere a escada de abrir. Os pés P e
Q se movem com velocidade constante, v.

29. (INDIANO)Dois meninos estão parados


nas extremidades A e B de um terreno onde
O intervalo de tempo decorrido, desde o AB = a. O menino em B começa a correr na
início da abertura, para que o triângulo POQ direção perpendicular a AB com velocidade
se torne equilátero será: v, O menino em A começa a correr
simultaneamente com velocidade v1 e
alcança o outro menino em um tempo, onde
t é:

a)𝑎/√𝑣 2 + 𝑣12

b)√𝑎2 /(𝑣 2 − 𝑣12 )


27. Uma partícula se move com MRU em um
plano xy, com velocidade igual a 4 m/s. c)𝑎/(𝑣 − 𝑣1 )
Sabendo que o vetor posição descreve uma
área de 12 m2 em cada segundo, determine d)𝑎/(𝑣 + 𝑣1 )
a distância mínima d à origem das
coordenadas.
30. (DESAFIO) Na figura duas formigas do
mesmo tamanho e com a mesma velocidade
v, saem de A no sentido horário, percorrendo
os quadrados A'B'C'D' e ABCD. Em que
ponto será que elas vão se encontrar pela
primeira vez? está em:
(Considere √2= 1,4)

28. Do ponto A localizado em uma rodovia


conforme mostrado na figura, deve-se
chegar de carro o mais rápido possível ao
ponto B localizado no campo distante da
rodovia. Sabe-se que o carro se move no
campo η tempo mais lento que na rodovia. A
que distância do ponto D deve-se sair da
rodovia? a) A

b) B

c) C

d)D
Gabarito e resoluções

1. Tente imaginar quadras. Você tem que


partir de uma esquina e tem que chegar em
outra esquina um pouco mais distante (da
esquina M até a esquina A por exemplo).

6
a) Como cada trem viaja a 45 km/h,
concluímos, de imediato, que eles se
aproximam 90 km em 1h.
Portanto, o instante da colisão é t = 1 h.

b) Se a supermosca sempre esteve a 120


km/h, em 1 h ela percorreu uma distância
O caminho percorrido tem o mesmo igual a 120 km.
tamanho, não importa se você pega pra
esquerda e depois pega pra direita, ou se 7. Resolução:
você vai reto até achar uma rua. Temos que:
Imagine o plano cartesiano, e cada ponto é AB = 10A + B
um cruzamento: Você partindo do ponto M BA = 10B + A
tem que chegar ao ponto A. Seguindo A0B = 100A + B
qualquer direção das setas você percorrerá Então, como o movimento é
a mesma distância. uniforme:
2. A0B – BA = BA – AB
(100A + B) – (10B + A) = (10B + A) – (10A + B)
99A – 9B = 9B – 9A
B = 6A
3. Não. Um vaso de seção transversal de Para A = 1 ; B = 6
área maior coletaria, proporcionalmente, Para A = 2 ; B = 12 (não serve)
maior quantidade de água. Assim, o nível da Portanto:
água atingiria a mesma altura. km AB = km 16
km BA = km 61
4 km A0B = km 106
Em cada hora, Δs = 45 km. Então:
v = 45 km/h

8. Resolução:

Δs = 3 990 km
Δt = 3 h

vm = 1 330 km/h

5.
Considerando que V0 seja a velocidade inicial
e que esta velocidade dobra a cada 30 anos,
podemos escrever que a velocidade V no
tempo tem anos, será dada por:
t
v = v0 2 30

Portanto:
t t t
t
64  v 0 = v 0  2 30  64 = 2 30  26 = 2 30  =6 t = 180
30

13. 1ª Solução:

De acordo com o enunciado, os módulos das


velocidades são:
 Alberto : v A = v
  No mesmo
9. Bernardo : vB = 3 v
intervalo de tempo, Bernardo percorre o triplo
da distância que Alberto.
 Alberto : dA = d
10. Resolução: 
Tratando-se de uma estrada em boas Bernardo : dB = 3 d
condições, podemos estimar a velocidade do
caro em cerca de 100 km/h: Entre dois encontros sucessivos, a soma das
Δs = vm · Δt = 100 · 1,5 distâncias percorridas é igual ao
Δs = 150 km = 1,5 · 105 m comprimento da pista (L = 1.000 m). Então,
A potência de dez que melhor se aproxima até o terceiro, esta soma é 3.000 m.
do resultado é 105 m. Assim:
3.000
dA + dB = 3.000  d + 3d = 3.000  d =  d = 750m
4
11. 32 km/h
Portanto:
12. [Resposta do ponto de vista da
disciplina de Física] Alberto : dA = d = 750m
Quando uma grandeza genérica X varia com Bernardo : dB = 3 d = 2.250m
o tempo (t), dobrando a cada período T, a
partir de um valor inicial (X0), a expressão
que relaciona essa variação é: 2ª Solução:
t
X = 2 T X0 . • De acordo com o enunciado, os módulos
das velocidades são:
Para a situação descrita, o período é T = 30  Alberto : v A = v

anos e a velocidade inicial de derretimento é Bernardo : vB = 3 v
v0.
Aplicando essa expressão acima: • Como os dois se deslocam em sentidos
t t t
V= 2 30 V0  64 V0 = 2 30 6
V0  2 = 2 30  6=
t
30
 t = 180anos opostos, o módulo da velocidade relativa
entre eles é igual à soma dos módulos das
[Resposta do ponto de vista da disciplina velocidades.
de Matemática] vrel = v A + vB = v + 3 v  vrel = 4 v
16. A distância total é de 20 km. Se ele viajar
durante 15 min com velocidade de 40 km/h,
• Entre dois encontros sucessivos, a soma ele deslocará 5 km. Logo, restarão 15 km.
das distâncias percorridas é igual ao Nestes últimos 15 km ele deverá andar a
comprimento da pista (L = 1.000 m). uma velocidade que compensará o fato dele
andar mais lento no primeiro trecho de forma
• Tempo para o 1º encontro (t1): que a velocidade escalar média (𝑣𝑚) dele
L L 1.000 250 nos 20 km seja 80 km/h. Então, vamos
vrel =  t1 = =  t1 = calcular 𝑣𝑚:
t1 vrel 4v v

• Tempo até o 3º encontro (t3):


250 750
t3 = 3 t1 = 3  t3 =
v v

• Calculando as distâncias percorridas: 17. A distância entre os dois trens quando


 750 saem de Moscou para Pushkino é dado por:
 dA = v t 3 = v  dA = 750m
 v

d = 3 v t = 3 v 750  d = 2.250m


B 3
v
A
Agora, vamos colocar nosso referencial no
segundo trem que vai de Moscou a Pushkino,
14. Analisando dois triângulos sobrepostos, então o trem que está indo para Moscou
temos: passa a ter a velocidade 𝑢+30. Ele leva 4 min
para ir de um trem ao outro. Logo:

18. Considere que um primeiro carro vai do


primeiro para o segundo semáforo com
velocidade de 45 km/h. Então, podemos
dizer que a distância entre os semáforos
1000
será:
tg45 =  L = 2000 m 𝑑 = 45. 𝑇 (1)
L2
Um segundo carro, andando a 50 km/h passa
pelo primeiro semáforo (indicação do painel)
Distância percorrida pelo avião entre duas
e dentro de 8 segundos o painel alterará sua
fotos:
velocidade para 60 km/h. Portanto, o
d = 0,8  2000 m = 1600 m 8
segundo carro terá T - 3600 para percorrer a
Portanto, o intervalo de tempo procurado é mesma distância com velocidade de 50
de: km/h. Então, podemos escrever a distância
d 1600 m percorrida pelo segundo carro:
Δt = =
v 50 m s
 Δt = 32 s
Isolando 𝑇 em (1) e substituindo em (2),
temos:
15. 𝑳=𝝅𝑹
26. Gabarito B.

27. A lei de Kepler: o móvel percorre áreas iguais


19. Resolução do Prof. Laerte Pereira: em tempos iguais, no que diz respeito à origem
𝑣𝑟𝑒𝑙 = 66 𝑘𝑚/ℎ das coordenadas.
60 A partícula percorre 4 m em cada segundo.
∆𝑡 = ≌ 0,909 ℎ
66
Ida + volta = 1,8181 h
d=6m
Se Gutiérrez estivesse parado, ∆𝑡 ′ = 2ℎ
Fazendo a diferença 28. Suponha que a x distância do ponto D o carro
2h - 1,8181 h = 0,1818...h sai da rodovia. Que v seja a velocidade do carro
na rodovia, então sua velocidade no campo será
10,9 minutos v/η. O tempo de movimento será dado por

37.𝐿
20. √2.𝑣
Diferenciando em relação a x
21. 𝑣⁄cos 𝜃

22. D = 1122 m

23.

29. Os Dois garotos devem se encontrar no


24.
ponto C após o tempo t desde o início.
𝑣𝐴 = 1,25 𝑚/𝑠
𝑣𝐵 = 5 𝑚/𝑠
B completa uma volta em 8s e, nesse tempo A
anda 10m. Então AC = vt, BC = v1 t
𝑑𝐵 = 5. ∆𝑡 (AC)2 = (AB)2 + (BC)2
 v 2 t 2 = a2 + v12 t 2
Mas, depois que B completar uma volta, pra B
a2
alcançar A ele terá que andar t = √v2 −v2
1
𝑑𝐵 = 10 + 1,25. ∆𝑡
30. Resolução do professor Laerte Pereira:
10 + 1,25. ∆𝑡 = 5. ∆𝑡
Na figura, a primeira formiga (f1) passa por
8 A'B'C'D' e a segunda (f2) por ABCD, então os
∆𝑡 = 𝑠 encontros possíveis só podem ocorrer nos
3
pontos A, B, C ou D Agora, suponha que o
8 32
𝑡0 = 8 + = 𝑠 primeiro encontro ocorre depois de decorrido
3 3 um tempo "t", então durante esse tempo as
Gabarito
32
𝑠 distâncias percorridas pelas formigas (f₁) e
3 (f2) são:

25. Gabarito18. km/h d₁ = p (2a) = v.t, eq. (1)


d₂ = q (√2 a) = vt eq. (2) Movimento Uniformemente variado
com p e q ∈ Z.
Dividindo (2) por (1): Objetivos
1,4. aq Reconhecer conceitos básicos para a análise
=1 de movimento uniformemente variado.
2. ap
Logo Representar graficamente variáveis de
q 10 movimento uniformemente variado em
= eq 3 função do tempo.
p 7

Na relação (3), p e q podem assumir apenas Introdução


os valores (7;14;21;...) e (10;20;30;...) Denomina-se movimento uniformemente
respectivamente, usando (1), (2) e com a variado (em qualquer trajetória) todo aquele
ajuda da Figura fazemos uma tabela das em que a velocidade escalar varia
distâncias percorridas e as posições para os uniformemente com o passar do tempo, isto
diferentes valores de p e q: é, sofre variações iguais em intervalos de
distancia Posição tempo iguais.
p Q recorrida da
d1 d2 formiga
1ª 2ª
7 10 14a 14a D C
14 20 28a 28a C A
21 30 42a 42a B C
28 40 56a 56a A A Aceleração
A taxa de variação temporal da velocidade de
A partir disto, pode-se ver que as formigas um objeto é chamada de aceleração do
vão se encontrar pela primeira vez no ponto objeto. É uma quantidade vetorial. A sua
A. direção é a mesma da mudança de
velocidade. Tem dimensão [M0 L1 T-2],
unidade de metro/segundo2 (S.I.).
Aceleração uniforme
Diz-se que um corpo tem aceleração
uniforme se a magnitude e a direção da
aceleração permanecerem constantes
durante todo o movimento das partículas.
Aceleração média é calculada

⃗⃗ v
Δv ⃗⃗2 − v⃗⃗1
a⃗⃗m = =
Δt Δt

Função da velocidade escalar


A expressão que fornece a velocidade
escalar v num instante t qualquer do
movimento é denominada função da
velocidade escalar.

v = v0 + a.t
Gráfico da velocidade escalar em função Propriedade do gráfico da aceleração
do tempo escalar em função do tempo
A seguir, temos as representações gráficas Considere um movimento uniformemente
vxt, quando a aceleração escalar é positiva, variado (MUV) com aceleração escalar (a)
para v0 < 0, v0 = 0 e v0 > 0. positiva. A figura a seguir mostra o gráfico
axt.

Dizemos, então, que a área A, em destaque


na imagem, delimitada pela aceleração a e
pelos instantes t1 e t2, é uma medida da
variação de velocidade escalar da partícula.

A = Δv

Propriedade média da velocidade escalar

Gráfico da aceleração escalar em função


do tempo
v1 + v2 Δs
Em um movimento uniformemente variado vm = =
(MUV), a força resultante e a aceleração 2 Δt
escalar é constante e diferente de zero, e
pode ser positiva ou negativa. Função da posição no MUV
A equação horária dos espaços de um móvel
em MUV é dada por:

a. t 2
s = s0 + v0 t +
2

Gráfico do espaço em função do tempo


Considere as representações gráficas sxt,
quando a > 0.
Como condição do problema, A tem MRU
ao norte. Portanto, para um tempo t dado:
dA = v.t
Ao mesmo tempo o móvel C terá saído do
repouso rumo ao sul, e com aceleração
MRUV a, e ao mesmo tempo que A e, terá
se deslocado:
𝑎. 𝑡 2
𝑑𝑐 =
2
Fazendo uma figura para localizar os
móveis A e C, observamos que na linha

que os une, deve-se localizar o móvel B,


que deve ser movido para o norte, e terá
percorrido a distância dB no tempo t. A
seguir, usando o teorema de Tales dos
pontos médios, diremos que:

dB + dc=dA-dB

(dA − dc )
dB =
2
Exemplo: Três carros A, B e C estão em
linha reta, estando B à mesma distância v 1 (−a) 2
de A e C. Se o carro A se desloca para o dB = .t + .t
2 2 2
norte com velocidade constante v, e o
carro C parte do repouso para o sul com Onde
aceleração constante a, como o carro B V0B = v/2
deve se mover paralelo a A para que os aB = - a/2 (Movimento desacelerado).
três estejam sempre na mesma linha reta?
U = uniforme, A = acelerado, D =
desacelerado. Exemplo: (Enem 2017) Um motorista que
atende a uma chamada de celular é
Resolução do professor Laerte Pereira: levado à desatenção, aumentando a
possibilidade de acidentes ocorrerem em
razão do aumento de seu tempo de
reação. Considere dois motoristas, o
primeiro atento e o segundo utilizando o
celular enquanto dirige. Eles aceleram
2
seus carros inicialmente a 1,00 m s . Em
resposta a uma emergência, freiam com
2
uma desaceleração igual a 5,00 m s , O
motorista atento aciona o freio à
velocidade de 14,0 m s, enquanto o
desatento, em situação análoga, leva
1,00 segundo a mais para iniciar a
Como são trajetórias paralelas, os móveis frenagem.
manterão sempre a mesma separação Que distância o motorista desatento
lateral, que neste caso é dada por m. percorre a mais do que o motorista atento,
até a parada total dos carros? Exemplo: A aceleração no sentido da
a) 2,90 m velocidade de um carro equipado com um
b) 14,0 m motor turbo, ao longo de uma pista
c) 14,5 m orientada, tem valor escalar, a, variando
em função do tempo, t, conforme o gráfico
d) 15,0 m
a seguir.
e) 17,4 m

Resolução:

Para o motorista atento, temos:

Tempo e distância percorrida até atingir


14 m s a partir do repouso: Sabendo-se que no instante t0 = 0 a
v = v 0 + at velocidade escalar do veículo é de 10,0
m/s. Determine a distância D percorrida
14 = 0 + 1 t1  t1 = 14 s
pelo carro no intervalo de t0 = 0 a t = 12,0
v 2 = v 02 + 2aΔs s.
142 = 02 + 2  1 d1  d1 = 98 m
Resolução:
De t0 = 0 a t = 4,0 s, a aceleração escalar
Distância percorrida até parar:
é constante e o movimento é
02 = 142 + 2  (−5)  d1 '  d1 ' = 19,6 m uniformemente variado (acelerado). Isso
não ocorre, porém, entre t = 4,0 s e t = 12,0
Distância total percorrida: s, em que a aceleração escalar cresce
Δs1 = d1 + d1 ' = 98 + 19,6  Δs1 = 117,6 m segundo um gráfico reto. Nesse caso, o
movimento é variado (acelerado), mas
Para o motorista que utiliza o celular, não uniformemente. Podemos, no
temos: entanto, determinar uma aceleração
t2 = t1 + 1  t 2 = 15 s escalar média para esse intervalo, que é a
média aritmética das acelerações
Velocidade atingida e distância percorrida escalares nos instantes extremos dele
em 15 s a partir do repouso: (propriedade da função afim).
v 2 = 0 + 1 15  v 2 = 15 m s
Essa aceleração escalar média se
comporta como uma aceleração escalar
152 = 02 + 2  1 d2  d2 = 112,5 m constante entre t = 4,0 s e t = 12,0 s.

Distância percorrida até parar:


02 = 152 + 2  (−5)  d2 '  d2 ' = 22,5 m
Um gráfico alternativo que equivale à
situação proposta está esboçado a seguir:
Distância total percorrida:
Δs2 = d2 + d2 ' = 112,5 + 22,5  Δs2 = 135 m

Portanto, a distância percorrida a mais


pelo motorista desatento é de:
Δs = Δs2 − Δs1 = 135 − 117,6
 Δs = 17,4 m
𝟐. 𝐝
𝐭𝐞 = √
𝐚𝟏 + 𝐚𝟐

Instante de alcance (ta)


Dois móveis, separados por um d de
distância, que começam do repouso ao
mesmo tempo, e mover-se na mesma
direção e sentido, e um, vai para o alcance
do outro. O instante de alcance é

D = 256 m

Exemplo: (Halliday)Um elétron com


velocidade inicial v0 = 1,50 × 105 m/s
penetra em uma região de comprimento L
𝟐. 𝐝
= 1,00 cm, em que é eletricamente 𝐭𝐚 = √
acelerado, e sai da região com v = 5,70 × 𝐚𝟏 − 𝐚𝟐
106 m/s. Qual é a aceleração do elétron,
supondo que seja constante? Com a1 > a2

Exemplo: Calcule o tempo de encontro se


ambos os móveis começam a partir do
repouso.

Resolução:
Da função
Resolução:
𝟐. 𝐝
𝐭𝐞 = √
𝐚𝟏 + 𝐚𝟐

𝟐. 𝟐𝟎𝟎
𝐭𝐞 = √
𝟏+𝟒
10s.
Instante de encontro (te)
Dois móveis, separados por uma distância d, Função de Torricelli
partem do repouso simultaneamente (ao A função de Torricelli é uma função do
mesmo tempo), e se movem na mesma movimento uniformemente variado (MUV)
direção e em sentidos opostos, um para que permite determinar a velocidade escalar
encontrar o outro. de um móvel sem que o tempo seja
conhecido.

v2 = v20 + 2aΔs

Números De GALILEO e M.U.V.


Galileu Galilei estabeleceu que se todos os
corpos que parte do repouso com aceleração Outra maneira de provar essa propriedade e
constante, terá a característica de viajar, em usando a função da posição do
tempos iguais, distâncias proporcionais aos MRUV. Veja:
números impares 1; 3; 5; 7, ..., (2n- 1); estes
números são conhecidos como os Números
Galileo.

Queda livre e lançamento vertical para


cima
Nas proximidades do solo,
independentemente de suas massas, formas
ou materiais, todos os corpos em queda livre
caem verticalmente com a mesma
aceleração: a aceleração da gravidade, g.
Para calcular o deslocamento escalar desse
corpo utilizamos a equação:
g 2
Δs = t
2
Para o cálculo do tempo de subida utilizamos
a equação:
v0
T=
g

E para encontrar a altura máxima:


v2
Hmáx
2. g

Propriedade para corpos em queda livre -


Proporção de Galileu
Um corpo em queda livre percorre, em
intervalos de tempos iguais, distancias
proporcionais aos números impares, isto e,
1d, 3d, 5d e assim por diante. Lembre-se que
no gráfico da velocidade em função do tempo
a área é numericamente igual ao Exemplo: Num edifício pode-se notar um
deslocamento. aparelho de ar-condicionado instalado a
80 m de altura. Esse aparelho goteja água
periodicamente, em intervalos de tempos
iguais.
Percebe-se que a 1ª gota atinge o solo no
exato momento em que a 5ª gota se
desprende. Determine no referido
instante, a distância que separa a 1ª da 2ª
gota, a 2ª da 3ª gota, a 3ª da 4ª gota e a
4ª da 5ª gota.

A1 + A2 + A3 = 270
A1 + 3A1 + 5A1 = 270 ⇒ 9A1 = 270 ⇒
A1 = 30 m

Respostas: 30 m, 90 m e 150 m

Exemplo: Uma torneira libera gotas em


intervalos iguais de tempo. As gotas
abandonam a torneira com velocidade
nula. Considere desprezível a resistência
do ar. A figura abaixo mostra uma
representação instantânea das cinco
primeiras gotas.

Podemos observar que:

d + 3d + 5d + 7d = 80
16d = 80 ⇒ d = 5m

Logo:
A distância que separa a 1ª da 2ª gota =
7d =35m
A distância que separa a 2ª da 3ª gota =
5d=25m
A distância que separa a 3ª da 4ª gota = As distâncias d1 e d2 indicadas valem
3d=15m respectivamente:
A distância que separa a 4ª da 5ª gota =
d=5m a) 6 cm e 2 cm d) 10 cm e 13 cm
b) 8 cm e 10 cm e) 10 cm e 14 cm
c) 10 cm e 12 cm
Exemplo: Uma partícula é abandonada a
partir do repouso, de um ponto situado a Resposta: e
270 m acima do solo. Divida essa altura
em três partes tais que sejam percorridas
em intervalos de tempo iguais. Aprofundamento Ime/ITA (Distância
percorrida durante o enésimo segundo de
Resolução: queda livre.)
Usando o gráfico v × t: Observemos que:
1ª segundo equivale a: 0 – 1s
2ª segundo equivale a: 1s - 2s
3ª segundo equivale a: 2s - 3s
...
nº segundo equivale a: (n-1)s-(n)s Exemplo: Exemplo (ITA) Um corpo cai em
Sendo queda livre de uma altura tal que, durante
𝑔. 𝑡 2 o último segundo de queda, ele percorre
𝑦= 1/4 da altura total. Calcule o tempo de
2
, observemos que: queda supondo nula a velocidade inicial
Para t1 = n - 1, obtemos: do corpo.
𝑔.(𝑛−1)2 1
𝑦1 = A) (2−√3) 𝑠
2
Para t2 = n, obtemos:
2
𝑔. 𝑛2 B) (2−√3) 𝑠
𝑦2 =
2
Assim, para o espaço percorrido durante o 3
C) (2−√3) 𝑠
enésimo segundo de queda, teremos:
𝑔. 𝑛2 𝑔. (𝑛 − 1)2
𝛥𝑦 = − 4
D) (2−√3) 𝑠
2 2
𝑔. 𝑛2 𝑔. (𝑛)2 + 1 − 2𝑛
𝛥𝑦 = − 2
2 2 E) (2+√3) 𝑠
𝑔. (2𝑛 − 1)
𝛥𝑦 =
2
Resolução do Prof. Laerte Pereira:
Aprofundamento Ime/ITA (Tempo Fracione a queda em distâncias Iguais.
transcorrido durante o enésimo metro de
queda livre.)
Observemos que:
1º metro equivale a: 0 - 1m
2º metro equivale a: 1m - 2m
3º metro equivale a: 2m - 3m
...
nº metro equivale a: (n - 1)m - (n)m
Sendo
𝑔. 𝑡 2
𝑦=
2
, observemos que:
Para y1 = n -1, obtemos:
2
𝑔.𝑡𝑛−1 2(𝑛−1)
𝑛−1= 𝑙𝑜𝑔𝑜, 𝑡𝑛−1 = √ ∆𝑡4 = (2 − √3) ∆𝑡1 = 1𝑠
2 𝑔
Para y2 = n, obtemos: 1
∆𝑡1 = 𝑠
𝑔. 𝑡𝑛2 2𝑛 (2 − √3)
𝑛= 𝑒𝑛𝑡ã𝑜, 𝑡𝑛 = √ Logo
2 𝑔
𝛥𝑡𝑡𝑜𝑡𝑎𝑙 = ∆𝑡1 + ∆𝑡2 + ∆𝑡3 + ∆𝑡4
Assim, para o tempo transcorrido durante o
enésimo segundo de queda, teremos: 𝛥𝑡𝑡𝑜𝑡𝑎𝑙 = ∆𝑡1 + (√2 − 1)∆𝑡1
2𝑛 2(𝑛 − 1) + (√3 − √2)∆𝑡1
𝛥𝑡 = √ −√
𝑔 𝑔 + (2 − √3)∆𝑡1
Então 𝛥𝑡𝑡𝑜𝑡𝑎𝑙 = 2. ∆𝑡1
2
2 𝛥𝑡𝑡𝑜𝑡𝑎𝑙 = 𝑠
𝛥𝑡 = √ . (√𝑛 − √𝑛 − 1) (2 − √3)
𝑔
2ª Resolução: b) durante o 5º segundo de queda.

Se no último segundo de queda ele Resolução:


percorreu 1/4 da altura total, quer dizer
que no tempo t-1 ele percorreu 3/4 da
altura total:
a) Os últimos 4 segundos iniciam-se em t1
= 6 s e terminam em t2 = 10 s. Calculemos
as velocidades escalares em t1 = 6 s e em
t2 = 10 s, considerando a trajetória do
corpo orientada para baixo
(α = g):
v = v0 + α t
v = 10t
Nós achamos pela equação horária do Em t1 = 6 s, temos: v1 = 10 · 6 ⇒
percurso de 3/4H o próprio H. Agora, v1 = 60 m/s
igualaremos pela equação horária da Em t2 = 10 s, temos: v2 = 10 · 10 ⇒
altura total H: v2 = 100 m/s. Aplicando a função de
Torricelli entre t1 e t2, vem:
Δs = 320 m
Logicamente H = H, então: b) O 5º segundo inicia-se em t’1 = 4 s e
termina em t’2 = 5 s.
Em t’1 = 4 s, temos: v’1 = 10 t’1 = 10 · 4 ⇒
v’1 = 40 m/s
Em t’2 = 5 s, temos: v’2 = 10 t’2 = 10 · 5 ⇒
v’2 = 50 m/s. Aplicando a função de
Torricelli, vem:
Δs’ = 45 m

Exemplo: Um bloco de chumbo cai do topo


de uma torre. Considerando desprezível a
influência do ar e sendo g a intensidade do
campo gravitacional, calcule a distância
percorrida pelo bloco durante o enésimo
segundo de queda livre.

Resolução:

Exemplo: Um corpo com velocidade inicial


nula cai no vácuo durante 10 s. Sendo g =
10 m/s2, determine a distância percorrida
pelo corpo:
a) durante os últimos 4 segundos de v = v0 + g t ⇒ v = gt
queda; vn – 1 = g.(n – 1)
vn = g.n
EXERCÍCIOS
v2n = v2n – 1+ 2gd
(gn)2 = [g(n – 1)]2 +2gd 1. (Esc. Naval 2022) Observe a figura abaixo.
g.n2 = (gn)2 – 2gn + g + 2d

Uma pequena pedra é lançada verticalmente


para baixo de cima de uma passarela de
altura H = 8,75m, a uma velocidade de
15,0m s, para atingir um automóvel de
comprimento L = 3,5 m, que trafega em uma
rodovia plana se aproximando da passarela.
No instante em que a pedra é lançada, o
automóvel está com uma velocidade de
12,0m s e aceleração constante de

4,0m s2 . Sabendo que a pedra atingiu a


rodovia imediatamente após a traseira do
automóvel ter passado pelo ponto de colisão
entre a pedra e a rodovia, qual a distância D
em que o automóvel se encontra da
passarela no instante do lançamento?

2
Dado: g = 10m s
a) 2,0 m
b) 2,5 m
c) 3,0 m
d) 4,5 m
e) 6,5 m

2. (Epcar (Afa) 2023) Duas partículas, A e B,


se movem, em sentidos opostos, em uma
mesma trajetória.
No instante t0 = 0, a partícula A inicia do
repouso e da origem dos espaços um
movimento uniformemente variado, e a
partícula B passa pela posição 3,0 m com
velocidade constante, permanecendo em
movimento uniforme.
No instante t = 2 s, as duas partículas, A e B,
se encontram, tendo a partícula B percorrido
uma distância igual a duas vezes a distância
percorrida pela partícula A, conforme indica
figura a seguir:

Nessas condições, a velocidade da partícula


A, em m/s, no momento em que as partículas
se encontram, é igual a
a) 0,25
b) 0,50 Um projetil atinge um colete balístico sem
c) 1,0 perfurá-lo. A ação da fibra do tecido balístico
d) 2,0 no projetil está representada na figura acima.
A deformação da fibra é transmitida pela
3 (Efomm 2017) Um trem deve partir de uma propagação de pulsos longitudinais e
estação A e parar na estação B, distante transversais que se afastam radialmente do
ponto de impacto, em que o projetil produz
4 km de A. A aceleração e a desaceleração
uma deformação em forma de cone no
2
podem ser, no máximo, de 5,0 m s , e a tecido. O pulso longitudinal, que se propaga
maior velocidade que o trem atinge é de ao longo da fibra, faz com que ela se
72 km h. O tempo mínimo para o trem
deforme, afinando na direção radial. O pulso
transversal, que se propaga com velocidade
completar o percurso de A a B é, em menor que a velocidade longitudinal, está
minutos, de: associado à depressão. À medida que o
a) 1,7 projetil penetra no tecido, o raio r da
b) 2,0 depressão aumenta fazendo com que o
c) 2,5 material do colete se mova na mesma
d) 3,0 direção do projetil, mantendo o ângulo θ.
e) 3,4 Sabe-se que a velocidade do projetil logo
após atingir o colete é dada pela função
4(Efomm 2016) Um automóvel, partindo do 6
horária v(t) = 250 − 5  10 t [m s].
2
repouso, pode acelerar a 2,0 m s e
2 Dados:
desacelerar a 3,0 m s . O intervalo de
- velocidade do projetil antes do impacto:
tempo mínimo, em segundos, que ele leva 250 m s;
para percorrer uma distância de 375 m, - velocidade do pulso longitudinal na fibra:
retornando ao repouso, é de 2000 m s; e
a) 20 - ângulo θ = 60.
b) 25
c) 30 No instante em que a velocidade do projetil
d) 40 for nula, os raios aproximados das regiões
e) 55 deformadas pelo pulso transversal (r) e pelo
longitudinal (R), são, respectivamente:
5 (Ime 2021)
a) 0,1 e 0,01
b) 0,01 e 0,01
c) 0,01 e 0,1 c) 2  v0 + t
d) 0,1 e 0,001 d) v0 − 5t
e) 0,001 e 0,1
8(Ita 2016) A partir do repouso, um foguete
6(Epcar (Afa) 2021) A partir do instante de brinquedo é lançado verticalmente do
t0 = 0, uma partícula com velocidade inicial chão, mantendo uma aceleração constante
v 0 é uniformemente acelerada. 2
de 5,00 m s durante os 10,0 primeiros
No instante t, a aceleração cessa e a
segundos. Desprezando a resistência do ar,
partícula passa a se movimentar com
a altura máxima atingida pelo foguete e o
velocidade constante v. Do instante 2t ao
tempo total de sua permanência no ar são,
instante 4t, uma nova aceleração constante respectivamente, de
atua sobre a partícula, de tal forma que, ao a) 375 m e 23,7 s.
final desse intervalo, sua velocidade vale −v.
b) 375 m e 30,0 s.
Nessas condições, o módulo da velocidade c) 375 m e 34,1s.
média (𝑣⃗𝑚 ) da partícula, no intervalo de 0 a d) 500 m e 23,7 s.
4t, é igual a e) 500 m e 34,1s.
a) 5v − v 0
3v + v 0 9(Ime 2013) Existe um intervalo mínimo de
b)
8 tempo entre dois sons, conhecido como
2v + 4v 0 limiar de fusão, para que estes sejam
c) percebidos pelo ouvido humano como sons
3
d) v + 2v0 separados. Um bloco desliza para baixo, a
partir do repouso, em um plano inclinado com
7(Epcar (Afa) 2020) Em um local onde a ressaltos igualmente espaçados que
aceleração da gravidade é g, as partículas produzem ruídos. Desprezando o atrito do
bloco com o plano inclinado e a força
idênticas, 1 e 2, são lançadas exercida pelos ressaltos sobre o bloco,
simultaneamente, e sobem sem atrito ao determine o limiar de fusão τ de uma pessoa
longo dos planos inclinados AC e BC, que escuta um ruído contínuo após o bloco
respectivamente, conforme figura a seguir. passar pelo enésimo ressalto.
Observação: Despreze o tempo de
propagação do som.
Dados: ângulo do plano inclinado com a
horizontal: θ; aceleração da gravidade: g;
distância entre os ressaltos: d.

10 (ITA) Um móvel A parte da origem O com


velocidade nula no instante t0 = 0 e percorre
A partícula 2 é lançada do ponto B com o eixo Ox com aceleração constante a. Após
velocidade v 0 e gasta um tempo t para um intervalo de tempo Δt contado a partir da
saída de A, um segundo móvel B parte de O
chegar ao ponto C. com uma aceleração igual a n.a, sendo n>1.
B alcançará A no instante:
Considerando que as partículas 1 e 2
colidem no vértice C, então a velocidade de
lançamento da partícula 1 vale
a) 3  v0 − 5t
b) 3  v0 − t
a) a aceleração do móvel em t = 2s;
b) a distância que ele percorre durante todo
o processo de frenagem.

13 DESAFIO. Tente resolver este exercício


sem usar Cálculo.
(Ita 2018) Os pontos no gráfico indicam a
velocidade instantânea, quilômetro a
11 (ITA) De uma estação parte um trem A quilômetro, de um carro em movimento
com velocidade VA = 80 km/h. Depois de um retilíneo. Por sua vez, o computador de bordo
certo tempo, parte desta mesma estação um do carro calcula a velocidade média dos
outro trem B com velocidade VB = 100 km/h, últimos 9 km por ele percorridos. Então, a
no mesmo sentido de A e sobre os mesmos curva que melhor representa a velocidade
trilhos. Depois de certo tempo de percurso, o média indicada no computador de bordo
maquinista de B verifica que seu trem se entre os quilômetros 11 e 20 é
encontra a 3 km de A.
A partir desse instante ele aciona os freios,
comunicando ao trem B uma aceleração a =
– 50 km/h2.
Nestas condições:

A) não houve encontro dos trens.

B) depois de 2 h, o maquinista do trem B nota


que a distância que o separa de A é de 64 a) a tracejada que termina acima de
50 km h.
km.
b) a cheia que termina acima de 50 km h.
C) houve encontro dos trens depois de 12 c) a tracejada que termina abaixo de
min. 50 km h.
d) a pontilhada.
D) houve encontro dos trens depois de 36 e) a cheia que termina abaixo de 50 km h.
min.
14. (Gupta) Um carro acelera a partir do
E) n.d.a. repouso a um ritmo constante α por algum
tempo, após o qual desacelera a um ritmo
12 Um móvel que se desloca ao longo do constante β para chegar ao repouso. Se o
eixo x inicia um processo de frenagem ao tempo total decorrido for t segundo, calcule
passar pela posição x = 0 no instante t = 0 s, então:
movendo-se de acordo com o gráfico a
seguir. a) a velocidade máxima atingida pelo carro,
e;

b) o deslocamento total percorrido pelo carro


em termos de α, β e t.

15. Um veículo está parado ao lado do marco


que indica “km 20” (o marco “km 0” fica em
Fortaleza, no bairro Aerolândia) da rodovia
BR 116, que liga Fortaleza ao Sul do Brasil.
Determine: No instante t = 0, o veículo começa a se
mover, afastando-se de Fortaleza. O gráfico c) 72 m.
abaixo mostra como varia sua velocidade d) 96 m.
escalar em função do tempo. Ao lado de que e) 120 m.
marco estará o veículo após se mover
durante 60 segundos? 18. São dados a seguir os gráficos referentes
aos movimentos de dois veículos A e B. O
gráfico de A é um arco de parábola com
vértice em t = 0.

16. Numa corrida de 100 m rasos, a Calcule a velocidade escalar de A em t = 2 s.


velocidade escalar de um atleta variou com o
tempo, aproximadamente, conforme o 19. (Moysés adaptada) Uma pessoa na
gráfico: calçada de um apartamento lança
verticalmente para cima uma bola com
velocidade inicial desconhecida no instante t
= 0s. Uma senhora, que está a uma altura H
do solo, presencia a passagem da bola pela
sua janela nos instantes t1 e t2. Determine
quanto vale a aceleração da gravidade local
g em função de t1, t2 e H.

Sabendo que esse atleta concluiu a prova 20. (Renato Brito) Qual o tempo necessário
em 10,0 s, faça uma estimativa (cálculo para um corpo, abandonado do repouso, em
aproximado) do valor de sua velocidade queda livre, percorrer o n-ésimo metro do seu
máxima V. trajeto?

17. Gustavo, estudando o movimento 21. Um móvel tem movimento


retilíneo de um pequeno corpo, a partir do uniformemente acelerado com velocidade
repouso, verifica que a aceleração escalar inicial V0 e aceleração a. Determine a
varia com o tempo de acordo com o gráfico distância percorrida pelo móvel no n-ésimo
dado. O espaço efetivamente percorrido pelo segundo do movimento.
móvel nos primeiros 10 s de movimento é:
22 (Olimpíada de Física da Bolívia) Deixa-
se cair, a partir do repouso, cinco esferinhas
intercaladas por intervalos de tempos iguais,
desde o topo de um edifício de altura H num
local onde g = 10 m/s2. Quando a primeira
esferinha atinge o solo, a quinta está
iniciando a sua queda e, nesse instante, a
distância entre a segunda e a terceira
esferinha vale 5 m. A altura H vale:
a) 24 m. a)5m
b) 48 m. b)10m
c)16m Partindo da estação A, o tempo necessário
d)20m e o espaço percorrido até o trem atingir a
e)24m velocidade máxima de 72 km h (20 m s)
são:
GABARITO Δv 20 − 0
a= 5=  Δt1 = 4 s
Δt1 Δt1
1
Tempo decorrido desde o lançamento da v 2 = v 02 + 2aΔs1  202 = 02 + 2  5  Δs1  Δs1 = 40 m
pedra até a sua colisão com a rodovia:
gt 2 Da mesma forma, depois de atingida a
H = v 0P t +
2 velocidade máxima, no último trecho o trem
10t 2 gastará o mesmo tempo e percorrerá a
8,75 = 15t + mesma distância até parar. Logo: Δt3 = 4 s
2
t 2 + 3t − 1,75 = 0 e Δs3 = 40 m.
t = 0,5 s
Para o trecho intermediário, o trem deve
Do movimento do automóvel, obtemos: desenvolver uma velocidade constante igual
à máxima para que o tempo de percurso seja
at 2
L + D = v 0C t + mínimo. Desse modo:
2
Δs2 = 4000 − 2  40  Δs2 = 3920 m
4  0,52
3,5 + D = 12  0,5 + Δs2 3920
2 v=  20 =  Δt 2 = 196 s
Δt 2 Δt 2
3,5 + D = 6 + 0,5
D = 3 m
Portanto, o tempo total será:
Δt = Δt1 + Δt 2 + Δt3 = (4 + 196 + 4) s = 204 s
2 Como a partícula B percorre o dobro da
 Δt = 3,4 min
distância da partícula A até o encontro, o
espaço percorrido por esta última terá sido
de: 4
sA + sB = 3
Dividindo o movimento em duas partes, de
acordo com o gráfico, temos:
sA + 2sA = 3
sA = 1m

Da equação horária do espaço da partícula


A, obtemos o valor da sua aceleração:
aA t2
s A = s0A + v 0A t +
2
a A  22
1= 0 + 02 +
2
a A = 0,5 m s2 As equações da velocidade para o trecho 1 e
2, são:
Logo, a velocidade da partícula A no v1 = 2t1
momento do encontro é igual a: v1 = 3 ( t 2 − t1 )  v1 = 3t 2 − 3t1
v A = v 0A + aA t
v A = 0 + 0,5  2
Juntando as duas equações:
 v A = 1m s 3
2t1 = 3t 2 − 3t1  t1 = t2
5
3
Logo, usando as equações para o cálculo da Δs ( 3v + v 0 ) t 1
vm = = 
área dos triângulos juntos, temos o Δt 2 4t
deslocamento do móvel em todos os trechos: 3v + v 0
t v t  2t1  vm =
Δs = Δs1 + Δs2 = 2 1  375 = 2 8
2 2
3 7
t2  2  t2
375 = 5  t 2 = 625  t = 25 s
2
2 2 Aceleração das partículas 1 e 2 :
1
mgsen30 = ma1  10  = a1  a1 = 5 m s2
2
5
No instante em que a velocidade do projétil mgsen60 = ma2  10 
3
= a2  a2 = 5 3 m s2
se anula, temos: 2
0 = 250 − 5  106 t  t = 5  10 −5 s
Deslocamentos das partículas 1 e 2 :
R = vpulso  t = 2000  5  10 −5
5t 2
 R = 0,1m Δs1 = v 01 t −
2
5 3t 2
6 2 Δs2 = v 0 t −
Como v0 = 250 m s e a = −5  10 m s , a 2
distância percorrida pelo projétil é de:
v 2 = v 02 + 2aΔs  0 = 2502 + 2  ( −5  106 ) Δs  Δs = 6,25  10 −3 m Da geometria da figura, temos que:
5t 2
Δs1 v 01 t − 2 2
tg60 =  3= 2  v t − 5t = 3v t − 15t
Da figura, obtemos: Δs2
v0 t −
5 3t 2 01
2
0
2
r r 2
tg60 =  3=  v 01 = 3v 0 − 5t
Δs 6,25  10−3
 r = 0,01m

6 Deslocamento durante o percurso: 8

H0 = 0 m
t1 = 10 s
a1 = 5 m s2
a2 = g = −10 m s2

1ª etapa: foguete sobe do chão durante 10 s


2
com aceleração de 5 m s .
1
H1 = H0 + Vo t + a1t12
2
5
H1 = 102  H1 = 250 m (i)
2
V1 = V0 + a1t1
( v + v0 ) t ( 3t − 2t ) v ( 4t − 3t ) v V1 = 50 m / s (ii)
Δs = + ( 2t − t ) v + −
2 2 2
vt + v 0 t + 2vt + vt − vt
Δs =
2 2ª etapa: foguete continua subindo com
( 3v + v 0 ) t desaceleração da gravidade até atingir a
Δs =
2 altura máxima H2 .

Portanto, a velocidade média foi de:


V22 = V12 + 2a2 (H2 − H1) Portanto, o intervalo de tempo entre esses
pontos é menor ou igual que o limiar de fusão
0 = 502 + 2  ( −10)  (H2 − 250)
( τ) :
H2 = 375 m (iii) ΔtBC  τ.

V2 = V1 + a2  (t 2 − t1) Assim:
0 = 50 + ( −10)  (t 2 − 10) ΔtBC  τ  Δt AB . ( eq. I)
t 2 = 5 s (iv)
Como o bloco desce o plano inclinado de θ
Depois dos 10 s subindo, o foguete subiu livre de atritos, o movimento é
por mais 5 s, até adquirir velocidade zero. uniformemente variado (MUV) e a
aceleração escalar é:
a = g sen θ. ( eq. II)
3ª etapa: Foguete desce com aceleração da
gravidade.
1
Da função horária do espaço para o MUV:
H3 = H2 + V2  Δt + a  Δt 2 a 2 2S
2 S= t  t= . ( eq. III)
2 2 a
0 = 375 + 0 − 5t 3
t 3  8,7 s Aplicando a equação (III) aos pontos A, B e
C:
t t = t1 + t 2 + t 3  t t = 10 + 5 + 8,7  t t  23,7 s 
t A =
2 (n − 2) d

( )
 a 2nd
  Δt AB = tB − t A = n −1 − n−2 . ( eq. IV )
 2 ( n − 1) d a
Bt =
Portanto, conforme indica a alternativa [A], a  a

altura máxima atingida será de 375 m e o  2 ( n − 1) d


 tB =
tempo total de 23,7 s. 

a
 ΔtBC = tC − tB =
2nd
( n − )
n −1 . ( eq. V )
 2nd a
tC =
 a
9
Ao sair do 1º ressalto, ponto adotado como Substituindo as equações (II), (IV) e (V) em
origem dos espaços, nenhum som é emitido. (I):
A figura mostra as posições dos sucessivos  2nd   2nd 
ressaltos, separados pela distância d.

 g sen θ
( n − )
n −1   τ  
  g sen θ
( n −1 −

)
n − 2 .
   

10
Façamos as equações horárias dos
movimentos de A e B:

Entre os pontos A e B, ressaltos (n – 1)º e nº,


respectivamente, o som ainda não é Como queremos que B alcance A, faremos
contínuo. Portanto, o intervalo de tempo SA = SB
entre esses pontos é maior que o limiar de
fusão ( τ ) :
Δt AB  τ.

Entre os pontos B e C, ressaltos nº e (n + 1)º,


respectivamente, o som já é contínuo. 11
Vamos pôr o referencial em A, no instante em
que ele é visto pelo maquinista do trem B.
Assim, a partir desse instante, as equações
horárias dos trens A e B são:

O gráfico dessa função V(t) é mostrado na


figura anterior.
Como no tempo t = 12 minutos os trens a) O carro se move com aceleração escalar
colidirão, a resposta é o item C. constante a = -2 m/s2;
12 b) A distância que o móvel percorre até parar
é numericamente igual à área hachurada no
gráfico acima, isto é, D = 9 m.

13
Resolução do Prof. Laerte Pereira

O gráfico revela uma dependência linear


entre a variável dependente V2 e a variável
independente X, relação essa que pode ser
escrita na forma V2 = a.X + b, onde a e b são
coeficientes a serem determinados. 𝛥𝑠 9
𝑣𝑚 = =
Para X = 1, tem-se 32 = a.1 + b 𝛥𝑡 𝛥𝑡1 + 𝛥𝑡2
Para X = 5, tem-se 16 = a.5 + b 5 1
Resolvendo-se o sistema nas variáveis a e b, 𝛥𝑡1 = = ℎ
10 2
encontram-se a = -4 e b = 36. A velocidade escalar média no MUV é a
V2 = 36 - 4.X média aritmética
Comparando com a função de Torricelli para 𝑣1 + 𝑣2 10 + 50
o MUV v 2 = v02 + 2.a.X 𝑣𝑚 = = = 30𝑘𝑚/ℎ
2 2
obtém-se V0 = 6 m/s e a = -2 m/s2. Logo
V=6 - 2.t 4 2
𝛥𝑡2 = = ℎ
30 15
2 1 19
𝛥𝑡1 + 𝛥𝑡2 = ℎ+ ℎ= ℎ
15 2 30
O computador de bordo aos 15 km vai estar
marcando a rapidez média dos últimos 9 km
9
𝑣𝑚 =
19
(30)
Aproximando 19 pra 20 só pra facilitar os
cálculos, já que no momento da prova não
podemos fazer uso de calculadora

𝑣𝑚 ≌ 13,5 𝑘𝑚/ℎ

𝛥𝑠 = 𝑣𝑎𝑙𝑜𝑟 𝑑𝑎 á𝑟𝑒𝑎
(10 + 8). 𝑣
100 =
2
𝑣 ≌ 11 𝑚/𝑠

17
Temos a alternativa E como resposta. v0 = 0
De 0 a 4 s: Δv = “área” ⇒ v4 – v0 = 4 · 2 ⇒
14 v4 = 8 m/s
a) v6 = v4 = 8 m/s
De 6 s a 10 s: Δv = “área” ⇒
v10 – v6 = 4 · (– 4) ⇒
b) v10 – 8 = – 16 ⇒ v10 = – 8 m/s

15
km 22
Para cálculo da área, o gráfico dado equivale
a:

distância percorrida = A1 + | A2 | = 48 m

18
16
De 0 a 2s, temos:
Façamos uma aproximação utilizando dois
segmentos de reta, de modo que se 𝑣0 + 𝑣2 𝛥𝑠
mantenha quase a mesma “área” do gráfico 𝑣𝑚𝐵 = =
original: 2 𝛥𝑡
0 + 𝑣2
6=
2
𝑣2 = 12 𝑚/𝑠

19
Os instantes t1 e t2, nos quais o móvel passa
pela altura H (S = H), são as soluções da
equação do 2° grau abaixo, na variável t:

𝑔. 𝑡 2
𝐻 = 𝑣0 𝑡 −
2
Usando o conceito de soma e produto das
raízes de uma equação do 2° grau, o produto
das raízes da equação obtida acima permite
escrever:

2𝐻
𝑡1 . 𝑡2 =
𝑔
2𝐻
𝑔=
𝑡1 . 𝑡2
20
2
𝛥𝑡 = √ (√𝑛 − √𝑛 − 1)
𝑔
21
Seja V0 a velocidade inicial e a a aceleração
escalar de um móvel em MUV. Do conceito
de aceleração escalar, sabemos que:
No instante t = 0s, a velocidade do corpo
vale: V0
No instante t = 1 s, a velocidade do corpo
vale: V0 + a
No instante t = 2 s, a velocidade do corpo
vale: V0 + 2a
No instante t = 3 s, a velocidade do corpo
vale: V0 + 3a
No instante t = n-1, a velocidade do corpo
vale: V0 + (n-1 )a
No instante t = n, a velocidade do corpo vale:
V0 + na
Num MUV, a velocidade escalar média, num
certo intervalo de tempo, pode ser calculada
pela média aritmética das velocidades do
móvel no início e no término do referido
intervalo de tempo.

22
Letra C
Aprofundamento ITA IME coeficiente angular. Esse resultado já era
esperado, já que no MU a função horária do
VELOCIDADE ESCALAR INSTANTÂNEA espaço é dada por:
𝑠 = s0+𝑣.𝑡
Para determinarmos a velocidade escalar em Onde 𝑣 é constante.
um dado instante, temos que usar a definição
de velocidade escalar instantânea: ACELERAÇÃO ESCALAR INSTANTÂNEA

𝑣 = lim
Δs
Diante das semelhanças nas definições, a
Δt→0 Δt análise gráfica da aceleração escalar
instantânea segue o mesmo modelo da
Essa definição mostra que devemos calcular
velocidade escalar instantânea:
a velocidade escalar média em dois instantes Δv
muito próximos. Vamos ilustrar essa a = lim Δt
Δt→0
operação matemática utilizando o gráfico Assim, a aceleração escalar instantânea, em
abaixo: um gráfico 𝑣×𝑡, corresponde numericamente
a tangente do ângulo entre a reta tangente e
o eixo do tempo. Graficamente:

Observação: para simplificar e se tornar mais


visual a reta tangente, não desenhamos as
Quando fazemos o Δ𝑡 tender a zero, retas secantes, como feito no gráfico da
estamos aproximando 𝑡2 de 𝑡1, isto é, velocidade escalar instantânea.
pegando intervalos de tempo cada vez Portanto, dizemos que:
menores. Assim, a reta secante que une os
pontos A e C vai se tornando a reta tangente 𝑅𝑒𝑝𝑟𝑒𝑠𝑒𝑛𝑡𝑎
no ponto A. Dessa forma, a deixamos de 𝑡𝑔𝜃 → 𝑎
calcular a velocidade escalar média e
obtemos a velocidade escalar instantânea no Concluímos que para calcular a aceleração
ponto A. escalar instantânea no gráfico 𝑣×𝑡, basta
Assim, podemos dizer que a velocidade acharmos a tangente do ângulo da reta
escalar instantânea, em 𝑡1, é numericamente tangente no instante desejado.
igual ao valor da tangente do ângulo que No caso do MUV, sabemos que a velocidade
passa por A, isto é: escalar é dada por:

𝑅𝑒𝑝𝑟𝑒𝑠𝑒𝑛𝑡𝑎 𝑣 = 𝑣0+𝑎.𝑡
𝑡𝑔𝜃 → 𝑣
Ou seja, função horária da velocidade é uma
Para o caso do MU, sabemos que a reta, portanto, tem inclinação constante,
velocidade escalar instantânea é igual a logo, a aceleração escalar instantânea
velocidade escalar média, portanto, em coincide com o coeficiente do gráfico. Sendo
qualquer ponto do gráfico de 𝑠×𝑡, a reta terá assim, a aceleração escalar instantânea é
a mesma inclinação, ou seja, o mesmo
igual a aceleração escalar média, 4.
𝑑
(𝑥 𝑚 ) = 𝑚 ∙ 𝑥 𝑚−1
𝑑𝑥
característica desse movimento. 𝑑 1
Vimos anteriormente que para calcular a 5.𝑑𝑥 𝑙𝑛𝑥 = 𝑥
velocidade a partir da função horária do 𝑑
6.𝑑𝑥 𝑒 𝑥 = 𝑒 𝑥
espaço, basta derivarmos a função em 𝑑
relação ao tempo. O mesmo processo é feito 7.𝑑𝑥 𝑠𝑒𝑛𝑥 = 𝑐𝑜𝑠𝑥
para calcular a aceleração a partir da função 𝑑
8.𝑑𝑥 𝑐𝑜𝑠𝑥 = −𝑠𝑒𝑛𝑥
horária da velocidade. No Cálculo Diferencial 𝑑
Integral é demonstrado o significado da 9.𝑑𝑥 𝑡𝑔𝑥 = 𝑠𝑒𝑐 2 𝑥
derivada, que nada mais é a reta tangente a 𝑑
10.𝑑𝑥 𝑐𝑜𝑡𝑔𝑥 = −𝑐𝑜𝑠𝑠𝑒𝑐 2 𝑥
função, num dado ponto desejado. 𝑑
Assim, dado um gráfico de 𝑠×𝑡, basta 11.𝑑𝑥 𝑠𝑒𝑐𝑥 = 𝑡𝑔𝑥 ∙ 𝑠𝑒𝑐𝑥
𝑑 𝑑𝑢
traçarmos a reta tangente num ponto 12.𝑑𝑥 𝑒 𝑢 = 𝑒 𝑢 ∙ 𝑑𝑥
desejado e verificamos como a velocidade se 𝑑 𝑑𝑢
comporta: 13.𝑑𝑥 𝑠𝑒𝑛𝑢 = 𝑐𝑜𝑠𝑢 ∙ 𝑑𝑥
𝑑 𝑑𝑢
14.𝑑𝑥 𝑐𝑜𝑠𝑢 = −𝑠𝑒𝑛𝑢 ∙ 𝑑𝑥
a) inclinação para cima: velocidade positiva;
TABELA DE INTEGRAIS
b) inclinação para baixo: velocidade 1.∫ 𝑑𝑥 = 𝑥
negativa.
2.∫ 𝑎 ∙ 𝑢 ∙ 𝑑𝑥 = 𝑎 ∙ ∫ 𝑢 ∙ 𝑑𝑥
Outro fato importante é a inclinação da reta 3.∫(𝑢 + 𝑣) ∙ 𝑑𝑥 = ∫ 𝑢 ∙ 𝑑𝑥 + ∫ 𝑣 ∙ 𝑑𝑥
𝑥 𝑚+1
tangente: quanto mais inclinada a reta 4.∫ 𝑥 𝑚 𝑑𝑥 = (𝑚 ≠ 1)
𝑚+1
tangente, mais cresce a função derivada. Isto 𝑑𝑥
5.∫ 𝑥 = 𝑙𝑛|𝑥|
é, se tomarmos dois pontos com inclinações
diferentes, podemos dizer se uma cresce 6.∫ 𝑒 𝑥 ∙ 𝑑𝑥 = 𝑒 𝑥
mais que a outra ou dizer qual inclinação é 7.∫ 𝑠𝑒𝑛𝑥𝑑𝑥 = −𝑐𝑜𝑠𝑥
maior. 8.∫ 𝑐𝑜𝑠𝑥 𝑑𝑥 = 𝑠𝑒𝑛𝑥
9.∫ 𝑡𝑔𝑥 𝑑𝑥 = 𝑙𝑛|𝑠𝑒𝑐𝑥|
𝑥 𝑠𝑒𝑛2𝑥
10.∫ 𝑠𝑒𝑛2 𝑥 𝑑𝑥 = 2 − 4
1
11.∫ 𝑒 −𝛼𝑥 𝑑𝑥 = − 𝛼 ∙ 𝑒 −𝛼∙𝑥

TEOREMA BINOMIAL
𝑛 ∙ 𝑥 𝑛 ∙ (𝑛 − 1) ∙ 𝑥 2
(1 ± 𝑥)𝑛 = 1 ± +
1! 2!
+ ⋯ (𝑥 2 < 1)
𝑛∙𝑥
(1 ± 𝑥)−𝑛 = 1 ∓
1!
Pelo gráfico, temos que 𝑡𝑔𝛼 > 𝑡𝑔𝛽, logo: 𝑛 ∙ (𝑛 − 1) ∙ 𝑥 2
𝑣𝐵>𝑣𝐴. +
2!
A mesma análise é válida para o gráfico 𝑣×𝑡, + ⋯ (𝑥 2 < 1)
analisando a aceleração em instantes
diferentes. SÉRIE EXPONENCIAL
𝑥
𝑥2 𝑥3
TABELA DE DERIVADAS 𝑒 =1+𝑥+ + +⋯
𝑑𝑥 2! 3!
1.𝑑𝑥 =1
𝑑 𝑑𝑢
2.𝑑𝑥 (𝑎 ∙ 𝑢) = 𝑎 ∙ 𝑑𝑥 SÉRIE LOGARÍTMICA
𝑑 𝑑𝑢 𝑑𝑣
3.𝑑𝑥 (𝑢 + 𝑣) = 𝑑𝑥 + 𝑑𝑥
𝑥2 𝑥3
𝑙𝑛(1 + 𝑥) = 𝑥 − +
2 3!
+ ⋯ (|𝑥| < 1)

SÉRIES
TRIGONOMÉTRICAS(𝜃 𝑒𝑚 𝑟𝑎𝑑𝑖𝑎𝑛𝑜𝑠)
𝜃3 𝜃5
𝑠𝑒𝑛𝜃 = 𝜃 − + − ⋯.
3! 5!
2 4
𝜃 𝜃
𝑐𝑜𝑠𝜃 = 𝜃 − + −⋯
2! 4!
𝜃 3 2𝜃 5
𝑡𝑔𝜃 = 𝜃 + +
3! 15!
EXERCÍCIOS DE CLASSE instante? (d) A velocidade escalar está
aumentando ou diminuindo nesse instante?
1. A função horária do espaço para o
movimento de um ponto material é: 6. (Halliday) A função posição x(t) de uma
partícula que está se movendo ao longo do
s = 2t2 - 1 (SI) eixo x é
Determine: x = 4,0 − 6,0t2, com x em metros e t em
segundos. (a) Em que instante e (b) em que
a) a função da velocidade escalar posição a partícula para
instantânea; (momentaneamente)?

b) a velocidade escalar no instante 2 s. 7. (Halliday) A posição de uma partícula que


se move ao longo do eixo x é dada por
2. Os espaços s de uma partícula variam x = 12t2 − 2t3, em que x está em metros e t
com o tempo t, de acordo com a função: em segundos. Determine (a) a posição, (b) a
velocidade e (c) a aceleração da partícula em
s = 4t2 - 2t (SI) t = 3,0 s. (d) Qual é a coordenada positiva
máxima alcançada pela partícula e (e) em
Determine: que instante de tempo é alcançada? (f) Qual
é a velocidade positiva máxima alcançada
a) a função da velocidade escalar pela partícula.
instantânea;
8. O gráfico na figura mostra a posição x de
b) a velocidade escalar no instante 5 s. um objeto em movimento sobre uma
trajetória retilínea, em função do tempo t.
3. A função horária do espaço referente ao
movimento de uma partícula é s = 5t3 - 6t,
válida no SI. Determine:
a) a função da velocidade escalar
instantânea;
b) a velocidade escalar no instante 2 s;
c) a função da aceleração escalar
instantânea;
d) a aceleração escalar no instante 2 s. A partir desse gráfico, é possível concluir que
a velocidade instantânea
4. A velocidade escalar instantânea de um do objeto anulou-se somente:
móvel varia com o tempo, conforme a função
v = 5t2 + 4, válida no SI. a) no instante 0 segundo;
Determine: b) nos instantes 9 e 14 segundos;
a) a função horária da aceleração escalar c) nos instantes 2 e 7 segundos;
instantânea; d) nos instantes 5 e 11 segundos;
b) a aceleração escalar no instante 4 s. e) nos instantes 2, 5, 7 e 11 segundos.

5. (Halliday)(a) Se a posição de uma 9. Um carro está andando ao longo de uma


partícula é dada por x = 4 −12t + 3t2 (em que estrada reta e plana. Sua posição em função
t está em segundos e x em metros), qual é a do tempo está representada neste gráfico:
velocidade da partícula em t = 1 s? (b) O
movimento nesse instante é no sentido
positivo ou negativo de x? (c) Qual é a
velocidade escalar da partícula nesse
12. Um elétron partindo do repouso tem uma
velocidade que aumenta linearmente com o
tempo que é v=kt, onde k=2m/seg2. A
distância percorrida nos primeiros 3
segundos será
a) 9 𝑚
Sejam vp, vQ e vR os módulos das
velocidades do carro, respectivamente, nos b) 16 𝑚
pontos P, Q e R indicados nesse gráfico. c) 27 𝑚
Com base nessas informações, é correto d) 36 𝑚
afirmar que:
13. A relação entre tempo(t) e distância(x) de
a) vQ < vP < vR. uma partícula em movimento é t=αx2+βx,
onde α e β são constantes. A desaceleração
b) vP < vR < vQ. é
a) 2𝛼𝑣 3
c) vQ < vR < vP.
b) 2𝛽𝑣 3
d) vP < vQ < vR. c) 2𝛼𝛽𝑣 3
d) 2𝛽 2 𝑣 3
10. Na figura está representada graficamente
a função horária
14. Na figura, a barra de comprimento l= 5 m
s = t2 – 2t (SI).
está apoiada sobre superfícies
perpendiculares entre si. Sua extremidade
superior sobe com uma velocidade de v=4
m/s. Encontre o valor da velocidade escalar
da extremidade inferior para t= 1 s. Se em
t0=0 s a extremidade superior estava no
vértice reto.

Calcule a velocidade escalar em t = 2 s:

a) por meio da função horária da velocidade;

b) por meio do gráfico dado.

11. As coordenadas de uma partícula em


movimento a qualquer instante são dadas 15. (JEE) No arranjo mostrado na figura, os
por extremos P e Q de uma corda não extensível
x=at2 e y=bt2. se movem para baixo com velocidade
uniforme U. As polias A e B são fixas. A
A velocidade da partícula a qualquer instante massa M se move para cima com velocidade
é
a) 2𝑡(𝑎 + 𝑏)
b) 2𝑡√(𝑎2 − 𝑏 2 )
c) 𝑡√𝑎2 + 𝑏 2
d) 2𝑡√(𝑎2 + 𝑏 2 )
a) 2𝑈 𝑐𝑜𝑠 𝜃 movimentos de satélites em torno de
b) 𝑈 𝑐𝑜𝑠 𝜃 planetas (ou de planetas em torno e
estrelas), o momento angular não se altera
2𝑈
c) 𝑐𝑜𝑠 𝜃 (permanece constante).
𝑈 Mostre que o momento angular será
d) 𝑐𝑜𝑠 𝜃 constante durante o movimento do satélite.

16. Dois anéis O e O' são colocados em duas 18. Segunda Lei de Kepler:
hastes rígidas ideais e estacionárias na
vertical AB e A'B', respectivamente,
conforme mostrado na figura. Uma corda v=dr/dt
ideal e inextensível é fixada no ponto A' e no
anel O e passa por O'. Supondo que o anel dA
O' se move para baixo com uma velocidade
constante v, encontre o módulo da
velocidade do anel O em termos de α. r

Considere o raio vetor 𝑟⃗ que liga um planeta


a um satélite descreva uma área infinitesimal
𝑑𝐴⃗ em um intervalo de tempo 𝑑𝑡, conforme
𝑑𝐴⃗ ⃗⃗
𝐿
a figura acima. Mostre que = 2𝑚, onde
𝑑𝑡
⃗⃗ é o momento angular.
𝐿

17. Recentemente muitos problemas de


Gravitação Universal na prova do ITA, para
serem resolvidos necessitam da aplicação
de duas leis de conservação, são elas:
conservação da energia mecânica e
conservação do momento angular.

Conservação do momento angular:

m ∙ v ∙ r ∙ senθ = constante

Justificativa rápida: A força gravitacional é


uma força central, logo torque resultante em
relação a qualquer ponto ao longo de sua
linha de ação é nulo. Portanto, para
GABARITO
Derivando em relação a t
8. Letra E
9. Letra C 𝑑𝑥 𝑑𝑦
10. 2𝑥 + 2𝑦 =0
𝑑𝑡 𝑑𝑡
a) s = t 2 − 2t Logo
v = -2 + 2t 𝑦
𝑢 = − .𝑣
v = 2 m/s 𝑥
Temos que
Δs
b) v = = 2m/s
Δt 𝑥 = √𝑙 2 − 𝑦 2
e y = v.t
11.
Vel. No eito X
de maneira que
𝑑𝑥
𝑣𝑥 = = 2𝑎𝑡
𝑑𝑡 𝑣 2. 𝑡
Vel. no Y 𝑢=− ≌ 5,33m/s
𝑑𝑦 √𝑙 2 − 𝑣 2 . 𝑡 2
𝑣𝑦 = = 2𝑏𝑡
𝑑𝑡 15.
O modulo da vel. Da partícula, 𝑙2 = 𝑏2 + 𝑦 2
𝑣 = √𝑣𝑥2 + 𝑣𝑦2 = 2𝑡√𝑎2 + 𝑏 2 𝑑𝑦 𝑙 𝑑𝑙
= ( ).
𝑑𝑡 𝑦 𝑑𝑡
12. 𝑑𝑦 1 𝑈
3 3
1 3 ⇒ =( ).𝑈 =
𝑑𝑡 𝑐𝑜𝑠 𝜃 𝑐𝑜𝑠 𝜃
𝑆 = ∫ 𝑣 𝑑𝑡 = ∫ 𝑘𝑡 𝑑𝑡 = [ 𝑘𝑡 2 ]
0 0 2 0
1
= . 2.9 = 9𝑚 16.
2

13.
𝑑𝑡 1
= 2𝛼𝑥 + 𝛽 ⇒ 𝑣 =
𝑑𝑥 2𝛼𝑥 + 𝛽
𝑑𝑣 𝑑𝑣 𝑑𝑥
 𝑎= = 𝑑𝑥 . 𝑑𝑡
𝑑𝑡
𝑑𝑣 −𝑣.2𝛼
𝑎 = 𝑣 𝑑𝑥 = (2𝛼𝑥+𝛽)2 = −2𝛼. 𝑣. 𝑣 2 =
−2𝛼𝑣 3
 desaceleração = 2𝛼𝑣
3

Temos que
14.
𝑑𝑦
𝑣=
𝑑𝑡

−𝑑𝑥
𝑣0 =
𝑑𝑡
(𝐿 − 𝑦)2 = 𝑠 2 + (𝑥 − 𝑦)2
Derivando em relação ao tempo

−𝑑𝑦 𝑑𝑥 𝑑𝑦
2(𝐿 − 𝑦) = 2(𝑥 − 𝑦). ( − )
𝑑𝑡 𝑑𝑡 𝑑𝑡
X2 + y2 = l2
(−𝑣0 − 𝑣). cos 𝛼 = −𝑣 constante é uma consequência do momento
angular ser constante.
𝑣(1 − cos 𝛼 )
𝑣0 =
cos 𝛼

17. resolução:
Considere um satélite em órbita em torno do
planeta da figura anterior, estará sujeito a
uma força gravitacional dada por:
𝐺∙𝑚∙𝑀
𝐹⃗ = − 𝑟̂
𝑟2
𝐺∙𝑚∙𝑀 𝑟⃗
𝐹⃗ = − 3
𝑟⃗, 𝑒𝑚 𝑞𝑢𝑒 𝑟̂ =
𝑟 𝑟

Assim o torque exercido pelo planeta sobre o


satélite será determinado por:
𝐺∙𝑚∙𝑀
𝜏⃗ = 𝑟⃗ × 𝐹⃗ = 𝑟⃗ × [− 𝑟⃗] = ⃗0⃗
𝑟3
Sabemos: 𝑟⃗ × 𝑟⃗ = ⃗0⃗

Lembrando que o momento angular é dado


por:
⃗⃗ = 𝑟⃗ × 𝑝⃗
𝐿

Podemos determinar a taxa de variação do


momento angular será dada por:
⃗⃗ 𝑑(𝑟⃗ × 𝑝⃗) 𝑑𝑟⃗
𝑑𝐿 𝑑𝑝⃗
= = × 𝑝⃗ + 𝑟⃗ ×
𝑑𝑡 𝑑𝑡 𝑑𝑡 𝑑𝑡

𝑑𝐿⃗⃗
= 𝑣⃗ × (𝑚𝑣⃗) + 𝑟⃗ × 𝐹⃗ = ⃗0⃗
𝑑𝑡
Assim, podemos concluir que o momento
angular será constante durante o
movimento do satélite.

18. Resolução:
Fazendo o uso do produto vetorial, em que
podemos escrever:
1 1
𝑑𝐴⃗ = (𝑟⃗ × 𝑑𝑟⃗) = (𝑟⃗ × 𝑣⃗ ∙ 𝑑𝑡)
2 2
𝑑𝐴⃗ 1 𝑚 ∙ (𝑟⃗ × 𝑣⃗)
= (𝑟⃗ × 𝑣⃗) =
𝑑𝑡 2 2𝑚

𝑑𝐴⃗ ⃗⃗
𝐿
=
𝑑𝑡 2𝑚

Conclusão: A velocidade areolar de um


satélite em torno de um planeta é constante.
Em resumo, a velocidade areolar ser
Aprofundamento ITA/IME pode ser maior ou menor ou igual à
velocidade do objeto.
VELOCIDADE DA IMAGEM PARA
A relação entre a velocidade do objeto e a
ESPELHO ESFÉRICO
velocidade da imagem também é válida para
Quando a posição do objeto muda com o o espelho convexo. No espelho convexo, a
tempo no eixo principal em relação ao velocidade da imagem é mais lenta do que a
espelho, a posição da imagem também do objeto, qualquer que seja a posição do
muda com o tempo em relação ao espelho. objeto. Esta relação não é verdadeira em
Assim, para conhecer a relação da termos de aceleração do objeto e da
velocidade do objeto e da imagem, usamos imagem.
a equação do espelho.
Caso II: Componente da velocidade da
imagem ao longo do eixo y
Como sabemos, a ampliação transversal.

𝐼 𝑡𝑎𝑚𝑎𝑛ℎ𝑜 𝑑𝑎 𝑖𝑚𝑎𝑔𝑒𝑚
𝐴= =
𝑂 𝑡𝑎𝑚𝑎𝑛ℎ𝑜 𝑑𝑜 𝑜𝑏𝑗𝑒𝑡𝑜
𝑦𝐼
Figura 1 =
𝑦𝑂
Consideremos que um objeto está se
movendo com velocidade vO na frente de um
espelho côncavo, a posição do objeto neste 𝑓
𝑦𝐼 = ( ) . 𝑦𝑂
instante é (xO, yO). Uma imagem é formada 𝑓 − 𝑥𝑂
no local cuja coordenada é (xI, yI) neste
Diferenciando com respeito ao tempo,
instante. Como de costume, tomamos o eixo
x como eixo principal. 𝑓 𝑦𝑂 . 𝑓
𝑣𝑚𝐼𝑦 = ( ) 𝑣𝑚𝑂𝑦 + 𝑣
𝑓 − 𝑥𝑂 (𝑓 − 𝑥𝑂 )2 𝑚𝑂𝑥
Caso I: Componente da velocidade da
imagem ao longo do eixo x. Onde,

1 1 1 𝑣𝑚𝐼𝑦 = Velocidade da imagem em relação ao


Como, 𝑥 + 𝑥 =
𝐼 𝑜 𝑓 espelho ao longo do eixo y;
Diferenciando os dois lados no tempo, 𝑣𝑚𝑂𝑦 = Velocidade do objeto em relação ao
1 𝑑𝑥𝐼 1 𝑑𝑥𝑂 𝑑𝑥𝐼 𝑥𝐼2 espelho ao longo do eixo y;
− 2 . 𝑑𝑡 − 2 . 𝑑𝑡 = 0 𝑜𝑢 𝑑𝑡 = − 2
𝑥𝐼 𝑥𝑂 𝑥𝑂
𝑦𝑂 = 𝑦 = coordenada ou altura da imagem
Ou 𝑣𝑚 = − 𝐴 . 𝑣𝑂𝑚 2 a partir do eixo y naquele instante;
𝑥 𝑣
Com, 𝐴 = − 𝑥 𝐼 = 𝑢 = 𝑎𝑚𝑝𝑙𝑖𝑎çã𝑜 𝑣𝑚𝑂𝑥 = Velocidade do objeto, em relação ao
𝑂
espelho ao longo do eixo x.
O sinal negativo indica que o objeto e a
Nota: Na cinemática, temos estudado que a
imagem estão sempre em movimento oposto
velocidade dos objetos nas direções x e y
um em relação ao outro.
são independentes um do outro. Mas aqui, a
No espelho côncavo, dependendo da componente de velocidade da imagem na
posição do objeto, a velocidade da imagem
direção y dependerá de sua coordenada x e lente. Sabendo que a partícula, a lente e o
velocidade na direção x também. anteparo estão em movimento em
velocidade escalar v nos respectivos
EXEMPLO: Determine o valor da sentidos indicados na figura, a aceleração
velocidade escalar da imagem em relação do ponto de impacto do feixe, no
ao espelho. referencial do anteparo, é:

Passo I: Antes de mais nada, precisamos


2
desenhar a situação. a) v 4f
2
b) v 3f
2
c) v 2f
2
d) 2v f
2
e) 4v f
Passo-II: Coloque a origem de um sistema de
coordenadas no vértice. Resolução:
No referencial do anteparo, temos:
Passo-III: Encontre o componente da
velocidade do objeto ao longo dos eixos x e
y.
Onde, 𝑣𝑜𝑥 = 5 𝑚/𝑠, 𝑣𝑜𝑦 = 0

Passo IV: Finalmente temos que aplicar a


fórmula da velocidade da imagem ao longo
dos dois eixos.
Por semelhança de triângulos, vem:
Componente da velocidade ao longo do eixo z x + 2vt
=
x em relação ao espelho. y + vt f
xy ( 2y + x ) v 2v 2 2
𝑣𝑚𝑥 = − 𝐴2 . 𝑣𝑂𝑚𝑥 z= + t+ t
f f f

a 2
Comparando com s = s0 + v0 t + t ,
2
obtemos:
a 2v 2
=
2 f
4v 2
a =
Exemplo: (Ime 2020) Uma partícula emite f
um feixe laser horizontal de encontro a Resposta letra E.
uma lente convergente de distância focal
f. Após ser desviado, o feixe atinge um
anteparo localizado depois do foco da
Capítulo 5 – Movimento curvilíneo I. A velocidade vetorial é tangente à
trajetória e orientada no sentido do
movimento. Logo: vetor (1).
II. Se o movimento é acelerado, a
aceleração vetorial deve admitir uma
componente tangencial no sentido do
movimento.
III. Se o movimento é curvilíneo, a
aceleração vetorial deve admitir uma
Se um móvel segue uma trajetória curva, a componente centrípeta.
aceleração vetorial instantânea 𝑎⃗ pode ser
decomposta segundo as retas t e n, obtendo,
respectivamente, as componentes at
(tangencial) e an (normal).
A componente normal an, pelo fato de estar
dirigida para o centro de curvatura da
trajetória em cada instante, recebe a
denominação componente centrípeta (acp).
Assim, relacionando vetorialmente a, at e acp, Exemplo: (IJSO) Uma atleta decide fazer
temos: um pequeno teste de velocidade,
⃗⃗⃗⃗=𝑎⃗t + 𝑎⃗cp
𝑎 primeiramente em linha reta e depois em
movimento circular. Durante o percurso
A aceleração tangencial está relacionada em linha reta, sua velocidade obedece ao
com as variações de intensidade da gráfico conforme exibido na figura a
velocidade vetorial. seguir.
A aceleração centrípeta está relacionada
com as variações de direção da velocidade
vetorial.

Exemplo: A figura a seguir representa um


instante do movimento curvilíneo e
acelerado de uma partícula:

a) Encontre a aceleração instantânea da


atleta nos pontos A, B e C.
Se o movimento ocorre da esquerda para b) Calcule a distância que ela percorre nos
a direita, os vetores que melhor dois primeiros segundos da corrida.
representam a velocidade vetorial e a c) O percurso circular começa no instante
aceleração vetorial da partícula no t = 4 s, com a velocidade indicada pelo
instante considerado, e nessa ordem, são: gráfico da figura acima. A força de atrito
a) 1 e 2. limitante entre o tênis e o solo não permite
b) 5 e 3. que a atleta tenha uma aceleração
c) 1 e 4. centrípeta maior do que 3,0 ms-2. Calcule
d) 5 e 4. o raio mínimo de seu percurso circular.
e) 1 e 1. Considere uma velocidade constante ao
longo de todo o percurso.
Resolução:
Resolução:
a) Entre os instantes 0 e 2,0 s (inclusive no - A trajetória descrita pelo corpo é uma
ponto A) a aceleração escalar é constante circunferência.
e é dada por: - A partícula percorre deslocamentos
𝛥𝑣 angulares iguais em intervalos de tempo
𝑎= = 2𝑚/𝑠 2 iguais.
𝛥𝑡
- A magnitude da velocidade linear 𝑣⃗,
Entre os instantes 2,0 s e 3,0 s (inclusive permanece constante, mas sua direção
ponto B) a aceleração escalar é igual a 0. varia.
Entre os instantes 3,0 s e 4,0 s (inclusive - A velocidade angular 𝑤 ⃗⃗⃗, permanece
no ponto C) a aceleração escalar é constante tanto em magnitude quanto em
constante e é dada por: direção.
𝛥𝑣 - A velocidade linear 𝑣⃗ em qualquer instante
𝑎= = −2𝑚/𝑠 2
𝛥𝑡 é tangente à trajetória.

b) 𝛥s = “área” (v x t)
𝛥s = 4,0 m

𝑣2
c) 𝑎𝑐𝑡𝑝 = 𝑅

logo Rmín = 2m.

- A velocidade angular 𝑤 ⃗⃗⃗⃗⃗ em qualquer


Movimento circular instante é perpendicular ao plano que
contém a trajetória.
- A aceleração angular é zero ou nula (α = 0).
- O responsável por mudar a direção da
velocidade 𝑣⃗ é a aceleração centrípeta (ac),
que em qualquer instante aponta para o
centro do círculo.
- O movimento da partícula é periódico.
- A aceleração total a da partícula é a
Deslocamento linear (S) aceleração centrípeta (normal).
É o comprimento do arco de circunferência
percorrido pela partícula, durante o intervalo Aceleração centrípeta (𝒂𝒄𝒕𝒑 ) e sua
de tempo Δt, entre quaisquer dois pontos A e dedução alternativa.
B em seu caminho.
Deslocamento angular (θ) Suponha que o corpo descreva um
É o ângulo central θ delimitado ou movimento circular uniforme de raio r com
subtendido pelo deslocamento linear (S). A velocidade escalar constante v.
relação entre os deslocamentos linear (S) e
angular (θ) é dada por:

S = R.θ

sendo, R o raio da circunferência.


A unidade de θ é em (rad).

Movimento circular uniforme (M.C.U.)

Características
O vetor velocidade 𝒗
⃗⃗ é tangente à trajetória curvatura, de A e B descrevem áreas
e perpendicular ao vetor de posição 𝒓
⃗⃗. iguais em tempos iguais, determinar a
relação entre suas velocidades angulares.
Os componentes retangulares do vetor de
velocidade 𝒗
⃗⃗ são Resolução do Prof. Laerte Pereira:

𝑦
𝑣𝑥 = −𝑣 sin 𝜃 = −𝑣
𝑟
𝑥
𝑣𝑦 = 𝑣 cos 𝜃 = 𝑣
𝑟

Como v/r é constante, os componentes do Igualando áreas: AA = AB


Vetor de aceleração 𝒂
⃗⃗ são 𝛼𝑅 2 𝛽(2𝑅)2
=
𝑣 𝑑𝑦 𝑣 𝑣2 2 2
𝑎𝑥 = − = − 𝑣𝑦 = − cos 𝜃
𝑟 𝑑𝑡 𝑟 𝑟 α = 4β
logo:
𝑣 𝑑𝑥 𝑣 𝑣2 ωA = 4ωB
𝑎𝑦 = = 𝑣 = − sin 𝜃
𝑟 𝑑𝑡 𝑟 𝑥 𝑟

O módulo de aceleração 𝒂
⃗⃗ em movimento Equações
circular uniforme é

𝑣2
𝑎 = √𝑎𝑥2 + 𝑎𝑦2 =
𝑟

Sua direção é radial (a mesma do vetor 𝒓


⃗⃗) e
seu sentido é em sentido ao centro (no
sentido oposto ao vetor 𝒓
⃗⃗).

A lei de Kepler para MCU


Consideremos o movimento de uma
partícula ao longo de uma circunferência de
raio R, percorrendo o comprimento de arco S
entre A e B, em um intervalo de tempo Δt=t-
t0, conforme mostra a figura.

Velocidade linear (𝒗
⃗⃗)
Qualquer partícula com movimento circular É uma grandeza física vetorial, cuja
uniforme descreve áreas iguais em tempos magnitude é definida como a derivada
iguais, com respeito a de um sistema de temporal do deslocamento linear S, ou seja:
referência localizado no centro da
circunferência. 𝑑𝑠
𝑣=
𝑑𝑡
Exemplo: Dois satélites A e B descrevem O comprimento do arco entre dois pontos A
trajetórias circulares de raios de curvatura e B qualquer da trajetória, encontra-se
R e 2R, respectivamente. Se os vetores de quando você integra essa equação da
posição, com respeito a o centro da seguinte maneira:
Período (T)
𝐵 𝑡 Chama-se período do M.C.U, o tempo que a
∫ 𝑑𝑠 = ∫ 𝑣. 𝑑𝑡 partícula gasta para fazer uma volta ou
𝐴 𝑡0
revolução completa, ou seja:
𝑡
𝑇=
s = v. (t − 𝑡0 ) 𝑁
sendo, t o tempo decorrido, e N o número de
Velocidade angular (𝝎
⃗⃗⃗⃗) voltas. A unidade de "T" é medido em
segundos.

Frequência (f)
A frequência do MCU é chamada de número
de voltas que a partícula faz em cada
unidade de tempo, ou seja:
𝑁
𝑓=
𝑡
É uma grandeza física vetorial, cuja Unidade de f se mede em 1/s (Hz).
magnitude é definida como a derivada
temporal do deslocamento angular θ, ou Relação entre T, f e ω.
seja:
𝑑θ 2𝜋
𝑤= ω= = 2𝜋. 𝑓
𝑑𝑡 𝑇

O ângulo θ varrido pelo vetor raio da Aprofundamento ITA/IME - Velocidade


partícula, durante o intervalo de tempo Δt=t- angular e produto vetorial
t0, será encontrado integrando esta equação,
assim:
𝐵 𝑡

∫ 𝑑θ = ∫ 𝑤. 𝑑𝑡
𝐴 𝑡0
θ = w. (t − 𝑡0 )

A unidade de w é rad/s.

IMPORTANTE: No ITA/IME velocidade Considere uma partícula que gira em torno


angular é considerada escalar. de um eixo com uma velocidade angular 𝜔 ⃗⃗
Seja 𝑟⃗⃗⃗ o vetor posição da partícula em
Relação entre v e ω relação a qualquer ponto do eixo. Como a
relação entre as magnitudes da velocidade partícula descreve um círculo de raio (r senθ)
linear v e velocidade angular ω, é dada por: situado no plano perpendicular ao eixo de
rotação, o módulo da velocidade linear
v = ω.R v = ω. r. senθ
e sua direção é perpendicular ao plano que
Demonstração: contém 𝑟⃗⃗⃗ e 𝜔
⃗⃗.
Pela regra do parafuso ou da mão direita,
S=Rθ podemos escrever
v. (t − 𝑡0 ) = w. (t − 𝑡0 ) 𝑣⃗ = ω
⃗⃗⃗ × r⃗
logo
v = ω.R Exemplo: Dois móveis, A e B, percorrem a
mesma circunferência com velocidades
angulares constantes e iguais a 2π rad/s e
3π rad/s. Se eles partem juntos do mesmo
ponto, determine o tempo que levam para
se encontrar:

a) quando se deslocam no mesmo


sentido.
Resolução do Professor Laerte Pereira:
b) quando se deslocam em sentidos
contrários. A Aceleração centrípeta em cada trecho
𝑣2
Resolução: a) Após um tempo t, os 𝑎=
𝑅
espaços angulares percorridos por A e B 𝑎𝐴𝐵 = 6𝑚/𝑠 2
são: 𝑎𝐵𝐶 = 5𝑚/𝑠 2
ΔθA = 2 π·t e ΔθB = 3 π·t Calculando o tempo
𝑑
𝑡=
𝑣

𝑡 = 2𝑠.

Transmissão de movimento
Podemos transmitir o movimento circular de
uma roda, ou polia, para outra utilizando
uma correia, ou corrente, entre as duas
Se os móveis deslocam-se no mesmo rodas.
sentido e B é mais rápido que A, eles se Esse procedimento é muito utilizado, por
encontram quando B dá uma volta a mais exemplo, na bicicleta, em que A representa
que A, ou seja: a coroa do pedal e B, a catraca da roda
ΔθB = ΔθA + 2 π ⇒ 3 π·t = 2 π·t + 2 π traseira.
⇒t=2s

b) Se os móveis deslocam-se em sentidos


contrários, eles se encontram quando a
soma dos ângulos descritos por A e B vale
2π rad:
Nesse caso, A e B movem-se no mesmo
ΔθB + ΔθA = 2 π sentido, e todos os pontos da corrente têm a
mesma velocidade escalar linear para
t = 0,4 s qualquer instante de tempo. Não havendo
escorregamento entre a corrente e a coroa
e/ou a catraca, qualquer ponto de A e de B
em contato com a corrente também deve ter
Exemplo: Um móvel viaja ao longo da
a mesma velocidade.
trajetória ABC, com módulo constante de
wA · RA = wB · RB
velocidade pela etapa AB e pela a etapa
Essa análise também é válida quando, em
BC. É conhecido que a aceleração na AB
vez de usar uma correia ligando as rodas,
é 6 m/s2 e em BC 5 m/s2. Encontre o
colocamo-las em contato diretamente.
intervalo de tempo que é necessário
percorrer a distância total ABC.
wA = wB

Exemplo: A figura mostra duas roldanas


concêntricas de raios a = 20 cm e b = 30
cm. Se as roldanas rodarem no sentido
horário com velocidade angular w
constante = 4 rad/s, determine o valor da
velocidade relativa entre os blocos A e B.
Exemplo: A figura representa um
acoplamento de três rodas dentadas A, B
e C que possuem 40, 10 e 20 dentes
respectivamente.

Resolução:
V = ωR
Para A: vA = (4)(20) = 80 cm/s (↓)
Lembrando que os dentes são todos
Para B: vB = (4)(30) = 120 cm/s (↓)
iguais, quantas voltas da a roda A
Logo
enquanto a roda C completa 10?
𝑣𝑟𝑒𝑙 = 40 𝑐𝑚/𝑠
Resolução:
perímetro é proporcional ao número de
dentes. Como o raio e o perímetro também Aprofundamento ITAIME - Transmissão
são proporcionais, o raio é proporcional ao de movimento através de cabos tensos
número de dentes. (Ideais).
Sejam ⃗⃗⃗⃗⃗
𝑣𝐴 e ⃗⃗⃗⃗⃗
𝑣𝐵 as respectivas velocidades
de dois objetos que estão ligados por uma
corda tensa como está representado na
figura. Se suas velocidades são ⃗⃗⃗⃗⃗
𝑣𝐴 e ⃗⃗⃗⃗⃗
𝑣𝐵 em
um dado instante, será verdade que as
nA = 5 magnitudes das componentes dessas
velocidades na direção da corda, para a qual
eles são vinculados são iguais.
Eixo de rotação comum a duas rodas
Também podemos transmitir o movimento
circular acoplando duas rodas ao mesmo
eixo de rotação.

Então temos:
Nesse caso, as rodas giram com a mesma
velocidade escalar angular do eixo de 𝑣𝐴 . cos θ𝐴 =𝑣𝐵 . cos θ𝐵
rotação. Assim:
Exemplo: A figura mostra um bloco
movendo-se horizontalmente por causa Resolução do Prof. Laerte Pereira:
de uma corda que a puxa no ponto A. Se Nossa estratégia será projetar a
no instante mostrado v1 = 15 m/s e a corda velocidade v2 na direção da corda.
AB forma o ângulo α = 53° com o piso,
somos solicitados a determinar em m/s a
o valor da velocidade v2 com que o bloco
desce.

Portanto, e como mostra o esquema


vetorial, será satisfatório que:
𝑣2 cos 𝜃 = 𝑣1
Onde
𝜃 = 90° − β
cos( 90° − β) = sin β
Então
𝑣2 sin β = 𝑣1
Resolução do Prof. Laerte Pereira: Dica do Laerte: aluno de turma ITA/IME
tem que saber deste triângulo.
Onde a = 37˚ e B = 53˚

Então
4
sin 53° =
𝛥𝑠1 cos 𝛼 = 𝛥𝑠2 5
Derivando em relação ao tempo
𝑣1 cos 𝛼 = 𝑣2 4
𝑣2 . = 18
Logo 5
𝑣2 = 9𝑚/𝑠
𝑣2 = 22,5𝑚/𝑠

Exemplo: A figura mostra um sistema


composto por dois blocos e uma corda MOVIMENTO CIRCULAR
ideal que é mantida esticada enquanto UNIFORMEMENTE VARIADO (M.C.U.V.)
eles estão em movimento. Se no instante Aceleração escalar média angular
mostrado v = 18,0 m/s e 𝛽 = 53°, o A figura abaixo mostra as posições de uma
Professor Laerte Pereira pede pra você partícula em movimento circular nos
determinar a velocidade v2 em m/s instantes t1 e t2. Nesses instantes, as
correspondente ao bloco 2. velocidades escalares instantâneas lineares
valem v1 e v2, enquanto as angulares valem
ω1 e ω2.
2𝜋
𝛾= 𝑟𝑎𝑑/𝑠 2
5
Função de Torricelli
𝜔2 = 𝜔02 + 2𝛾∆𝜃

2𝜋
100𝜋 2 = 4𝜋 2 + 2 ∆𝜃
5

Δϕ = 120 π rad

O quociente da variação de velocidade ∆ω regra de três


pelo intervalo de tempo ∆t em que ela ocorre
é a aceleração escalar média angular (γm: lê- 1 volta ⇒ 2 π rad
se gama m) nesse intervalo: n voltas ⇒ 120 π rad
logo
∆ω n = 60
𝛾𝑚 =
∆t
a unidade de medida da aceleração angular
será radiano por segundo ao quadrado Exemplo: A parte mais externa de um
(rad/s2). disco, com 0,25 m de raio, gira com uma
velocidade linear de 15 m/s. O disco
Relação entre a aceleração escalar média começa então a desacelerar
angular e a linear uniformemente até parar, em um tempo de
A aceleração escalar média angular (γm) é 0,5 min. Qual o módulo da aceleração
igual à aceleração escalar média linear (αm) angular do disco em rad/s2?
dividida pelo raio (R) da circunferência:
𝑎𝑚 Resolução do Prof. Laerte Pereira:
𝛾𝑚 =
R 𝑣 = 𝑣0 + 𝑎𝑡
Neste movimento, assim como em qualquer 15 = 𝑎30
outro movimento uniformemente variado, 𝑎 = −0,5𝑚/𝑠 2
valem as seguintes expressões: 𝑎𝑚
-Função da posição angular 𝛾𝑚 =
R
𝛾𝑡 2 −0,5
𝜃 = 𝜃0 + 𝜔0 𝑡 + 𝛾𝑚 = = −2 𝑟𝑎𝑑/𝑠 2
2 0,25
-Função da velocidade angular
𝜔 = 𝜔0 + 𝛾𝑡
-Função de Torricelli
𝜔2 = 𝜔02 + 2𝛾∆𝜃

Exemplo: Uma partícula em movimento Aprofundamento ITA/IME - Centro


circular uniformemente variado tem sua
velocidade angular alterada de 2π rad/s instantâneo de Rotação (C.I.R.)
para 10π rad/s durante 20 s. Calcule o Para corpos rígidos que descreve um
número de voltas que a partícula efetua
movimento genérico no plano (em duas
nesse intervalo de tempo.
dimensões), o conceito de centro
Resolução: instantâneo de rotação permite calcular
𝜔 = 𝜔0 + 𝛾𝑡 convenientemente a velocidade angular
10 π = 2 π + 𝛾20 desconhecida do corpo rígido ou
Logo velocidades lineares desconhecidas de
pontos no corpo rígido. O centro DICA IMPORTANTE: As distâncias rA e
instantâneo é um ponto imaginário, é como rB são geralmente determinadas a partir da
se fosse um "atalho" matemático que geometria do corpo rígido e da
permite o cálculo dessas incógnitas. trigonometria.
Observe que as linhas rA e rB são
O centro instantâneo também é chamado desenhadas na direção que resultará na
de centro instantâneo de velocidade zero convergência para um ponto (o centro
(CI). Está em um eixo imaginário de instantâneo IC). Observe também que a
velocidade zero, sobre o qual o corpo direção das velocidades vA e vB
parece girar em um dado instante. Este corresponde à direção de rotação do corpo
eixo é sempre perpendicular ao plano de rígido. Avelocidade do ponto A é dada por
movimento. vA = ω. rA (1)
Existem três casos básicos a serem A velocidade do ponto B é dada por
considerados ao resolver problemas
vB = ω. rB (2)
usando a abordagem do centro
instantâneo. Combinando as equações (1) e (2) para
Caso 1 - Considere um corpo rígido eliminar a variável ω, e obtemos
girando em um plano. Desejamos 𝑣𝐴 𝑟𝐴
determinar a velocidade de um ponto A no = (3)
𝑉𝐵 𝑟𝐵
corpo rígido, dada a velocidade conhecida
de outro ponto B no corpo rígido. As Exemplo: Encontre a velocidade do ponto
direções das velocidades são conhecidas B no mecanismo de manivela se a
e não são paralelas. A figura ilustra a velocidade do ponto A for vA, na direção
configuração do problema. mostrada. A informação é dada na figura.

Solução:
Configure a solução conforme descrito
abaixo, com as novas variáveis
Onde: apresentadas como mostrado. A linha rA é
vA é a velocidade do ponto A no instante desenhada de maneira que fique
mostrado, vB é a velocidade do ponto B, no perpendicular à velocidade vA. A linha rB é
instante mostrado, CI é a intersecção das desenhada como uma extensão da linha
linhas de rA e rB sendo rA a distância a partir que passa através do ponto O e ponto
do CI para o ponto A, no instante mostrado. B. O ponto B traça um círculo durante o
Essa distância é traçada de maneira que movimento da cambota. Por geometria, a
fique perpendicular ao vetor velocidade vA, velocidade vB é tangente a este círculo no
rB é a distância do CI ao ponto B, no ponto B. Também pela geometria, a linha
instante mostrado. Essa distância é rB é perpendicular à velocidade vB.
traçada como perpendicular à velocidade
vB e ω é a velocidade angular do corpo
rígido, no instante mostrado.
significa que rA e rB são infinitamente
longos, e o CI reside no infinito. Como
resultado, ω = 0 e todas as partículas no
corpo rígido têm a mesma velocidade (ou
seja, a magnitude das velocidades e a
direção das velocidades de todas as
partículas no corpo rígido são as mesmas).
Caso 2 - Considere um corpo rígido
girando em um plano. Desejamos
Pela geometria, a distância OA é dada por determinar a velocidade angular do corpo
OA = Lcos 𝛼 + cos 𝜃 (4) rígido, dadas as velocidades conhecidas
dos pontos A e B no corpo rígido. Essas
Também temos,
velocidades são paralelas e apontando na
Lsin 𝛼 = 𝑟 sin 𝜃 mesma direção. A linha que une os pontos
𝑟 A e B é perpendicular à direção das
α = sin−1 ( sin 𝜃) (5)
𝐿 velocidades. A figura abaixo ilustra a
Substitua a equação (5) na equação (4) e configuração do problema.
obtemos
𝑟
OA = Lcos (sin−1 ( sin 𝜃)) + cos 𝜃
𝐿
Os pontos O, A e CI formar um triângulo
retângulo. Portanto, podemos usar a
trigonometria para resolver rA e rB:
𝑟𝐴 = 𝑂𝐴. tan 𝜃 (6)
𝑂𝐴
𝑟𝐵 + 𝑟 =
cos 𝜃
𝑂𝐴 Observe que CI é a interseção da linha que
𝑟𝐵 = − 𝑟 (7)
cos 𝜃 passa pelos pontos A e B, e a linha que une
a ponta da vetores vA e vB. A distância entre
Com vA conhecido, substitua as
os pontos A e B é d. Por semelhança de
equações (6) e (7) pela equação (3),
triângulos:
para resolver a velocidade do ponto B
(vB). Portanto, 𝑣𝐵 − 𝑣𝐴 𝑣𝐵 𝑣𝐴
= = (8)
𝑑 𝑟𝐵 𝑟𝐴
𝑂𝐴 − 𝑟. cos 𝜃
𝑣𝐵 = 𝑣𝐴 . A velocidade do ponto A é dada por:
𝑂𝐴. sin 𝜃
vA = ω. rA (9)

Para o Caso 1, existe um caso especial A velocidade do ponto B é dada por:


em que as velocidades nos pontos A e B VB= ω. rB (10)
estão apontando na mesma direção
Combinação das equações (8), (9) e (10),
(paralela), e a linha que passa pelos pontos
segue-se que a velocidade angular é
A e B não é perpendicular à direção dessas
𝑣𝐵 − 𝑣𝐴
velocidades. Nesse caso, as linhas rA e rB 𝛚= (11)
são paralelas e não se cruzam. Isto 𝑑
Exemplo: Uma roda de raio R tem A velocidade do ponto A é dada por
velocidades como mostrado nos pontos A
vA = ω. rA (13)
e B. Determine a velocidade angular da
roda em torno do ponto B por exemplo. A velocidade do ponto B é dada pela
equação

VB = ω. rB (14)
Da combinação das equações (12) a (14),
segue-se que a velocidade angular é
𝑣𝐵 + 𝑣𝐴
𝛚= (15)
Resolução: 𝑑
A distância entre os pontos A e B é 2R. Na
equação (11), a velocidade angular é Exemplo: Uma roda de raio R tem
𝑣𝐵 − 𝑣𝐴 velocidades como mostrado nos
𝛚=
2𝑅 pontos A e B. Determine a velocidade
angular da roda.
Caso 3 - Considere um corpo rígido
girando em um plano. Desejamos
determinar a velocidade angular do corpo
rígido, dadas as velocidades dos pontos A
e B no corpo rígido. Essas velocidades
estão apontando em paralelo, mas em
direções opostas. A linha que une os
pontos A e B é perpendicular à direção Solução:
das velocidades. A figura abaixo ilustra a A distância entre os pontos A e B é 2 R.
configuração do problema. Na equação (15), a velocidade angular é
𝑣𝐵 + 𝑣𝐴
𝛚=
2𝑅

Observe que a localização espacial do


centro instantâneo pode mudar com o
tempo. Portanto, em geral, seu uso se
aplica apenas no instante considerado, o
que corresponde às informações
fornecidas no problema, no instante
Observe que CI é a interseção da linha que
considerado. Em geral, se as velocidades
passa pelos pontos A e B, e a linha que
dos pontos A e B mudam, e / ou suas
une a ponta de vetores de vA e vB. Ao
direções mudam, um novo centro
contrário do caso anterior,o CI encontra-
instantâneo deve ser calculado para
se entre os pontos A e B. A distância entre resolver as (novas) velocidades lineares
os pontos A e B é d. Por triângulos desconhecidas ou velocidades angulares.
semelhantes:
𝑣𝐵 + 𝑣𝐴 𝑣𝐵 𝑣𝐴 Em geral, a abordagem de centro
= = (12) instantâneo não deve ser usada para
𝑑 𝑟𝐵 𝑟𝐴
encontrar a aceleração de pontos em um Exercícios
corpo rígido. Isso ocorre porque o centro
instantâneo (CI) geralmente não tem 1. (Espcex (Aman) 2022) Em um parque de
aceleração zero. diversão, dois carrinhos, A e B, descrevem
um movimento circular uniforme em pistas
distintas, concêntricas, muito próximas e de
raios RA e RB respectivamente. Quando se
movem no mesmo sentido, os carrinhos
encontram-se, lado a lado, a cada 40 s e,
quando se movem em sentidos opostos, o
encontro ocorre a cada 10 s. Os carrinhos
possuem velocidades escalares diferentes, e
os respectivos módulos das velocidades
escalares são os mesmos nas duas
situações descritas. Podemos afirmar que a
razão entre o módulo da velocidade escalar
do carrinho A e do carrinho B é de:
a) 10RA 3RB
b) 2RA RB
c) 5RA 3RB
d) 8RA 5RB
e) RA 4RB

2. Uma pessoa parada sobre a linha do


equador terrestre apresenta uma velocidade
tangencial, devido à rotação da Terra, de
módulo próximo a 1.700 km h.

Sabendo que sen 21 = 0,36 e


cos 21 = 0,93, uma pessoa em repouso
sobre o solo, em São José do Rio Preto, cuja
latitude é aproximadamente  = 21 Sul,
tem uma velocidade tangencial de módulo
próximo a
a) 1.830 km h.
b) 610 km h.
c) 1.700 km h.
d) 4.700 km h.
e) 1.580 km h.

3. Em uma fábrica, um técnico deve medir a


velocidade angular de uma polia girando. Ele
apaga as luzes do ambiente e ilumina a peça
somente com a luz de uma lâmpada
estroboscópica, cuja frequência pode ser
continuamente variada e precisamente
conhecida. A polia tem uma mancha branca
na lateral. Ele observa que, quando a
frequência de flashes é 9 Hz, a mancha na a) Considerando-se os pontos x e y
polia parece estar parada. Então aumenta indicados na figura, qual deles terá menor
vagarosamente a frequência do piscar da velocidade linear? Explique sua resposta.
lâmpada e só quando esta atinge 12 Hz é b) Pedalando em uma bicicleta com as
que, novamente, a mancha na polia parece dimensões descritas, um ciclista foi
estar parada. Com base nessas instruído de que, para vencer uma corrida,
observações, ele determina que a velocidade deve se manter à velocidade constante de
angular da polia, em rpm, é 65 km h durante toda a prova. Qual o
a) 2.160 número de pedaladas por segundo que
b) 1.260 ele deve dar para manter a velocidade
c) 309 indicada?
d) 180
e) 36 6. Considere o movimento de rotação de dois
objetos presos à superfície da Terra, sendo
4. Uma partícula de massa 12,5 g se move um deles no equador e o outro em uma
em movimento circular e uniforme com latitude norte, acima do equador.
frequência igual a 120 rpm, numa trajetória Considerando somente a rotação da Terra,
de raio igual a 0,5 m. Considerando g = 10,0 para que a velocidade tangencial do objeto
m/s2 e π = 3,14, a respeito do movimento da que está a norte seja a metade da velocidade
partícula, é CORRETO afirmar que do que está no equador, sua latitude deve ser
a) a força resultante sobre a partícula é nula, a) 60.
já que o movimento é uniforme. b) 45.
b) a partícula completa uma volta a cada 2 s. c) 30.
c) o vetor aceleração centrípeta é paralelo ao d) 0,5.
vetor velocidade durante todo o
movimento. 7. Um pequeno motor a pilha é utilizado para
d) a força resultante que atua sobre a movimentar um carrinho de brinquedo. Um
partícula possui módulo constante igual a sistema de engrenagens transforma a
1,25  10−1N. velocidade de rotação desse motor na
e) o módulo da aceleração vetorial média velocidade de rotação adequada às rodas do
para o deslocamento entre duas posições carrinho. Esse sistema é formado por quatro
diametralmente opostas é igual a 50,24 engrenagens, A, B, C e D, sendo que A
m/s2. está presa ao eixo do motor, B e C estão
presas a um segundo eixo e D a um terceiro
5. Assuma que as dimensões das eixo, no qual também estão presas duas das
engrenagens e do pneu de uma bicicleta quatro rodas do carrinho.
sejam as indicadas abaixo.
Nessas condições, quando o motor girar com
frequência fM, as duas rodas do carrinho
girarão com frequência fR . Sabendo que as Considere rA , rB , rC , e rD , os raios das
engrenagens A e C possuem 8 dentes, que engrenagens A, B, C e D,
as engrenagens B e D possuem 24 dentes, respectivamente. Sabendo que rB = 2  rA e
que não há escorregamento entre elas e que que rC = rD , é correto afirmar que a relação
fM = 13,5 Hz, é correto afirmar que fR , em ω1
é igual a
Hz, é igual a ω2
a) 1,5. a) 1,0.
b) 3,0. b) 0,2.
c) 2,0. c) 0,5.
d) 1,0. d) 2,0.
e) 2,5. e) 2,2.

8. A figura representa, de forma simplificada, 9 O funcionamento de um dispositivo seletor


parte de um sistema de engrenagens que de velocidade consiste em soltar uma esfera
tem a função de fazer girar duas hélices, H1 de uma altura h para passar por um dos
e H2 . Um eixo ligado a um motor gira com orifícios superiores (O1, O2, O3, O4) e,
sucessivamente, por um dos orifícios
velocidade angular constante e nele estão
inferiores (P1, P2, P3, P4) conforme ilustrado
presas duas engrenagens, A e B. Esse eixo a seguir.
pode se movimentar horizontalmente
assumindo a posição 1 ou 2. Na posição 1,
a engrenagem B acopla-se à engrenagem C
e, na posição 2, a engrenagem A acopla-se
à engrenagem D. Com as engrenagens B e
C acopladas, a hélice H1 gira com
velocidade angular constante ω1 e, com as
engrenagens A e D acopladas, a hélice H2
gira com velocidade angular constante ω2 .

Os orifícios superiores e inferiores mantêm-


se alinhados, e o sistema gira com
velocidade angular constante .
Desprezando a resistência do ar e
considerando que a esfera é liberada do
repouso, calcule a altura máxima h para que
a esfera atravesse o dispositivo.

10 Considere um computador que armazena A frequência de giro do ponteiro, em rpm, é


informações em um disco rígido que gira a a) 1
uma frequência de 120 Hz. Cada unidade de b) 2
informação ocupa um comprimento físico de c) 4
0,2 mm na direção do movimento de rotação d) 81
do disco. Quantas informações magnéticas e) 162
passam, por segundo, pela cabeça de
leitura, se ela estiver posicionada a 3,0 cm do 12. Um disco horizontal dotado de um furo
centro de seu eixo, como mostra o esquema circular gira horizontalmente em rotação
simplificado apresentado abaixo? (Considere uniforme com frequência f, conforme ilustra o
π ≅ 3.) esquema a seguir. Uma bola, cujo diâmetro
é pouco menor que o desse furo, será
abandonada do repouso de uma altura H em
relação à superfície girante e deverá
despencar verticalmente em queda livre de
modo a trespassar o furo existente no disco
enquanto este realiza um número N (inteiro)
de voltas.

a) 1,62.106.
b) 1,8.106.
c) 64,8.108.
d) 1,08.108.

11. (Enem) A invenção e o acoplamento


entre engrenagens revolucionaram a ciência
na época e propiciaram a invenção de várias Sendo g a intensidade da aceleração da
tecnologias, como os relógios. Ao construir gravidade, determine o valor de N em função
um pequeno cronômetro, um relojoeiro usa o de H, g e f.
sistema de engrenagens mostrado. De
acordo com a figura, um motor é ligado ao 13. No esquema abaixo estão
eixo e movimenta as engrenagens fazendo o representadas, fora de escala, duas polias, A
ponteiro girar. A frequência do motor é de 18 e B, rigidamente acopladas entre si e
rpm, e o número de dentes das engrenagens trespassadas pelo eixo de um motor em O. O
está apresentado no quadro. raio de A é RA = 40 cm e o de B é RB = 30
cm. Em A e B estão enrolados fios ideais que
prendem em suas extremidades livres dois
blocos, 1 e 2, de dimensões desprezíveis,
inicialmente distantes 4,0 m do nível
horizontal que contém o eixo O.
Filma-se um ventilador cujas pás estão
girando no sentido horário. O ventilador
possui quatro pás simetricamente dispostas,
uma das quais pintada de cor diferente, como
ilustra a figura abaixo:

Ao projetarmos o filme, os fotogramas


aparecem na tela na seguinte sequência:
Num determinado instante, o motor é ligado,
impondo ao seu eixo uma aceleração escalar
angular de módulo 0,2 rad/s2, o que faz as
polias girarem no sentido horário a partir do
repouso e os blocos subirem.
a) Qual dos blocos, 1 ou 2, atinge primeiro o que nos dá a sensação de que as pás estão
nível horizontal que contém o eixo O? girando no sentido anti-horário.
b) Depois de quanto tempo da situação inicial Calcule quantas rotações por segundo, no
o bloco “vencedor” atinge esse nível? mínimo, as pás devem estar efetuando para
que isso ocorra.
14. Uma motocicleta encontra-se em Nota:
• A ilusão de que as pás estão girando no sentido oposto ao
movimento em uma estrada asfaltada. Cada real é devida ao fato de nosso cérebro interpretar que o
uma de suas rodas tem raio R = 25 cm e gira movimento, de um fotograma para o outro, se dá no sentido
com frequência f = 10 Hz. Sabendo que as do menor deslocamento angular.
rodas não deslizam no asfalto, calcule a
velocidade da moto em km/h. (Use π = 3,1.) 17. A função horária do espaço angular de
uma partícula que descreve uma
15. (Fuvest-SP) Qual a ordem de grandeza circunferência de raio igual a 2 m é:
do número de voltas dadas pela roda de um
automóvel ao percorrer uma estrada de 200
km? com ϕ em radianos e t em segundos.
a) 102 Determine:
b) 103 a) a fase inicial;
c) 105 b) o período e a frequência;
d) 107 c) a velocidade escalar linear (admite-se π
e) 109 na resposta)

16. (UFRJ) O olho humano retém durante 18. A figura representa dois discos de
1/24 de segundo as imagens que se formam papelão fixados a um mesmo eixo, com
na retina. Essa memória visual permitiu a rotação de frequência igual a 50 Hz. Os
invenção do cinema. A filmadora bate 24 discos foram fixados em locais do eixo
fotografias (fotogramas) por segundo. distantes 2 m um do outro.
Uma vez revelado, o filme é projetado à
razão de 24 fotogramas por segundo. Assim,
o fotograma seguinte é projetado no exato
instante em que o fotograma anterior está
desaparecendo de nossa memória visual, o
que nos dá a sensação de continuidade.
Um projétil é disparado paralelamente ao em volta de toda a roda, como ilustrado em
eixo, descolando-se em movimento suposto (b). A marca do pneu volta a ser nítida,
retilíneo e uniforme, per furando os dois mesmo com o carro em movimento, quando
discos. O ângulo entre o plano que contém o esse atinge determinada velocidade. Essa
eixo e o furo no primeiro disco e o plano que ilusão de movimento na imagem gravada é
contém o eixo e o furo no segundo disco é devida à frequência de gravação de 30
igual a 45°. Determine a velocidade do quadros por segundo (30 Hz). Considerando
projétil, sabendo que, entre as duas per que o diâmetro do pneu é igual a 0,6 m e π =
furações, os discos giraram menos que meia 3,0, responda:
volta.
a) Quantas voltas o pneu completa em um
19. (ITA-SP) Uma partícula move-se ao segundo quando a marca filmada pela
longo de uma circunferência circunscrita em câmera aparece parada na imagem, mesmo
um quadrado de lado L com velocidade estando o carro em movimento?
angular constante. Na circunferência inscrita b) Qual a menor frequência angular ω do
nesse mesmo quadrado, outra partícula pneu em movimento quando a marca
move-se com a mesma velocidade angular. aparece parada?
A razão entre os módulos das respectivas c) Qual a menor velocidade linear (em m/s)
velocidades lineares dessas partículas é: que o carro pode ter na figura (c)?

a) √2 22. Considere a Terra perfeitamente esférica


b) 2√2 e suponha um aro nela ajustado, na linha do
√2 equador (que mede aproximadamente 40
c) 000 km).
2
3
d) √2
√3
e) 2

20. Considere dois pilotos A e B que, ao


disputarem uma prova de automobilismo,
percorrem o circuito no mesmo sentido e com
velocidades escalares constantes. O piloto A
completa uma volta em 1 min 40 s, enquanto Se o comprimento desse aro for aumentado
o piloto B faz o mesmo em 1 min 36 s. de 1 m, surgirá uma folga x entre ele e a
Supondo que, em determinado instante, B Terra, como está indicado na figura. Dentre
esteja ao lado de A, quanto tempo depois as alternativas seguintes, indique aquela que
dessa situação a vantagem de B sobre A traz o maior animal capaz de passar por essa
será de um quarto de volta? folga.
a) pulga
21. (Unicamp-SP) O quadro (a), abaixo, b) aranha
refere-se à imagem de televisão de um carro c) rato
parado, em que podemos distinguir d) gato
claramente a marca do pneu (“PNU”). e) elefante

23. (Olimpíada Brasileira de Física) Em


Física, define-se a quantidade de movimento
angular (momento angular), L, de um corpo
que gira com velocidade angular constante ω
Quando o carro está em movimento, a em torno de um eixo como sendo L = I ω, em
imagem da marca aparece como um borrão
que I é uma grandeza denominada momento
de inércia, que depende da massa do corpo
e de como ela está distribuída em torno do
eixo de rotação. Para um disco de massa M
e raio R, o momento de inércia em relação a
um eixo perpendicular a ele, passando pelo
𝑀𝑅 2
seu centro, é dado por 𝐼 = 2 .
Considere um disco como esse, de raio 10
cm, girando com frequência de 0,5 Hz.
26. (ITA) Um problema clássico de
Quantas voltas serão dadas em 15 segundos
cinemática considera objetos que, a partir de
por um outro disco que possui a mesma
certo instante, se movem conjuntamente
massa do primeiro disco e metade do seu
com velocidade de módulo constante a partir
raio, tendo, porém, o mesmo momento
dos vértices de um polígono regular, cada
angular?
qual apontando à posição instantânea do
objeto vizinho em movimento. A figura
24. Uma pequena fonte de luz (laser), situada
mostra a configuração desse movimento
a uma distância d de um paredão P, gira em
múltiplo no caso de um hexágono regular.
torno de um eixo E, com velocidade angular
constante ω, emitindo horizontalmente um
estreito feixe de luz que varre o paredão. A
“mancha” dessa luz no paredão move--se em
linha reta ao longo dele, com velocidade
instantânea de módulo v. A figura representa
o evento visto de cima:
Considere que o hexágono tinha 10,0 𝑚 de
lado no instante inicial e que os objetos de
movimentam com velocidade de módulo
constante de 2,00 𝑚/𝑠.
Após quanto tempo estes se encontrarão e
qual deverá ser a distância percorrida por
cada um dos seis objetos?
a) 5,8𝑠 e 11,5𝑚.
expresse v em função de ω, d e do ângulo θ b) 11,5𝑠 e 5,8𝑚.
indicado na figura. c) 10,0𝑠 e 20,0𝑚.
d) 20,0𝑠 e 10,0𝑚.
25. À noite, numa quadra esportiva, uma e) 20,0𝑠 e 40,0𝑚.
pessoa de altura h caminha em movimento
retilíneo e uniforme com velocidade escalar 27. (Laerte Pereira) A haste AB pode
v. Apenas uma lâmpada L, que pode ser deslizar livremente ao longo do piso plano e
horizontal e do plano inclinado ambos lisos.
considerada uma fonte luminosa puntiforme
Sabendo que a velocidade do ponto A no
e que se encontra a uma altura H do piso,
instante mostrado na figura é vA, determine o
está acesa. Determine, em função de H, h e módulo da velocidade da extremidade B.
v, a velocidade escalar média vE da
extremidade E da sombra da pessoa
projetada no chão.
Obs.: Tente resolver este exercício usando o
CIR.
31. Uma roda de raio R rola sem escorregar
28. (Laerte Pereira) No sistema, qual o sobre um plano horizontal, com o seu centro
número de revoluções em torno de seu eixo se movendo a uma velocidade V em relação
que a engrenagem A fará se ao mesmo ao solo (veja figura).
tempo a roda dentada maior (1) fizer n1
revoluções e a engrenagem central (2), fizer
n2 revoluções? O raio interno da roda
dentada é R e da engrenagem central (2) é r
n1 > n₂.

Determine a velocidade dos pontos A, B, C e


D em relação ao solo.

32. Uma roda de raio R rola sem escorregar


29. (Laerte Pereira) Determine o módulo da sobre um plano horizontal, com o seu centro
velocidade do colarinho quando ele está a 2 O movendo-se a uma velocidade V em
m do ponto A, sabendo que o colarinho relação ao solo (veja figura).
desce com rapidez constante de 5 m/s em
relação ao eixo AB.

O contato com o solo e feito por meio de um


volante de raio r < R. Determine a velocidade
dos pontos A, B, C e D.
30. Na figura, temos duas polias coaxiais A e 33. Uma roda de centro O e raio R rola sem
B de raios RA = 20 cm e RB = 10 cm e uma escorregar sobre um plano horizontal
outra polia C de raio RC = 50 cm: estando seu centro O animado de velocidade
V → em relação a Terra.

O bloco X, que parte do repouso em t = 0,


desce com aceleração escalar constante e
igual a 4 m/s2 . Não há deslizamento entre as
polias. Calcule a velocidade angular da polia Determine as velocidades dos pontos P e Q
C num instante genérico t. indicados na figura em relação ao solo.
34. Um cone reto de altura h tem seu vértice começa a se enrolar sobre a superfície do
O fixo no piso horizontal, e gira de modo que cilindro. Calcule quanto tempo após o qual o
sua velocidade angular em relação a um eixo disco atingirá o cilindro.
vertical que passa por O é Ω. Determine a
velocidade angular do cone em relação a seu 37. (Renato Brito) Um carro esportivo que
eixo OA. usa rodas de magnésio com o aspecto
mostrado na figura foi levado para fazer
balanceamento.

35. (Laerte Pereira) Uma haste AB está se


Lá chegando, as rodas foram encaixadas
movendo em um círculo fixo de raio R com
numa máquina que produz um giro de 900
uma velocidade constante v, como mostrado
RPM no sentido horário. Durante esse
na figura. P é o ponto de interseção da haste
processo, a sala fica escura e uma lâmpada
e do círculo. Em um determinado instante, a
estroboscópica pisca rapidamente com uma
haste está a uma distância x = 3R/5 do centro
frequência de 24 flashes por segundo.
do círculo. A velocidade da haste é normal ao
Durante esse processo, pergunta-se:
seu comprimento e a haste sempre
permanece paralela ao diâmetro CD.
a) As rodas parecerão girar no sentido
Encontre a velocidade do ponto P neste
horário ou anti-horário?
instante.
b) E vão girar com qual frequência aparente
em RPM?

38. Um míssil é disparado verticalmente e


rastreado pelo radar 𝑅, conforme mostrado
na figura abaixo. Em uma posição particular
𝑑𝜃
𝜃, os parâmetros medidos são 𝑟 e 𝑑𝑡 .

36. (Laerte Pereira) Um cilindro vertical de


raio R é colocado em uma superfície
horizontal. Um pequeno disco M está preso
ao cilindro por um fio horizontal MB de
comprimento l0 e uma das extremidades está
presa ao cilindro no ponto B, conforme
mostrado.

Mostre que a velocidade do foguete nesta


posição é dada por:
𝑑𝜃
O disco é lançado com uma velocidade inicial 𝑣 = (𝑟 ∙ 𝑐𝑜𝑠𝜃) ∙ 𝑠𝑒𝑐 2 𝜃 ∙ ( )
v0 (constante em valor) de modo que o fio 𝑑𝑡
Gabarito
1ª Solução: Usando conceitos
1ª. C matemáticos.
Equações horárias dos ângulos percorridos Como a mancha branca parece estar parada,
pelos carrinhos (partindo do mesmo ângulo a frequência de rotação da polia deve ser um
inicial): número múltiplo das frequências de 9 Hz e
θA = ω A t =
vA
t
12 Hz. E o menor valor para o qual isto é
RA possível deve ser o mínimo múltiplo comum
v entre eles:
θB = ωB t = B t
RB mmc ( 9,12 ) = mmc ( 32 ,3  22 ) = 32  22 = 36

Do enunciado, temos que: Sendo assim, a sua frequência é de:


 vA v f = 36 Hz = 36  60 rpm
  40 − B  40 = 2π
θA − θB = 2π RA RB  f = 2160 rpm
  
θ
 A ' + θB ' = 2 π  A  10 + vB  10 = 2π
v
 R A RB 2ª Solução: Usando conceitos físicos.
4v A 4vB v A vB 3v 5v Para que a mancha branca pareça parada, a
 − = +  A = B
RA RB R A RB RA RB cada período entre duas piscadas
v 5R A consecutivas da lâmpada, a polia deve
 A = efetuar um número inteiro de voltas. Quanto
vB 3RB
maior a frequência, menor o período: menos
voltas inteiras a polia efetua.
Usando a relação período-frequência, os
2 Resposta:
períodos correspondentes às frequências
[E]
dadas são:
A velocidade angular é a mesma para todos
 1
os pontos da Terra. T1 = 9 s
1
T= 
f T = 1 s
 2 12

O fenômeno de a mancha parecer parada à


medida que a frequência aumenta, repete-se
cada vez que a polia dá 1 volta a menos.
1
Assim, se para o período de s, a polia
9
1
efetua n voltas, para o de s, serão (n – 1)
12
𝑣𝑒𝑞 𝑣𝑆𝐽 voltas. Sendo T o período da polia, têm-se:
 1
𝜔𝑒𝑞 = 𝜔𝑆𝐽 ⇒ = T1 = nT  9 = nT 1
𝑅̸ 𝑅̸ 𝑐𝑜𝑠 𝛷 
T = ( n − 1) T  1 = ( n − 1) T
 
1
9 = nT
(n − 1) T
 12 ( n − 1) = 9n 

⇒ 𝑣𝑆𝐽 = 𝑣𝑒𝑞 𝑐𝑜𝑠 𝛷 = 1700 × 0,93  2 12


12

𝑚 4 ( n − 1) = 3n  n = 4 voltas
⇒ 𝑣𝑆𝐽 ≅ 1.580 𝑠
Retomando a primeira expressão:
3 Resposta: 1
[A] 𝑇1 = 𝑛𝑇 ⇒ = 4𝑇
9
1
Nota: há uma imprecisão no enunciado, pois ⇒ T= 𝑠
36
rpm é unidade de frequência e não de ⇒ 𝑓 = 36𝐻𝑧 = 36(60)rpm
velocidade angular, que, no SI, é rad/s. ⇒ 𝑓 = 2.160𝑟𝑝𝑚
4 Resposta: 37) Os pontos x e y encontram-se
[E] sobre discos que giram com a
mesma velocidade angular por
Dados: f = 120 rpm = 2 Hz; m = 12,5 g = 12,5 terem o mesmo eixo de rotação.
 10–3 kg; R = 0,5; g = 10 m/s2; π = 3,14. Sendo assim:
vb v
ωb = ωc  = c
O período do movimento (T) é o inverso da Rb Rc
frequência.
T=
1 1
 T= s Como Rc  Rb  vc  vb . Portanto, o
f 2
ponto y terá maior velocidade linear.

Se o movimento da partícula é uniforme, a b) Da relação anterior, vem:


velocidade linear é constante. Calculando vb v v 65 3,6
para 1 volta: = c  b =
Rb Rc 6 20
ΔS 2 π R
v= =  v = 2 πR f vb = v a  5,42 m s
Δt T
v a = 2πfaRa  5,42 = 2  3,14  fa  0,18
O intervalo de tempo entre duas passagens  fa  4,8 Hz
consecutivas por dois pontos diametralmente
opostos, como mostrado na figura, é igual à 6 Resposta:
metade do período. [A]
T
1 A figura ilustra a situação, considerando a
Δt =  Δt = 2  Δt = 0,25s Terra esférica.
2 2

Todos os pontos da Terra têm a mesma


Calculando o módulo da aceleração vetorial velocidade angular. Assim, para V = 2v,
média: tem-se:
|𝛥𝑣⃗| |−𝑣⃗ − 𝑣⃗| v V v 2v R
|𝑎⃑𝑚 | = = =  =  r= .
𝛥𝑡 𝛥𝑡 r R r R 2
|−2𝑣⃗|
⇒ |𝑎⃗𝑚 | = =
𝛥𝑡 Mas:
2(2𝜋𝑅𝑓) 2(2 ⋅ 3,14 ⋅ 0,5 ⋅ 2) r R2 1
= = cos θ = =  cos θ =  θ = 60.
𝛥𝑡 0,25 R R 2
𝑚
|𝑎⃗𝑚 | = 50,24 2
𝑠 7 Resposta:
5 [A]
Resposta: Os raios das engrenagens ® e os números
de dentes (n) são diretamente proporcionais.
Assim:
RA RC nA 8 1 T
= = = = . t = .
RB RD nB 24 3 4
2
Mas: T = .
- A e B estão acopladas tangencialmente: 
v A = v B  2 π fA R A = 2 π f B R B  fA R A = f B R B .
2 
f Então: t =  t = .
Mas : fA = f M  f M R A = f B R B
R
 fB = fM A = fM
1
 fB =
M
. 4 2
RB 3 3
Aplicando a equação do espaço para o
- B e C estão acopladas coaxialmente:
percurso H, temos:
fM
fC = f B =
2
. g    g  
( t )
2
3 H = v t +  H = v +  
2  2  2  2 
- C e D estão acopladas tangencialmente:
v C = vD  2 π f C R C = 2 π f D R D  fC RC = f D R D . 
R fM 1 fM
   g 2

 = H−
Mas : f D = f R  f C RC = f R R D  f R = fC C  fR =  fR = 
RD 3 3 9 v
FR =
13,5
 f R = 1,5 Hz.
 2  2 42
9
2 g 2 2 
v= H− 
 2 4 2 
8 Resposta: 2
 2   (II).
[D] v2 =  H− g
  4 

Na posição 1:
 rB = 2 r A .
Igualando (I) e (II), vem:
 2
 ωB = ωA 
vB
= ωA 
vB
= ωA  vB = 2 ω A r A . 2 g h =  2 H −  
g 

 rB 2 rA   4 
 v C = vB  ωC rC = 2 ωA rA . 1  2 2
 h=  
 H− g
 ωC = ω1  ω1rC = 2 ωA rA . (I) 2 g   4 

Na posição 2: 10 Resolução:
 vD = v A  ω D rD = ωA rA . Cálculo do comprimento da trilha de

 ω2 = ωD .  ω2 rC = ωA rA . (II) informação, Δs, que passa sob o cabeçote
 r =r .
 C D leitor durante Δt = 1,0 s:

Dividindo membro a membro (I) por (II): Δs


= 2𝜋𝑅𝑓
ω1 rC 2 ωA rA ω1 Δt
=  = 2.
ω2 rC ωA rA ω2
Δs = 21,6 m

9 Resposta: Sendo N o número de unidades de


Seja v a velocidade da esfera ao atingir um informação magnética procurado,
orifício superior. Aplicando Torricelli: tem-se:
v 2 = v 02 + 2 g h .
Δs
𝑁= =1,08.108 unidades
Δt
Como a velocidade inicial é nula:
v2 = 2 g h (I)
11Resposta: b
A altura máxima h é aquela que faz com que
12 O tempo de queda
a esfera atravesse o dispositivo percorrendo
a altura 2𝐻
𝑡𝑞𝑢𝑒𝑑𝑎 = √
H no menor intervalo de tempo, 𝑔
correspondente ao tempo para o cilindro dar
O intervalo de tempo 𝑡𝑞𝑢𝑒𝑑𝑎 corresponde a
¼ de volta, ou seja, o tempo é ¼ do período
de rotação do dispositivo. Assim: N períodos T de rotação do disco, isto é:
𝑡𝑞𝑢𝑒𝑑𝑎 = 𝑁. 𝑇
Recordando-se que o período é o inverso da 𝑣𝑃 = 𝑣𝐸
frequência, vem: Como a velocidade do eixo em relação à
2𝐻 estrada é igual à velocidade vM da moto,
𝑁 = 𝑓. √ temos:
𝑔 𝑣𝑃 = 𝑣𝑀
Portanto, a velocidade da moto tem o mesmo
13 Resolução do prof. Laerte Pereira: valor da velocidade do ponto P em seu
a) Chega primeiro no nível horizontal que movimento circular em torno do eixo:
contém o eixo O, o bloco 1, içado pela polia vP = ωP R = 2π f R
A de maior raio, que imprime a esse bloco
uma velocidade escalar linear de maior vM = 2 · 3,1 · 10 · 0,25
intensidade que a do bloco 2. ⇒ vM = 15,5 m/s = 56 km/h
b) Δs1 = ΔwA RA ⇒ 4,0 = ΔwA.0,40
15. Estimando o raio da roda em 30 cm,
⇒ ΔwA = 10 rad calculemos seu perímetro, que é a distância
𝛾𝑡 2 percorrida por ela em cada volta:
𝜃 = 𝜃0 + 𝜔0 𝑡 + perímetro = 2π R = 2 · π · 30 cm ≌ 1,9 m
2
0,2𝑡 2
10 = 𝑑𝑖𝑠𝑡â𝑛𝑐𝑖𝑎 𝑡𝑜𝑡𝑎𝑙
2 𝑛ú𝑚𝑒𝑟𝑜 𝑑𝑒 𝑣𝑜𝑙𝑡𝑎𝑠 =
Logo 𝑃𝑒𝑟𝑖𝑚𝑒𝑡𝑟𝑜
200 000 m
T=10s 𝑛ú𝑚𝑒𝑟𝑜 𝑑𝑒 𝑣𝑜𝑙𝑡𝑎𝑠 =
1,9 m

14 Resolução: Ordem de grandeza = 105 voltas


Na figura a seguir, representamos uma roda
da moto em duas posições (1) e (2). Da 16 Resolução do prof. Laerte Pereira:
posição (1) até a posição (2), a roda Entre dois fotogramas consecutivos, a pá
completa uma volta. O ponto P está na destacada efetua, no mínimo, ¾ de volta, em
1
periferia da roda. um intervalo de tempo ∆𝑡 = 24 𝑠. . Então, a
frequência mínima de rotação das pás é
dada por:
3
𝑛 𝑣𝑜𝑙𝑡𝑎
𝑓= =4 = 18 𝑣𝑜𝑙𝑡𝑎𝑠/𝑠
∆𝑡 1
𝑠
24

Imagine que a periferia da roda, na posição 17 Basta comparar com


(1), esteja pintada com uma estreita faixa de
tinta vermelha fresca. O comprimento dessa 𝜃 = 𝜃0 + 𝜔𝑡
faixa é 2π R (perímetro da circunferência).
De (1) para (2), a roda deixa no asfalto uma
marca vermelha de mesmo comprimento,
pois a roda não desliza na pista. Note, então,
que, num mesmo intervalo de tempo, o ponto 18 Resolução do prof. Laerte Pereira:
P percorre 2π R em relação ao eixo da roda Enquanto o projétil desloca-se de um disco a
e este também percorre 2π R em relação à outro, percorrendo Δs = 2 m com velocidade
estrada. Portanto, a velocidade vP, do ponto v, o sistema sofre um deslocamento angular
P em relação ao eixo, é igual à velocidade vE,
do eixo em relação à estrada:
𝛥𝑠 𝛥𝑠 Isso significa que, a cada 1/30 s (período de
𝑣= → 𝛥𝑡 =
𝛥𝑡 𝑣 filmagem), a marca encontra-se na mesma
posição em relação ao eixo da roda.
𝛥𝜑 𝛥𝜑 Nesse período de 1/30 s, a roda pode ter
𝜔= → 𝛥𝑡 =
𝛥𝑡 𝜔 completado 1 volta, 2 voltas, 3 voltas, enfim,
𝛥𝑠 𝛥𝜑 um número inteiro n de voltas. Portanto, a
=
𝑣 𝜔 frequência f de rotação da roda (número de
𝜔𝛥𝑠 voltas completadas por segundo) é dada por:
𝑣= 𝑛
𝛥𝜑 𝑓= = 𝑛. 30 𝐻𝑧
Como ω = 2πf = 2 π · 50 = 100 π rad/s, 1
(30)
temos:
v = 800 m/s A roda completa 30n voltas em 1 s, sendo n
19 Resolução do prof. Laerte Pereira: = 1, 2, 3... .

b) 𝜔𝑚í𝑛 = 𝜋2𝑓𝑚í𝑛

𝑓𝑚í𝑛 é 𝑜 𝑐𝑎𝑠𝑜 𝑑𝑒 𝑛 = 1
𝜔𝑚í𝑛 = 180 𝑟𝑎𝑑/𝑠

c) 𝑣𝑚í𝑛 = 𝜔𝑚í𝑛 𝑟 = 180 · 0,3 = 54 m/s

22 Resolução do prof. Laerte Pereira:


O comprimento do aro aumentado é:

C + 1 = 2π (R + x)

temos:
𝐿√2
𝑣1 𝑟1
= = 2 2π R + 1 = 2π R + 2π x
𝑣2 𝑟2 𝐿
2 x = 16 cm
Logo
𝑣1 O gato é o maior animal capaz de passar
= √2
𝑣2 pela folga x.

20. Resolução do Prof. Laerte Pereira: 23 Resolução do Prof. Laerte Pereira:


Seja n o número inteiro de voltas
completadas por A e B a partir do instante em
que estavam emparelhados. Então, como o
tempo que passou para A (ΔtA) é igual ao que
passou para B (ΔtB), temos: 𝑀. 𝑅 2 𝑀. 𝑅 2
. 2𝜋𝑓 = . 2𝜋𝑓 ′
2 2
ΔtA = ΔtB
𝑓 ′ = 4𝑓
1
(𝑛)(1 𝑚𝑖𝑛 40 𝑠) = (𝑛 + ) (1 𝑚𝑖𝑛 36 𝑠)
4 𝑓 ′ = 2𝐻𝑧
Resolvendo
10 min 𝑛
𝑓′ =
∆𝑡
21 Resolução do Prof. Laerte Pereira: n = 30 voltas
37) A marca parece parada quando é
filmada na frequência de 30 Hz.
24 Resolução do Prof. Laerte Pereira: 2𝐿
𝛥𝑡 =
𝑣
𝛥𝑡 = 10𝑠
Sabemos que as partículas possuíam
velocidades de módulo constante, como se
movimentaram durante 10 𝑠, podemos
afirmar:
𝑑 = 𝑣 ∙ Δ𝑡
v. cos 𝜃 = 𝜔𝑟 A trajetória das partículas é uma espiral, mas
𝜔𝑟 isso não é relevante, tudo que precisamos
𝑣= saber é que a velocidade tangencial da
cos 𝜃
Mas partícula é constante e, portanto, a
𝑑 expressão de movimento uniforme se aplica.
cos 𝜃 = Substituindo os valores do enunciado na
𝑟
Então expressão acima, obtemos:
𝜔. 𝑑 𝑑 = 20 𝑚
𝑣=
(cos 𝜃)2
27Resolução do Professor Laerte Pereira:

25 Resolução do Prof. Laerte Pereira:

𝐴𝐶 𝐵𝐶 𝑙
= =
sin[90° °
− (𝛽 − 𝜃)] sin 90 − 𝜃 sin 𝛽
26 Resolução do Prof. Laerte Pereira:
A figura abaixo mostra a configuração do Temos que
movimento entre dois vértices do hexágono sin[90° − (𝛽 − 𝜃)] = cos(𝛽 − 𝜃) e
em um instante qualquer: sin(90° − 𝜃) = cos(𝜃)
𝑅𝐴 = 𝐴𝐶
𝑅𝐵 = 𝐵𝐶
Então
𝑙. cos(𝛽 − 𝜃)
𝑅𝐴 =
sin 𝛽
𝑙. cos(𝜃)
𝑅𝐵 =
sin 𝛽
𝑣𝐴 𝑣𝐵
𝜔= =
Note que a velocidade relativa entre dois 𝑅𝐴 𝑅𝐵
vértices se mantém a mesma:
𝑣 𝑣𝐴
𝑣𝑟𝑒𝑙 = 𝑣 − 𝑣. cos 60° = 𝑣𝐵 =
2 cos(𝛽 − 𝜃)
𝛥𝑑𝑟𝑒𝑙
𝑣𝑟𝑒𝑙 =
𝛥𝑡 28 Resolução do Prof. Laerte Pereira:
𝑣 𝐿
=
2 𝛥𝑡
Pedem-nos o número de rotações da
engrenagem A, se é sabido que a
engrenagem (1) executa n1 rotações,
enquanto (2) realiza n₂ rotações no mesmo
tempo t, com esses dados podemos Devido a rotação
determinar a velocidade angular de cada 𝑣𝑇 = 𝜔. 𝑟 = 5𝑚/𝑠
engrenagem, onde: Velocidade de deslizamento
2𝜋. 𝑛 𝑣𝑑 = 5𝑚/𝑠
𝜔 = A velocidade total
𝑡
Determinando o raio RA 𝑣𝐵 = 5. √2 𝑚/𝑠

2.RA = R-r 30.Resolução do Prof. Laerte Pereira:


𝑎𝐵
𝛾𝐵 = 𝛾𝐴 = = 40 𝑟𝑎𝑑/𝑠 2
𝑅−𝑟 𝑅𝐵
𝑅𝐴 =
2
então a velocidade tangencial com que a 𝑎𝐴 = 𝛾𝐴 𝑅𝐴 = 8𝑚/𝑠 2
engrenagem A gira é:
vA = v1 – v2 𝑎𝐴 = 𝑎𝑐

ωA RA = ω1 R – ω2 R 𝑎𝑐
𝛾𝑐 = = 16𝑟𝑎𝑑/𝑠 2
𝑅𝑐
𝑛𝐴 (𝑅 − 𝑟) 𝑛1 𝑛2
= 𝑅− 𝑟 𝜔𝑐 = 𝜔0𝑐 + 𝛾𝑐 𝑡
𝑡 2 𝑡 𝑡

2. (𝑛1 𝑅 − 𝑛2 𝑟) 𝜔𝑐 = 16𝑡
𝑛𝐴 = 31.
(𝑅 − 𝑟)

29
Resolução do Professor Laerte Pereira:
32. com movimento rotacional em relação ao ele,
𝑣(𝑅 + 𝑟) sendo a velocidade linear do ponto P em
𝑣𝐴 = relação ao O:
𝑟

𝑣√𝑅 2 + 𝑟 2 𝑣𝑃𝑂 = Ωℎ. sec 𝜃


𝑣𝐵 =
𝑟
Mas, se nos colocarmos no centro da base
𝑣. (𝑅 − 𝑟) do cone (A), veríamos que o ponto P teria
𝑣𝑐 = velocidade linear
𝑟

𝑣√𝑅 2 + 𝑟 2 𝑣𝑃𝐴 = 𝜔ℎ. tan 𝜃


𝑣𝐷 =
𝑟
Mas, como já dissemos, o ponto P está em
33. repouso em relação ao piso. Então: vp = 0, o
que nos leva concluir que:

𝜔ℎ. tan 𝜃 = Ωℎ. sec 𝜃

𝜔 = Ω. csc 𝜃

35.

𝛼
𝑣𝑃 = 2𝑣. cos
2
𝑣𝑄 = 2𝑣. cos 𝛽

34. Resolução do prof. Laerte Pereira:

3𝑅 3
𝑥= → cos 𝜃 = → 𝜃 = 53°
5 5

𝑣1 . sin 𝜃 = 𝑣

𝑣 5𝑣
𝑣1 = =
sin 𝜃 4
Este problema nos sugere vincular dois 36. Resolução do Prof. Laerte Pereira:
movimentos rotacionais. Para isso,
escolheremos convenientemente um ponto
que seja comum a ambos os movimentos e
que também relacione as características
destes movimentos. Colocando-nos no eixo
vertical, veríamos que a geratriz OP está em
repouso em relação ao chão, e se moveria
𝑣0 . 𝑑𝑡 = (𝑙0 − 𝑅𝜃). 𝑑𝜃
logo a fAPA = 60 rpm
𝑙0
𝑡 𝑅

∫ 𝑣0 . 𝑑𝑡 = ∫(𝑙0 − 𝑅𝜃). 𝑑𝜃
0 0

𝑙0
𝑅. 𝜃 2 𝑅 𝑙02
𝑣0 . 𝑡 = [𝑙0 𝜃 − ] =
2 0 2𝑅

𝑙02
𝑡=
2𝑅𝑣0

37. Resolução:
a)
900
900 𝑟𝑝𝑚 = = 15 𝑣𝑜𝑙𝑡𝑎𝑠/𝑠
60

Fazendo a regra de 3:
1volta ---3600
15 voltas---x

X = 54000

15 volt. ----1s ----24 flash ----54000


1 flash----x

5400°
𝑥= = 225° /1𝑓𝑙𝑎𝑠ℎ
24 𝑓𝑙𝑎𝑠ℎ

embora a roda gire 225 graus horários a cada


flash, cérebro humano não perceberá todo
esse giro a cada flash.
Para entender, note que o ângulo 225 graus
pode ser escrito de duas formas:
225 = 180 + 45 ou 225 = 240 -15
O cérebro humano prefere enxergar sempre
o menor giro. Sendo 15° < 45°, o cérebro
interpretará que essa roda está girando 15°
anti-horários a cada flash.

b) Mas girar 15° anti-horários a cada flash


significa girar com que frequência aparente?
Ora, lembrando que a lâmpada pisca num
ritmo de 24 flashes/seg, temos:

1volta----24 flash ---- 3600


fAPA ------- 150
Capítulo 6 - Movimento relativo e Consideremos um barco navegando em um
composição de Movimento rio, conforme ilustra a figura a cima. Sejam
O movimento relativo consiste em mudamos vrel a velocidade do barco em relação às
o referencial do movimento. águas e varr a velocidade das águas em
Imaginando três referenciais A, B e C relação às margens.
conforme a figura abaixo. O barco tem, portanto, dois movimentos
parciais: o movimento relativo, provocado
pelo motor em relação às águas, com
velocidade vrel, e o movimento de
arrastamento, provocado pela correnteza,
com velocidade varr.
Podemos concluir que estamos no Fazendo a composição desses movimentos,
referencial da terra, e que é o mesmo o barco apresentará em relação às margens
referencial do corpo C. Analisando o um movimento resultante com velocidade
referencial do corpo A (e usando o conceito vres, que é dada pela soma vetorial de vrel
que no referencial do corpo A o próprio corpo com varr.
A está em repouso). Com isso em mente
vamos somar a todos os corpos o vetor
oposto a velocidade de A (−𝑣 ⃗⃗⃗⃗⃗).
𝐴

Quando chamamos as velocidades Note que o movimento provocado pelo motor


de𝑣⃗⃗⃗⃗⃗, 𝑣𝐶 já entendemos que essas do barco (movimento relativo) é
𝐴 ⃗⃗⃗⃗⃗
𝑣𝐵 𝑒 ⃗⃗⃗⃗⃗
velocidades são em relação a um referencial o que a embarcação teria em relação às
fixo na terra, geralmente chamamos de solo. margens se no rio não houvesse correnteza
Podemos rescrever as velocidades no (se as águas estivessem em repouso).
referencial de A como:
Exemplo: Um rio de margens retilíneas e
𝑣𝐵𝐴 = ⃗⃗⃗⃗⃗
⃗⃗⃗⃗⃗⃗⃗ 𝑣𝐵 − ⃗⃗⃗⃗⃗
𝑣𝐴
largura constante igual a 5,0 km tem
águas que correm paralelamente às
𝑣𝐶𝐴 = ⃗⃗⃗⃗
⃗⃗⃗⃗⃗⃗⃗ 𝑣𝑐 − ⃗⃗⃗⃗⃗
𝑣𝐴
margens, com velocidade de intensidade
30 km/h. Um barco, cujo motor lhe imprime
𝑣𝐴𝐴 = 0
⃗⃗⃗⃗⃗⃗⃗ velocidade de intensidade sempre igual a
50 km/h em relação às águas, faz a
Movimento em rios travessia do rio.
Podemos estudar três tipos de movimentos
em um rio, que são: a) Qual o mínimo intervalo de tempo
Sobe o rio; possível para que o barco atravesse o rio?
Desce o rio;
Travessia do rio. b) Na condição de atravessar o rio no
Antes de tudo vamos definir alguns termos. intervalo de tempo mínimo, que distância
Velocidade relativa, de arrastamento o barco percorre paralelamente às
e resultante margens?

c) Qual o intervalo de tempo necessário


para que o barco atravesse o rio
percorrendo a menor distância possível?
Resolução:
a) A travessia do rio é feita no menor Exemplo: Um automóvel desenvolve, em
intervalo de tempo possível quando a uma estrada plana e horizontal,
velocidade do barco em relação às águas movimento retilíneo e uniforme com
é mantida perpendicular à velocidade da velocidade de módulo v. Supondo que
correnteza. suas rodas rolem sem escorregar, calcule,
(O movimento relativo é independente do em relação ao plano de rolamento, os
movimento de arrastamento.) módulos das velocidades instantâneas
dos pontos A, B, C, D e O, indicados na
figura a seguir.

Travessia em tempo mínimo


𝐿
𝑣𝑟𝑒𝑙 =
𝑡 Neste exercício, a roda não pode ser
5
𝑡= = 0,1 ℎ = 6𝑚𝑖𝑛 considerada uma partícula, mas um corpo
50 extenso. E o movimento em relação ao
b) A distância D que o barco percorre referencial inercial (no nosso caso o solo)
paralelamente às margens, arrastado de um ponto qualquer pertencente a um
pelas águas do rio, é calculada por: corpo extenso pode ser estudado como a
𝐷
𝑣𝑎𝑟𝑟 = composição de dois movimentos
𝑡 simultâneos:
𝐷
30 = = 3𝑘𝑚 • movimento do ponto em relação ao seu
0,1 centro de massa (por enquanto apenas
c) A travessia do rio é feita com o barco nos referiremos ao centro geométrico) –
percorrendo a menor distância possível também é chamado de movimento de
entre as margens quando sua velocidade rotação em torno do centro de massa.
em relação ao solo (velocidade resultante) • movimento do centro de massa em
é mantida perpendicular à velocidade da relação ao referencial inercial – também é
correnteza. chamado de translação do centro de
massa.
Em termos de velocidade, podemos
escrever:
𝑣𝑝𝑠 = 𝑣
⃗⃗⃗⃗⃗⃗ ⃗⃗⃗⃗⃗⃗⃗
𝑝𝑐 + ⃗⃗⃗⃗⃗⃗
𝑣𝑐𝑠
em que:
vPS é a velocidade do ponto P qualquer em
Travessia em distância mínima relação ao solo.
I. Pelo Teorema de Pitágoras: vPC é a velocidade do ponto P qualquer em
502 = 𝑣𝑟𝑒𝑠
2
+ 302 relação ao centro da roda, o ponto O.
vCS é a velocidade do centro da roda em
vres = 40 km/h relação ao solo.
5 No movimento de rotação, todos os pontos
∆𝑡 ′ = = 0,125ℎ giram com a mesma velocidade angular ω
40
em torno do centro.
Se o raio da roda é igual a R, então os velocidade nula. E isso acontece quando
pontos da periferia da roda, devido tivermos v = ωR.
exclusivamente à rotação, possuem
velocidades vetoriais (𝑣 𝑝𝑐 de módulos
⃗⃗⃗⃗⃗⃗⃗)
iguais a ωR:

E os módulos das velocidades vetoriais de


cada ponto, em relação ao referencial
inercial, são dados por:
vA = 2v
No movimento de translação, o centro da vB = v√2
roda possui velocidade de translação v e, vC = 0
portanto, devido exclusivamente à vD = v√2
translação, todos os pontos possuem essa vO = v
mesma velocidade vetorial (𝑣 ⃗⃗⃗⃗⃗⃗)
𝑐𝑠 de
módulo v:
Exercícios

1. (Laerte Pereira) Depois de 2 h de haver


cruzado com algumas jangadas que
flutuavam arrastadas pelo rio, uma lancha a
motor decide retornar rio abaixo. Em quanto
tempo o barco alcançará as jangadas?

2. (Laerte Pereira) Quando um pequeno


barco passa por baixo de uma ponte muito
A composição desses dois movimentos larga, leva 10 s se estiver navegando a favor
simultâneos, dada pela soma vetorial para da correnteza, caso contrário, leva 15 s. A
cada ponto, pode ser representada pela correnteza do rio tem uma velocidade de 1
figura a seguir: m/s. Encontre a largura da ponte.

3. (Laerte Pereira) A figura mostra um


hemisfério e uma haste apoiada. O
hemisfério está se movendo para a direita
com uma velocidade uniforme v2 e a
extremidade da haste que está em contato
com o solo está se movendo na direção
esquerda com uma velocidade v1. Encontre
a taxa na qual o ângulo θ está variando em
termos de v₁, v₂, R e θ.
No caso de a roda rolar sem escorregar, o
ponto da roda que estiver em contato com
o solo não poderá possuir nenhuma
velocidade em relação a este, ou seja,
4. (Laerte Pereira) Uma aeronave está soprando do Oeste. Qual é a velocidade do
descendo para pousar em um aeroporto na vento (da fumaça)?
manhã. A aeronave está aterrissando para o
leste, de modo que o piloto tem o sol em seus 8. A correia transportadora mostrada está se
olhos. A aeronave tem uma velocidade v e movendo com velocidade constante c e nela
está descendo a um ângulo α com a está um dispositivo (A) que lança n bolas por
horizontal, e o sol está em um ângulo ẞ segundo, que grudam na correia. Um
acima do horizonte. Encontre a velocidade contador (B) que também está na fita,
com que a sombra da aeronave se move registra apenas as bolas que passam por
sobre a terra. baixo. Quantas bolas o contador registra por
segundo? (V e u são menores que c).
5. Uma lancha dotada de um motor potente,
encontra-se parada em uma das margens de
um rio de margens paralelas. A velocidade
da correnteza em relação às margens é de 4
m/s e a partir de 𝑡 = 0, a lancha começa a
movimentar-se com MRUV em relação à
água com aceleração escalar de 2,0 m/s², de 𝑐−𝑢
a)(𝑐−𝑣 ) 𝑛
tal forma que o móvel fique perpendicular à
correnteza.
𝑐−𝑣
b) (𝑐−𝑢) 𝑛
a) determine a trajetória da lancha para um
observador fixo na Terra. 𝑐−2𝑣
c) (𝑐−2𝑢) 𝑛
b) determine a velocidade da lancha em
relação às margens em 𝑡 = 1,0 𝑠 (suponha d) (𝑐 − 𝑣 − 𝑢)/(𝑐 − 𝑣)𝑛
que a lancha ainda não atravessou o rio).
𝑢−𝑣 𝑛
e)( )
𝑐
6. (ITA) Um nadador, que pode desenvolver
uma velocidade de 0,900𝑚/𝑠 na água
9. Duas barras se cruzam formando entre si
parada, atravessa o rio de largura 𝐷 metros,
um ângulo 2α e se movem com velocidades
cuja correnteza tem velocidade de
iguais a v e perpendicularmente a si
1,08𝑘𝑚/ℎ. Nadando em linha reta ele quer
mesmas. A velocidade do ponto de
alcançar um ponto da outra margem situada
cruzamento das barras é igual a:
√3
𝐷 metros abaixo do ponto de partida. Para
3
que isso ocorra, sua velocidade em relação
ao rio deve formar com a correnteza o
ângulo:
√3
a) 𝑎𝑟𝑐𝑠𝑒𝑛 (√33 + 1).
12
√3
b)𝑎𝑟𝑐𝑠𝑒𝑛 2 .
c) zero grau.
√3
d) 𝑎𝑟𝑐𝑠𝑒𝑛 12.
e) o problema não tem solução.
a) v.sen2α
7. (H.Moysés) Um trem viaja para o norte a b) v/sen2α
120 km/h. A fumaça da locomotiva forma c) v.cos2α
uma trilha que se estende numa direção 14° d) v/cosα
ao E (leste) da direção sul, com o vento e) v/senα
10. Uma estrutura articulada consiste de três
losangos de lados com razão 3:2:1. O vértice d) 𝐻. tan 2𝜃
A3 se move na direção horizontal com uma
velocidade v. e)
𝐻.cot 𝜃
90° +𝜃
2.sin( )
2

13. (Renato Brito) Um barco a motor, que ia


subindo um rio, encontrou uma balsa que se
movia no sentido da correnteza. Após uma
hora do encontro, o motor do barco parou. O
conserto do motor durou 30 min e durante
Determine as velocidades dos vértices A1, A2 esse tempo o barco moveu-se livremente no
e B1 no instante em que os ângulos da sentido da corrente. Depois do conserto, o
estrutura são 90º. barco começou a se mover na direção da
corrente, seguindo rio abaixo com a mesma
11. (Peruano) Uma barra de comprimento velocidade relativa à água e encontrou a
R/2 movendo-se com velocidade constante balsa a uma distância de 7,5 km em relação
através do trilho perfazendo uma trajetória ao primeiro encontro. Determine a
em forma de circunferência. velocidade da correnteza.
a) 5 km/h
b) 4 km/h
c) 3 km/h
d) 2 km/h
e) 6 km/h

14.(Renato Brito) Um homem em uma


lancha deve sair do ponto A e chegar ao
ponto B, que se encontra na margem oposta
do rio. A distância BC é igual a a. A largura
Determinar a velocidade do ponto (1) em do rio AC é igual a b. Com que velocidade
relação ao ponto (2). mínima u, relativa à água, deve se mover a
lancha para chegar ao ponto B? A velocidade
12. No instante mostrado o sistema é da correnteza é Vo.
abandonado, de tal maneira que a esfera
desce. Determine a distância mínima de
separação entre o bloco e a esfera.

15. (Renato Brito) Numa avenida retilínea


de largura e, trafegam carros com velocidade
constante v. A largura dos carros vale b e a
𝐻.sec 𝜃 distância livre entre dois carros consecutivos
a) sempre vale a. Um pedestre, movendo-se
2
em trajetória retilínea, deseja atravessar
b)
𝐻.cos 𝜃 essa avenida com a menor velocidade
2
constante possível, sem ser atropelado. Qual
𝐻.sen(45° −𝜃)
a menor velocidade possível para essa
c) travessia?
sin 2𝜃
b) 30.
c) 20.
d) 15.
e) 10.

18. Um trem dotado de janelas laterais


retangulares de dimensões 80 cm (base)x60
16. O olho C de um furacão desloca-se em cm (altura) viaja ao longo de uma ferrovia
linha reta com velocidade de intensidade retilínea e horizontal com velocidade
vC=150 km/h em relação à Terra na direção constante de intensidade 40 km/h. Ao
Sul-Norte, dirigindo-se para o Norte. A mesmo tempo, cai uma chuva vertical (chuva
massa de nuvens desse ciclone tropical, sem vento), de modo que as gotas
contida em um plano horizontal paralelo ao apresentam, em relação ao solo, velocidade
solo, realiza uma rotação uniforme no constante de intensidade v. Sabendo que o
sentido horário em torno de C abrangendo trajeto das gotas de chuva observado das
uma região praticamente circular de raio R janelas laterais do trem tem a direção da
igual a 100 km, conforme ilustra a figura, em diagonal dessas janelas, determine:
que O1 e O2 são dois observadores em a) o valor de v;
repouso em relação à superfície terrestre. b) a intensidade da velocidade das gotas de
chuva em relação a um observador no trem.

19. Uma partícula se desloca sobre o plano


cartesiano 0xy tal que suas coordenadas de
posição, x e y, variam em função do tempo t,
conforme as expressões:

x =1,0t2 + 1,0t (SI) e y = 1,0t3 + 5,0 (SI)

Sabendo-se que em t0 = 0 a partícula se


encontra em um ponto A e que no instante
t1=2,0 s ela se encontra em um ponto B,
Sabendo que a velocidade angular da massa pede-se determinar:
de nuvens é constante e igual a 0,50 rad/h,
responda: a) o seno do ângulo u formado entre o
a) Qual a intensidade da velocidade dos deslocamento vetorial da partícula de A até
ventos medida por O1? B e o eixo 0x;
b) Qual a intensidade da velocidade dos
ventos medida por O2? b) a intensidade da velocidade vetorial média
c) De que lado (Leste ou Oeste) o furacão da partícula no trânsito de A até B.
tem maior poder de destruição?
20. A figura representa dois carros, A e B, em
17. Sob a ação de um vento horizontal com um instante t0 = 0. O referencial 0xy adotado
velocidade de intensidade v=15 m/s, gotas está contido no solo, suposto plano e
de chuva caem formando um ângulo de 30° horizontal. O carro A se desloca com
em relação à vertical. A velocidade de um velocidade de módulo constante de 72 km/h
vento horizontal capaz de fazer com que em uma curva circular de centro C e raio
essas mesmas gotas de chuva caiam R=50 m; o carro B tem uma velocidade inicial
formando um ângulo de 60° em relação à de módulo 54 km/h e se desloca em uma
vertical deve ter intensidade, em m/s, igual a: trajetória reta.
a) 45.
Para evitar a colisão, o motorista do carro B d)
2
𝑣𝐵 +𝑣𝑐2
começa a frear, no instante t0 = 0, com uma 𝑣𝐵
aceleração de módulo 4,0 m/s2. 2
𝑣𝐵 −𝑣𝑐2
e) 𝑣𝐵

22. (H. Moysés) A distância entre as cidades


A e B é . Um avião faz uma viagem de ida e
volta entre A e B voando em linha reta, com
velocidade V em relação ao ar. Calcule o
tempo total de voo, se o vento sopra com
velocidade 𝑣, numa direção que forma um
ângulo θ com a direção AB.
Pede-se determinar:
a) o módulo da aceleração de A em relação 23. (IME) Dois observadores em movimento
ao solo; acompanham o deslocamento de uma
partícula no plano. O observador 1,
b) o módulo da velocidade de A em relação considerando estar no centro do seu sistema
a B no instante t0 = 0; de coordenadas, verifica que a partícula
descreve um movimento dado pelas
c) o módulo de aceleração de A em relação equações 𝑥1(𝑡) = 3cos (𝑡) e 𝑦1(𝑡) = 4sen (𝑡),
a B no instante t0 = 0. sendo 𝑡 a variável tempo. O observador 2,
considerando estar no centro de seu sistema
21. Um barco vai de Manaus até Urucu de coordenadas, equaciona o movimento da
descendo um rio e, em seguida, retorna à partícula como 𝑥2(𝑡) = 5cos (𝑡) e 𝑦2(𝑡) = 5sen
cidade de partida, conforme esquematizado (𝑡). O observador 1 descreveria o movimento
na figura. do observador 2 por meio da equação:
Observações:
- Os eixos 𝑥1 e 𝑥2 são paralelos e possuem
o mesmo sentido; e
- Os eixos 𝑦1 e 𝑦2 são paralelos e possuem
o mesmo sentido.
a) 9𝑥2 + 16𝑦2 = 25.
A velocidade da correnteza é constante e 𝑥2 𝑦2
tem módulo vC em relação às margens. b) + 16 = 25
9
A velocidade do barco em relação à água é
constante e tem módulo vB. c) 4𝑥2+𝑦2=1.
Desconsiderando-se o tempo gasto na
manobra para voltar, a velocidade escalar d)
𝑥2
+ 𝑦2 = 1
média do barco, em relação às margens, no 4
trajeto total de ida e volta tem módulo dado
por: e) 4𝑥2+𝑦2=4.

𝑣𝐵 +𝑣𝑐 24.(ITA) Dois barcos, 1 e 2, partem


a) 2 simultaneamente de um ponto 𝐴 da margem
𝑣𝐵 −𝑣𝑐
de um rio, conforme a figura, com
b) velocidades constantes em relação à água
2
respectivamente iguais 𝑉1 e 𝑉2. O barco 1
c) √𝑣𝐵 𝑣𝑐 vai diretamente até o ponto 𝐵 da mesma
margem, rio abaixo, e volta a 𝐴. O barco 2
vai diretamente até o ponto 𝐶 da outra 26.(INDIANO) Um barco se move em relação
margem e volta a 𝐴. Os tempos de ida e volta à água com uma velocidade que é n = 2,0
para ambos os barcos são iguais. As vezes menor que a velocidade do fluxo do
distâncias 𝐴𝐶 e 𝐴𝐵 são iguais entre si e a rio. Em que ângulo em relação à direção do
velocidade da correnteza é constante e fluxo o barco deve se mover para minimizar
apresenta módulo 𝑉, em relação às margens a deriva?
do rio. Sabendo que a razão entre o tempo
de descida de 𝐴 para 𝐵 e o tempo de subida
de 𝐵 para 𝐴 é 𝑟, os módulos 𝑉1 e 𝑉2, valem
respectivamente:

27. A velocidade da correnteza de um rio


cresce proporcionalmente à distância da
2
margem e atinge seu máximo 𝑣0 no meio.
1−𝑟 2
a) 𝑉1 = 𝑉 (1+𝑟
1−𝑟
1+𝑟
) 𝑒 𝑉2 = 𝑉√( )
1−𝑟
−1+(
1+𝑟
) . Perto das margens, a velocidade é zero. Um
barco está se movendo ao longo do rio de tal
2 1−𝑟 2 maneira que é sempre perpendicular à
b) 𝑉1 = 𝑉 (1+𝑟 1+𝑟
) 𝑒 𝑉2 = 𝑉√( ) +1+( ) .
1−𝑟 1−𝑟 1+𝑟 correnteza e a velocidade do barco em águas
calmas é 𝑢. Calcule a distância pela qual o
2 1−𝑟 2
c) 𝑉1 = 𝑉 (1+𝑟 1+𝑟
) 𝑒 𝑉2 = 𝑉 √( ) −1−( ) . barco que cruza o rio será levado pela
1−𝑟 1−𝑟 1+𝑟
correnteza se a largura do rio for b.
2 1−𝑟 2
Determine também a trajetória do caminho
d) 𝑉1 = 2𝑉 (1−𝑟
1+𝑟
1+𝑟
) 𝑒 𝑉2 = 𝑉√( )
1−𝑟
−1+(
1+𝑟
) . do barco.
2 1−𝑟 2
e) 𝑉1 = 2𝑉 (1+𝑟
1−𝑟
1+𝑟
) 𝑒 𝑉2 = 𝑉√( )
1−𝑟
−1+(
1+𝑟
) .

25. Um pedaço longo de papel tem 10 cm de


largura e se move uniformemente ao longo
de seu comprimento com uma velocidade de
2 cm/s. Uma formiga começa a se mover
sobre o papel a partir do ponto A e se move
uniformemente em relação ao papel. Uma
aranha estava localizada exatamente oposta 28. Um nadador mergulha no rio
à formiga, do lado de fora do papel, no ponto perpendicularmente à correnteza com uma
B, no instante em que a formiga começou a velocidade inicial de 10 m/s. Se a sua
se mover sobre o papel. A aranha, sem se velocidade começa a diminuir por exaustão
mover, conseguiu agarrar a formiga 5 física de acordo com V = (10 - 2t) m/s, onde
segundos depois que ela (a formiga) "t" é o tempo medido a partir do instante do
começou a se mover. Encontre a velocidade lançamento, e assumindo que a velocidade
da formiga em relação ao papel. da correnteza do rio é constante e igual a 4
m/s, determine a velocidade resultante do
nadador quando ele atinge a margem
oposta.
Gabarito e/ou resoluções

1. Resolução do prof. Laerte Pereira:


vR = rapidez das águas do rio
vL = rapidez da Lancha

29. Encontre o tempo que um avião leva para


voar em torno de um quadrado de lado a com
o vento soprando a uma velocidade u, nos
dois casos, (a) Se a direção do vento for ao
longo de um lado do quadrado; (b) a direção
do vento é ao longo de uma das diagonais do Velocidade total (v1) quando o barco navega
quadrado. 2 h rio acima.

V1 = (vL – vR)

D1 = v1.t1

D1 = (vL – vR).(2)

Velocidade total (v2) quando as jangadas são


rebocadas para (2h + t3); 2h enquanto o
barco navega rio acima e t3 enquanto navega
rio abaixo.

D2 = vR.(2 + t3)

D3 = (vL + vR).t3

D1 + D2 = D3

t3 = 2h

2. Resolução do prof. Laerte Pereira:


𝑣𝑏 = 𝑣𝑒𝑙. 𝑑𝑜 𝑏𝑎𝑟𝑐𝑜;
𝑣𝑎 = 𝑣𝑒𝑙. 𝑑𝑎𝑠 á𝑔𝑢𝑎𝑠
A favor da correnteza:
𝑙
10 = 𝑒𝑞. 1
𝑣𝑏 + 𝑣𝑎

𝑙
15 = 𝑒𝑞2
𝑣𝑏 − 𝑣𝑎
Dividindo as eq.

2 𝑣𝑏 − 𝑣𝑎
=
3 𝑣𝑏 + 𝑣𝑎

5va = vb
𝑥. sin(𝛼 + 𝛽)
vA = 1m/s 𝑦=
sin 𝛽
Derivando
vb = 5m/s
𝑣. sin(𝛼 + 𝛽)
da eq.2 𝑣𝑠 =
sin 𝛽
l = 60m
5 Resolução:
3. Resolução do Prof. Laerte Pereira: a) Inicialmente, vamos adotar um sistema de
coordenadas cartesiana, onde a origem
Aqui x é a separação entre o centro do coincida com a posição inicial da lancha na
hemisfério e o extremo da haste. A taxa de margem do rio. Além disso, colocamos o eixo
variação de x pode ser tomada como a x orientado no sentido da correnteza,
velocidade relativa da extremidade da haste conforme figura abaixo:
e do centro do hemisfério, ou seja, (v1 + v₂).
Precisamos encontrar a taxa de variação de
θ e a taxa de variação de x que conhecemos,
então temos que desenvolver uma relação
entre x e θ, que é dada como:

𝑥 = R. cosec 𝜃
Derivando

𝑑𝑥 𝑑𝜃 Considerando um observador nas margens


= −𝑅 csc 𝜃 . cot 𝜃
𝑑𝑡 𝑑𝑡 do rio, isto é, ele está fixo na Terra, para ele
a lancha é levada rio abaixo, com módulo de
𝑑𝜃 (𝑣1 + 𝑣2 ) sin 𝜃 2 velocidade 𝑣𝐴/𝑀, em um movimento
=
𝑑𝑡 cos 𝜃 uniforme. Para este tipo de movimento, a
função horária do espaço é dada por:

4. Resolução do prof. Laerte Pereira: 𝑠 = 𝑠0 + 𝑣𝐴/𝑀. 𝑡

Logo, para o nosso caso, temos que:

𝑥 = 4. 𝑡 eq.(1)

Por outro lado, enquanto a lancha move-se


rio abaixo (na direção de x), ela também se
desloca na direção de y mas realizando um
MUV nesta orientação. Logo, a posição y
pode ser dada por:
𝑑𝑥
𝑣=
𝑑𝑡 𝑦 = 𝑡2 eq.(2)
𝑑𝑦
𝑣𝑠 = − Isolando o tempo em (1) e substituindo em
𝑑𝑡 (2), podemos encontrar a equação da
𝑥 𝑦 trajetória:
= 𝑥2
sin 𝛽 sin(𝛼 + 𝛽) 𝑦=
16
Este resultado, mostra que a equação da
trajetória da lancha é uma parábola e neste
intervalo de tempo ela irá descrever um arco
de parábola.

𝐴 é o ponto de partida do nadador, 𝐷 é a sua


b) Em 𝑡=1,0 𝑠, na direção x temos que reflexão na margem oposta e 𝐸 é o ponto de
𝑣𝑥=4,0 𝑚/𝑠 e na direção y, a lancha realiza chegada. Convertendo a velocidade da
um MUV, portanto: correnteza para 𝑚/𝑠 obtemos 0,3𝑚/𝑠, assim
temos a relação:
𝑣𝑦=𝑣0𝑦+𝑎𝑦.𝑡
𝑉𝑛𝑎𝑑𝑎𝑑𝑜𝑟 = 3 ∙ 𝑉𝑟𝑖𝑜
𝑣𝑦=2,0 𝑚/𝑠
Pelas distâncias dadas no enunciado
Usando o teorema de Pitágoras conseguimos determinar o ângulo 𝛼:

𝑣=2√5 𝑚/𝑠 √3𝐷


𝐷𝐸 √3
tan 𝛼 = = 3 =
𝐴𝐷 𝐷 3

𝛼 = 30°
A partir disso, podemos concluir que:
𝐴𝐸̂ 𝐷 = 60°
Note que o nadador deve ter sua velocidade
resultante na direção do segmento de reta
𝐴𝐸:

6. Resolução:

Considere a situação em que o nadador tem


a velocidade inclinada no sentido da
velocidade da correnteza (se for o contrário
você encontrará isso no valor de 𝛽):
Onde 𝛾=𝐴𝐸̂ 𝐷. Usando a lei dos senos
obtemos:
𝑣𝑛𝑎𝑑𝑎𝑑𝑜𝑟 𝑣𝑟𝑖𝑜 Onde D é o diâmetro de uma esferinha.
=
sin 𝛾 sin 𝛽 𝑐−𝑉
𝐷=
𝑛
𝑣𝑟𝑖𝑜 √3 No ref. De A o contador B:
sin 𝛽 = sin 60° =
𝑣𝑛𝑎𝑑𝑎𝑑𝑜𝑟 6

Perceba que a questão pede o menor


ângulo entre a velocidade do nadador
(𝑉𝑛𝑎𝑑𝑎𝑑𝑜𝑟) e a margem do rio (𝛽+60°):

√3
sin(𝛽 + 60° ) = (1 + √33)
12 𝐿′ = (𝑐 − 𝑉)𝑡 − (𝑢 − 𝑉)𝑡 = 𝑛′ . 𝐷
7 Resolução do prof. Laerte Pereira:
𝑐−𝑉
(𝑐 − 𝑢) = 𝑛′ ( )
𝑛
𝑐−𝑢
𝑛′ = ( ).𝑛
𝑐−𝑉

9. Resolução do prof. Laerte Pereira:


Vamos traçar as posições inicial (linha
sólida) e final (linha pontilhada) das barras:

𝑣⃗𝑣,𝑠 = 𝑣⃗𝑣,𝑇 + 𝑣⃗𝑇,𝑠

Na figura após um tempo (Δt), as barras


𝑣𝑣,𝑠 percorrem uma distância igual a,
tan 14° = = 0,25
120
d = v.Δt
𝑣𝑣,𝑠 = 29,9𝑘𝑚/ℎ
Também, no Δ ABC, ao mesmo tempo, a
8. Resolução do Prof. Laerte Pereira: distância percorrida pelo ponto de
intersecção das barras é,
No ref. De A:
1
𝑒 = (𝑥 2 + 𝑥 2 − 2𝑥 cos(180° − 2𝛼))2

e = 2x. cos 𝛼

Por outro lado, no triângulo direito BCD,


encontra-se:
𝐿 = 𝑛. 𝐷 = 𝑐 − 𝑉
𝑑
𝑥=
sin 2𝛼
𝑣. ∆𝑡
𝑒 = 2. cos 𝛼
sin 2𝛼
𝑒 𝑣
𝑢= =
∆𝑡 sin 𝛼

10. Resolução do prof. Laerte Pereira:


os comprimentos l1, l2 e l3 dos segmentos
A0A1, A0A2, e A0A3, durante o movimento:
l1 : l2 : l3 = 3 : 5 : 6. E, portanto,
1
sin 𝜃 =
𝑣 4
𝑣𝐴1 = 𝑣
2 𝑣𝐴/𝐵 = 2𝑣 sin 𝜃 =
2
5𝑣
𝑣𝐴2 = 12. Resolução do Prof. Laerte Pereira:
6
Consideremos agora a velocidade da união
central (A1 B2 A2 C2) no instante em que os No referencial de 1
ângulos da construção são iguais a 90º. No
referencial móvel com velocidade vA1 a
velocidade vA2 do ponto B2 está direcionada
nesse instante ao longo de B2A2. A
velocidade do ponto A2 está direcionada ao
longo da horizontal e é dada por:
𝑣
𝑣𝐴2 − 𝑣𝐴1 =
3
Como o fio é ideal
Da condição de extensibilidade da haste
B2A2 segue que:
V1 = V2
′ ′
𝜋 √2
𝑣𝐵2 = 𝑣𝐴2 . sin ( ) =
4 6 𝛽 = 𝛽′
Podemos encontrar a velocidade do ponto B2
com relação ao referencial estacionário Para o observador 1 a esfera descrave um
utilizando-se a lei dos cossenos: movimento retilíneo.

2 2 ′2
2√2
𝑣𝐵2 = 𝑣𝐴1 + 𝑣𝐵2 + 𝑣 𝑣2
2 𝐴1 𝐵2

√17
𝑣𝐵2 = 𝑣
6

11. Resolução do prof. Laerte Pereira:

No referencial de (2):
H. cot 𝜃 = 2𝑑𝑚í𝑛 (sin 𝛼)

H. cot 𝜃
→ 𝑑𝑚í𝑛 =
2. (sin 𝛼)

Também sabemos que

(𝛼 − 𝜃) + 𝛼 = 90°
Após permanecer parado no ponto B durante
90° + 𝜃 0,5 h (veja figura), o barco retorna com a
𝛼=( )
2 mesma velocidade V de antes (no referencial
da água), a fim de fazer percorrer na volta a
H. cot 𝜃 mesma distância da ida (AB = BA) no mesmo
𝑑𝑚í𝑛 = tempo gasto anteriormente (1 h) até alcançar
90° + 𝜃
2. sin ( 2 ) novamente a balsa (que permaneceu parada
em A o tempo todo).
13. Resolução: Assim, o tempo total para o barco perfazer a
A resolução dessa questão demonstra como distância AB + BA vale:
a escolha do referencial mais adequado Δ.t = 1 h (ida) + 0,5h (parado) + 1 h (volta) =
pode simplificar bastante a análise de um 2,5 h
problema de Física. Antes de iniciar a Conforme o enunciado afirma, porém, nesse
solução do problema, considere as seguintes intervalo de tempo de 2,5 h, a balsa percorre
informações, subentendidas no enunciado, uma distância de 7,5 km em relação às
mas que podem não ser conhecidas pelo margens, movendo-se arrastada pela
estudante: correnteza e, portanto, com a velocidade da
• navegar rio acima= navegar contra a correnteza (no referencial da Terra). Assim,
correnteza; qual a velocidade da correnteza (em relação
• navegar rio abaixo = navegar a favor da à Terra)?
correnteza;
• balsa = embarcação que move-se apenas 𝐷 7,5 𝑘𝑚
𝑣𝑐 = = = 3𝑘𝑚/ℎ
arrastada pela correnteza, não possui motor; ∆𝑡 2,5ℎ
• velocidade acusada pelo velocímetro do
barco a motor = velocidade do barco em 14. Resolução:
relação à água.
A chave é adotar o referencial na água, tomar
todas as velocidades em relação à água
inicialmente.
Nesse referencial, a balsa se encontra
parada durante todo o episódio (a balsa não
se move em relação à água). O barco se
afasta da balsa durante 1 h, percorrendo a
distância AB com velocidade constante V (no
referencial da água). Em seguida, o piloto do
barco desliga o motor para conserto durante
0,5 h, intervalo de tempo no qual barco, balsa
e água permanecem parados (no referencial
da água).
15. Resolução:
No referencial dos carros
Essa avenida é percorrida por uma
sequência ilimitada de carros, todos se
movendo com velocidade constante V (em
relação à Terra). Segundo o enunciado, eles
estão dispostos de forma que a distância livre
entre dois carros consecutivos sempre vale
a. Sendo ilimitada a sequência de carros, o
pedestre necessariamente atravessará a
avenida por uma trajetória retilínea que
passa entre dois carros consecutivos.

Entretanto, como o enunciado do problema


pede que a velocidade 𝑉 ⃗⃗ pT do pedestre em
relação à Terra seja mínima, essa condição
é satisfeita quando 𝑉⃗⃗ pT aponta
perpendicularmente à reta AB, como pode
ser visto na figura.

𝑣𝑃𝑇 = 𝑣𝑇𝑐 . sin 𝛼

Para que o barco parta do ponto A da 𝑣. 𝑏


margem e chegue ao ponto B da margem 𝑣𝑃𝑇𝑚í𝑛 =
√𝑎2 + 𝑏 2
oposta. seguindo a trajetória AB em relação
à Terra (margens), a 16 Resolução:
velocidade do barco em relação à Terra V
deve apontar na direção da reta AB.
Ora, quando o vetor O
representar a distância mínima entre o ponto
P e a reta AB, isto é, quando O for
perpendicular à direção AB como mostra a
Figura 2.

𝑈𝑚í𝑛 𝑏
sin 𝛼 = =
𝑣0 √𝑎 + 𝑏 2
2

𝑣0 𝑏
𝑈𝑚í𝑛 =
√𝑎2 + 𝑏 2
c) Lado Oeste, pois a velocidade resultante,
medida em relação à superfície terrestre, é
maior.

17 Resolução:

15
tan 30° =
𝑣𝑟𝑒𝑙
Aplicando-se o Teorema de Pitágoras ao
√3 15 triângulo retângulo destacado na figura, vem:
=
3 𝑣𝑟𝑒𝑙
45 d=10m logo, sin 𝜃 = 0,8
√3 =
𝑣𝑟𝑒𝑙
|𝑑⃗|
|𝑣⃗𝑚 | = = 5𝑚/𝑠
2º caso: ∆𝑡
𝑣 20 Resolução:
tan 60° = = √3
𝑣𝑟𝑒𝑙 a) O movimento de A em relação ao solo é
circular e uniforme, e sua aceleração vetorial
é centrípeta.
V = 45 m/s 𝑣2
𝑎𝐴 = = 8𝑚/𝑠 2
𝑅
18. Resolução: b)

𝑏 𝑣𝑇 Triângulo pitagórico
a) tan 𝛼 = 𝑎 = 𝑣
vAB = 25 m/s = 90 km/h
v = 30 km/h
c)
b) Teorema de Pitágoras:

𝑣𝑅2 = 𝑣𝑇2 + 𝑣
vR = 50 km/h
aAB = 12 m/s2
19 Resolução:
a) 21 Resolução:
∆𝑠
Subida: ∆𝑡1 = 𝑣 −𝑣
𝐵 𝑐
Solução de vR de volta:
∆𝑠
Descida: ∆𝑡2 = 𝑣
𝐵 +𝑣𝑐 𝑣𝑅2 − 2𝑣. cos(𝜋 − 𝜃) 𝑣𝑅 + (𝑣 2 − 𝑉 2 ) = 0

Da definição de vel. Escalar média: Como

2∆𝑠 cos(𝜋 − 𝜃) = − cos 𝜃


𝑣𝑚 =
∆𝑡1 + ∆𝑡2
𝑣𝑅2 + 2𝑣. cos 𝜃 𝑣𝑅 + (𝑣 2 − 𝑉 2 ) = 0
2∆𝑠
𝑣𝑚 = ∆= 4𝑉 2 − 4 𝑣 2 sin 𝜃 2
∆𝑠 ∆𝑠
(𝑣 − 𝑣 ) + (𝑣 + 𝑣 )
𝐵 𝑐 𝐵 𝑐
𝑣𝑣𝑜𝑙𝑡𝑎 = −𝑣 cos 𝜃 + √𝑉 2 − 𝑣 2 sin 𝜃 2
2
𝑣𝑚 = 𝑡 = 𝑡𝑖𝑑𝑎 + 𝑡𝑣𝑜𝑙𝑡𝑎
(𝑣𝐵 + 𝑣𝑐 ) + (𝑣𝐵 − 𝑣𝑐 )
(𝑣𝐵 + 𝑣𝑐 ). (𝑣𝐵 − 𝑣𝑐 )
Substituindo...
𝑣𝐵2 − 𝑣𝑐2 2
𝑣𝑚 = 2𝐿√𝑉2 − 𝑣2 sin 𝜃
𝑣𝐵 𝑡=
𝑉2 − 𝑣2
22 Resolução do Professor Laerte Pereira: Botando 𝑉 2 em evidencia...

𝑣𝐴,𝑆 = 𝑣𝑅
𝑣𝐴,𝑎𝑟 = 𝑉 𝑣2 sin 𝜃2
2𝐿𝑉. √(1 − )
𝑉2
𝑣𝑎𝑟,𝑆 = 𝑣
𝑡=
𝑣2
𝑉 2 (1 − )
𝑉2

√(1 −
𝑣2 sin 𝜃2
)
2𝐿 𝑉2
𝑡=( )
𝑉 𝑣2
(1 − )
𝑉2

23 Resolução:
podemos representar a posição do
observador 2 em relação ao 1 pela equação:
𝑟⃗ = (𝑥, 𝑦) = 𝑟⃗1 − 𝑟⃗2

𝑥 = −2 cos(𝑡)
{
𝑣𝑅2 − 2𝑣. cos 𝜃 𝑣𝑅 + (𝑣 2 −𝑉 2)
=0 𝑦 = − sin(𝑡)
Isolando os valores de cos(𝑡) e sen(𝑡) nas
∆= 4𝑉 2 − 4 𝑣 2 sin 𝜃 2 equações acima e usando a identidade
trigonométrica sin 𝑡 2 + cos 𝑡 2 = 1
Solução de vR de ida:
𝑥2
+ 𝑦2 = 1
𝑣𝑖𝑑𝑎 = 𝑣 cos 𝜃 + √𝑉 2 − 𝑣 2 sin 𝜃 2 4
24. Resolução: 𝑉4
Calculando o tempo de descida do primeiro 𝑉22 = 𝑉12 − 𝑉 2 +
𝑉12
barco, obtemos: Substituindo 𝑉1, temos:
𝑥
𝑡𝑑 =
𝑉1 + 𝑉
1+𝑟 2 1−𝑟 2
Tempo de subida: 𝑉2 = 𝑉 √( ) −1+( ) .
𝑥 1−𝑟 1+𝑟
𝑡𝑠 =
𝑉1 − 𝑉
A razão entre esses tempos é dada pela 25. Resolução do prof. Laerte Pereira:
questão: Uma mudança para o referencial do papel
𝑡𝑑 𝑉1 − 𝑉 pode facilitar.
𝑟= =
𝑡𝑠 𝑉1 + 𝑉
Logo
1+𝑟
𝑉1 = 𝑉 ( )
1−𝑟

O tempo total de viagem do barco 1 é: No referencial do solo, a formiga deve se


𝑡1 = 𝑡𝑠 + 𝑡𝑑 mover perpendicularmente à borda do papel.
Componente em Y de sua velocidade:
2𝑥𝑉1
𝑡1 = 10𝑐𝑚
𝑉12 − 𝑉 2 𝑣𝑦 = = 2𝑐𝑚/𝑠
A velocidade do barco 2 em relação ao rio 5𝑠
(desenvolvida pelo seu motor) deve ser
inclinada, de modo que a resultante tem a X componente da velocidade da formiga no
direção 𝐴𝐶: referencial do solo deve ser zero.

𝑣𝑥 = 2𝑐𝑚/𝑠

A velocidade da formiga em relação ao papel


é

𝑉 = √𝑣𝑥2 + 𝑣𝑦2

𝑉 = 2√2𝑚/𝑠

O tempo total de viagem do barco 2 é: 26. Suponha que a velocidade do fluxo do rio
𝑥 vr = v, então a velocidade do barco em
𝑡2 = 2 ( ) relação a água vbr = v/2. Deixe o barco se
√𝑉22 + 𝑉 2 mover em um ângulo θ com a direção do
fluxo, então o tempo para atravessar o rio
O problema nos diz que: 𝑡1=𝑡2 𝑏
𝑡=
𝑣𝑏𝑟 . sin 𝜃
2𝑉1 1
= 2( ) Onde 𝑣𝑏𝑟 . sin 𝜃 é a velocidade do barco ao
𝑉12 −𝑉 2
√𝑉22 + 𝑉 2 longo de AB. A velocidade do barco ao longo
Isolando V2 da direção do fluxo
(𝑉12 − 𝑉 2 )2
𝑉22 = 𝑉 2 + 𝑣𝑏𝑥 = (𝑣𝑟 + 𝑣𝑏𝑟 cos 𝜃)
𝑉12
A distância rio a baixo
𝑦 = 𝑢𝑡
𝑥 = (𝑣𝑟 + 𝑣𝑏𝑟 cos 𝜃). 𝑡
2𝑣0
𝑏 𝑣𝑥 = ( ) 𝑢. 𝑡
𝑏
𝑥 = (𝑣𝑟 + 𝑣𝑏𝑟 cos 𝜃). A aceleração ao longo do eixo x é constante
𝑣𝑏𝑟 . sin 𝜃

𝑣 𝑏 𝑑𝑣𝑥 2𝑣0
𝑥 = (𝑣 + cos 𝜃) . 𝑣 𝑎𝑥 = = .𝑢
2 𝑑𝑡 𝑏
2 . sin 𝜃
Então o deslocamento ao longo do eixo X é
2 + cos 𝜃
𝑥 = 𝑏. ( )
sin 𝜃 1
𝑥 = 𝑢𝑥 . 𝑡 + 𝑎𝑥 𝑡 2
2
onde b é a largura do rio. Para que o desvio
seja mínimo 1 2𝑣0 . 𝑢 2
𝑥 = 0. 𝑡 + ( )𝑡
2 𝑏
𝑑𝑥 Como
=0
𝑑𝑡 𝑦 = 𝑢𝑡
𝑦
sin 𝜃 . (− sin 𝜃) − (2 + cos 𝜃). cos 𝜃 = 0 𝑡=
𝑢
sin 𝜃 2 + 2 cos 𝜃 + cos 𝜃 2 = 0
1 2𝑣0 . 𝑢 𝑦 2
1 𝑥= ( )
cos 𝜃 = − 2 𝑏 𝑢
2

𝜃 = 120° Ou
Logo
𝑢𝑏. 𝑥
𝑦2 =
1 𝑣0
2−2
𝑥𝑚í𝑛 = 𝑏. ( ) = 𝑏. √3 A equação é uma equação de uma parábola,
√3
2 então, a trajetória do barco é uma parábola
OA até o meio do rio. A outra metade da
27. resolução do prof. Laerte Pereira: trajetória é da mesma natureza.
Quando o barco está em y = b/2
𝑣𝑥 = 𝑘. 𝑦 𝑣0 𝑏 2 𝑣0 𝑏
Onde k é constante. 𝑥= ( ) =
𝑢𝑏 2 4𝑢
Quando A equação acima fornece o desvio ao longo
do eixo x para a primeira metade. Durante a
𝑏 segunda metade do movimento, também é x.
𝑦= ; 𝑣 = 𝑣0
2 Assim, deriva total ao longo da direção do
fluxo do rio = 2x
𝑘. 𝑏 𝑣0 𝑏
𝑣0 = 𝑥=
2 2𝑢
2𝑣0 28. Resolução:
𝑘=
𝑏
O nadador tem velocidade dirigida
2𝑣0 perpendicularmente à correnteza (em
𝑣𝑥 = .𝑦
𝑏 relação ao rio) cuja equação é:
O deslocamento ao longo do eixo y é: 𝑣 = 10 − 2𝑡
Movimento desacelerado. 𝑢
= 𝑣√2
sin 𝜃
𝑢
= 𝑣√2
sin 𝛼
𝑢
sin 𝜃 = sin 𝛼 =
𝑣. √2
Movimento em y: 2𝑎 2𝑎
𝑡= 𝑢 + 𝑢
𝑣𝑓2 = 𝑣02 + 2𝑎𝑑𝑦 𝑣 cos 𝜃 + 𝑣 cos 𝜃 −
√2 √2
2𝑣 cos 𝜃
𝑣𝑓2 = 102 + 2. (−2). 16 𝑡 = 2𝑎 [ ]
𝑢2
𝑣𝑓 = 6𝑚/𝑠 𝑣 2 cos 𝜃 − 2

𝑣𝐵 = √42 + 62 = 2√13 𝑚/𝑠 4𝑣 cos 𝜃


𝑡 = 2𝑎 [ ]
2𝑣 2 cos 𝜃 − 𝑢2
29. Resolução:
a) 4𝑣. √2𝑣 2 − 𝑢2
( )
𝑣√2
𝑡 = 2𝑎
2𝑣 2 − 𝑢2 𝑢2 . 2𝑣 2
2𝑣 2 ( 2 )−
[ 2𝑣 2𝑣 2 ]

4. √2𝑣 2 − 𝑢2 2𝑣 2
𝑡 = 2𝑎 [ . ]
√2 (2𝑣 2 (2𝑣 2 − 𝑢2 )) − 𝑢2 . 2𝑣 2

4. √2𝑣 2 − 𝑢2 1
𝑡 = 2𝑎 [ . ]
√2 (2𝑣 − 𝑢2 ) − 𝑢2
2
𝑎 𝑎 𝑎 𝑎
𝑡= + + +
𝑣 + 𝑢 √𝑣 2 − 𝑢2 𝑣 − 𝑢 √𝑣 2 − 𝑣 2

2𝑎𝑣 2𝑎 4. √2𝑣 2 − 𝑢2 1
𝑡= + 𝑡 = 2𝑎 [ . ]
𝑣 2 − 𝑢2 √𝑣 2 − 𝑢2 √2 (2𝑣 2 − 2𝑢2 )

2𝑎𝑣 2𝑎 √𝑣 2 − 𝑢2 2. √2𝑣 2 − 𝑢2 1
𝑡= + . ( )
𝑣 2 − 𝑢2 √𝑣 2 − 𝑢2 √𝑣 2 − 𝑢2 𝑡 = 2𝑎 [ . 2 ]
√2 (𝑣 − 𝑢2 )
𝑣 + √𝑣2 − 𝑢2
𝑡 = 2𝑎 ( ) √2. √2𝑣 2 − 𝑢2
2
𝑣 −𝑢 2 𝑡 = 2𝑎 [ ]
(𝑣 2 − 𝑢2 )
b)
Lançamento Oblíquo 𝑟⃗ = 𝑥𝑖̂ + 𝑦𝑗̂ eq(i)
Deslocamento na horizontal para um tempo t é
dado por
𝑥 = 𝑣𝑥 𝑡 ⇒ 𝑥 = 𝑢 𝑐𝑜𝑠 𝜃 𝑡 eq(ii)
A velocidade vertical da partícula para um
tempo t é
𝑣𝑦 = (𝑣0 )𝑦 − 𝑔𝑡, eq(iii)
O deslocamento na vertical é
𝑦 = 𝑢 𝑠𝑖𝑛 𝜃 𝑡 − 1/2𝑔𝑡 2 eq(iv)
Combinando os valores de x e y na eq (i) nos
Equação da trajetória obtemos o vetor posição para um instante de
Para o movimento na horizontal tempo t
𝑥 1
x = u cos . t  𝑡 = 𝑢 𝑐𝑜𝑠 𝜃 eq (i) 𝑟⃗ = (𝑢 𝑐𝑜𝑠 𝜃)𝑡𝑖̂ + ((𝑢 𝑠𝑖𝑛 𝜃)𝑡 − 2 𝑔𝑡 2 ) 𝑗̂
Para a vertical Em módulo
1 2
𝑦 = (𝑢 𝑠𝑖𝑛 𝜃)𝑡 − 𝑔𝑡 2 eq(ii) 𝑟 = √(𝑢𝑡 𝑐𝑜𝑠 𝜃)2 + ((𝑢𝑡 𝑠𝑖𝑛 𝜃) − 𝑔𝑡 2 )
1
2
2

𝑔𝑡 2 𝑔𝑡 𝑠𝑖𝑛 𝜃
Das equações (i) e (ii) 𝑟 = 𝑢𝑡√1 + ( ) −
2𝑢 𝑢
𝑥 1 𝑥2 e
𝑦 = 𝑢 𝑠𝑖𝑛 𝜃 ( )− 𝑔( 2 )
𝑢 𝑐𝑜𝑠 𝜃 2 𝑢 𝑐𝑜𝑠 2 𝜃 𝑢𝑡 𝑠𝑖𝑛 𝜃− 𝑔𝑡 2
1

1 𝑔𝑥 2
𝜑 = 𝑡𝑎𝑛−1 ( 𝑦/𝑥) = 𝑡𝑎𝑛−1 ( (𝑢𝑡 𝑐𝑜𝑠 𝜃)
2
)
𝑦 = 𝑥 𝑡𝑎𝑛 𝜃 − 2 𝑢2 𝑐𝑜𝑠2 𝜃
ou
Esta equação mostra que a trajetória do projétil é 2𝑢 𝑠𝑖𝑛 𝜃−𝑔𝑡
parabólica porque é semelhante à equação da 𝜑 = 𝑡𝑎𝑛−1 ( )
2𝑢 𝑐𝑜𝑠 𝜃
parábola.
Note: A equação do projétil oblíquo também Momento angular (L)
pode ser escrita como
𝑥
𝑦 = 𝑥. 𝑡𝑎𝑛 𝜃 . [1 − 𝑅]
𝑢2 𝑠𝑖𝑛 2𝜃
(onde R = alcance horizontal = 𝑔
)
Vetor deslocamento (𝒓
⃗⃗)
A partícula adquire uma posição P com as
coordenadas (x, y) imediatamente após o tempo
t a partir do instante de lançamento. O vector 𝐿 = 𝑚𝑣𝑟
posição 𝒓 ⃗⃗ correspondente da partícula no 𝑢2 𝑠𝑖𝑛2 𝜃
[sendo 𝑟 = 𝐻 = ]
instante t é o indicado na imagem 2𝑔
𝑢2 𝑠𝑖𝑛2 𝜃 𝑚𝑢3 𝑐𝑜𝑠 𝜃 𝑠𝑖𝑛2 𝜃
 𝐿 = 𝑚𝑢 𝑐𝑜𝑠 𝜃 =
2𝑔 2𝑔
EXERCÍCIOS 2(h − 2r)
a) 3v
g
1. (Ita 2023) Considere um recipiente que 2(h − r)
b) 3v
contém uma coluna de água de altura H. Um g
pequeno furo é feito na parede a uma altura h,
2(h − r)
de tal forma que um filete de água é expelido c) v
horizontalmente, como na figura. Considere a g
água um fluido incompressível e de viscosidade 2(h − 2r)
desprezível. A aceleração local da gravidade d) v
g
vale g.
Determine: 2(h − R − r)
e) 3v
g

3. (Ita 2021) Um recipiente, de secção de área


constante e igual a A, é preenchido por uma
coluna de líquido de densidade ρ é altura H.
Sobre o líquido encontra-se um pistão de massa
M, que pode se deslocar verticalmente livre de
atrito. Um furo no recipiente é feito a uma altura
h, de tal forma que um filete de água é expelido
conforme mostra a figura. Assinale a alternativa
que contém o alcance horizontal D do jato de
água.
a) a trajetória y(x) do filete de água descrito;
b) o lugar geométrico dos pontos P(x; y) que
podem ser atingidos por um filete de água,
considerando que a altura h possa ser
escolhida entre 0 e H.

2. (Ita 2021) Uma bola de gude de raio r e uma


bola de basquete de raio R são lançadas contra
uma parede com velocidade horizontal v e com
seus centros a uma altura h. A bola de gude e
a bola de basquete estão na iminência de
contato entre si, assim como ambas contra a
parede. Desprezando a duração de todas as
colisões e quaisquer perdas de energia, calcule
o deslocamento horizontal ΔS da bolinha de
gude ao atingir o solo.

 m
a) 2 (H − h)  h + 
 ρA 
 m
b) 2h  H − h + 
 ρA 
 m
c) 2 h  H − h + 
 ρA 
 m 
d) 2 h  H − h + 
 2ρA 
 m
e) h  H − h + 
 ρA 
4. (Ime 2019)

A figura mostra uma haste de massa


desprezível com um apoio articulado em uma 6. (Ime 2018)
extremidade. A outra extremidade possui um
recipiente apoiado em uma mola e amarrado ao
solo por um fio. A haste é mantida na posição
horizontal e a mola comprimida. Uma bola é
colocada nesse recipiente e, após o corte do fio,
o sistema é liberado com distensão instantânea
da mola.

A constante elástica da mola, em N m, para


que, quando a prancha estiver perpendicular ao
solo, a bola seja lançada e acerte o cesto é:
Conforme a figura acima, um corpo, cuja
Dados: velocidade é nula no ponto A da superfície
- comprimento da prancha: 1 m; circular de raio R, é atingido por um projétil,
- distância do apoio ao cesto: 5 m; que se move verticalmente para cima, e fica
- massa da bola: 200 g; alojado no corpo. Ambos passam a deslizar sem
atrito na superfície circular, perdendo o contato
- deformação inicial da mola: 10 cm; e com a superfície no ponto B. A seguir, passam
- aceleração da gravidade: 10 m s2 . a descrever uma trajetória no ar até atingirem o
ponto C, indicado na figura. Diante do exposto,
Observação: a velocidade do projétil é:
- despreze as dimensões da bola.
a) 400 Dados:
b) 500
c) 2.900 - massa do projétil: m;
d) 3.400 - massa do corpo: 9m; e
e) 12.900 - aceleração da gravidade: g.
5Rg
5. (Ita 2018) Na figura, presa a um fio de a) 10
2
comprimento de 1,0 m, uma massa de 1,0 kg
3Rg
gira com uma certa velocidade angular num b) 10
plano vertical sob a ação da gravidade, com 2
eixo de rotação a h = 6,0 m do piso. Determine c) 10
5Rg
a velocidade angular mínima dessa massa para 3
a ruptura do fio que suporta no máximo a tração 3Rg
d) 10
de 46 N, bem como a distância ao ponto P do 5
ponto em que, nesse caso, a massa tocará o 2Rg
solo. e) 10
3

7. (Ita 2018) A partir de um mesmo ponto a


uma certa altura do solo, uma partícula é
lançada sequencialmente em três condições
diferentes, mas sempre com a mesma
velocidade inicial horizontal v 0 . O primeiro
lançamento é feito no vácuo e o segundo, na
atmosfera com ar em repouso. O terceiro é feito
na atmosfera com ar em movimento cuja
velocidade em relação ao solo é igual em
módulo, direção e sentido à velocidade v 0 .
Para os três lançamentos, designando-se
respectivamente de t1, t2 e t 3 os tempos de
queda da partícula e de v1, v 2 e v 3 os
módulos de suas respectivas velocidades ao Sendo g a aceleração da gravidade, e não
atingir o solo, assinale a alternativa correta. havendo atrito e nem resistência de qualquer
a) t1  t3  t2; v1  v3  v 2 outra natureza, o módulo da velocidade com
b) t1  t2 = t3 ; v1  v3  v 2 que o conjunto atinge o solo vale
2
c) t1 = t3  t2; v1 = v3  v2  mv 
a)  0
 + 2gh
d) t1  t2  t3 ; v1 = v3  v2  m + M
e) t1  t2 = t3 ; v1  v 2 = v3 2ghm2
b) v02 +
(m + M)2
8. (Ime 2014) Um banhista faz o lançamento
horizontal de um objeto na velocidade igual a c) v 02 +
2mgh
5 3 m s em direção a uma piscina. Após M
tocar a superfície da água, o objeto submerge d) v 02 + 2gh
até o fundo da piscina em velocidade horizontal
desprezível. Em seguida, o banhista observa mv 02
e) + 2gh
esse objeto em um ângulo de 30 em relação m+M
ao horizonte. Admitindo-se que a altura de
observação do banhista e do lançamento do 10. (Ime 2021)
objeto são iguais a 1,80 m em relação ao nível
da água da piscina, a profundidade da piscina,
em metros,

Dados:
- índice de refração do ar: nar = 1;
5 3
- índice de refração da água: nágua =
6
a) 2
b) 1,6
c) 1,6 3 Uma partícula de massa m e carga q positiva
d) 2 3 é lançada obliquamente com velocidade v 0 e
e) 3 ângulo α com a horizontal, conforme a figura.
Em certo instante t1, antes de alcançar a altura
9. (Ita 2007) Uma bala de massa m e máxima de sua trajetória, quando está a uma
velocidade v 0 é disparada contra um bloco de distância horizontal x1 do ponto de lançamento,
massa M, que inicialmente se encontra em a partícula é submetida a um campo magnético
repouso na borda de um poste de altura h, de intensidade B, na direção vertical.
Considerando g a aceleração da gravidade
conforme mostra a figura. A bala aloja-se no
bloco que, devido ao impacto, cai no solo. local, a menor intensidade B do campo
magnético para que a partícula atinja o solo na
posição (x1, 0) é:
2 πm que A e C designam as posições da partícula,
a)
 2v sen( α)  respectivamente, em t = t f − 5 s e t = t f − 2 s;
q 0 − t1 
 g  e que a resistência do ar pode ser desprezada,
πm responda o que se pede:
b)
 2v sen(α ) 
q 0 − t1  a) faça um esboço do gráfico da altura y da
 g 
partícula versus o tempo t, desde seu
2 πm
c) lançamento até alcançar o ponto D,
 v sen(α ) 
q 0 − t1  explicitando a altura máxima alcançada, a do
 g  ponto A e a do ponto C, com os
4 πm correspondentes tempos; e
d)
 2v sen(α )  b) determine as coordenadas xC e zC do
q 0 − t1 
 g  ponto C.
πm
e)
 v sen(α )  Dados:
q 0 − t1 
 2g  - plano de lançamento da partícula
225 3
z = z0 = m;
11. (Ime 2020) π
- aceleração da gravidade: g = 10 m s2 ;
- velocidade inicial: v0 = 100 m s;
- ângulo de lançamento da partícula: α = 30;
- altura inicial da partícula: h0 = 280 m.

12. (Ime 2015)

Uma mola comprimida por uma deformação x


está em contato com um corpo de massa m,
que se encontra inicialmente em repouso no
Uma partícula de massa m e carga elétrica Ponto A da rampa circular. O corpo é liberado
positiva + q é lançada obliquamente com e inicia um movimento sem atrito na rampa. Ao
atingir o ponto B sob um ângulo θ indicado na
inclinação α, em t = 0, no plano z = z0 , a
figura, o corpo abandona a superfície da rampa.
uma velocidade inicial v 0 a partir da altura No ponto mais alto da trajetória, entra em
y = h0 , conforme ilustra a figura. Em contato com uma superfície plana horizontal
determinado instante de sua trajetória, a com coeficiente de atrito cinético μ. Após
partícula é submetida a um campo magnético deslocar-se por uma distância d nesta
uniforme B = (0, B, 0), cuja intensidade varia superfície horizontal, o corpo atinge o repouso.
ao longo do tempo de acordo com o gráfico. Determine, em função dos parâmetros
Sabendo que t f representa o instante em que a mencionados:
partícula encerra seu movimento no ponto D de
coordenadas (xD , 0, 0), ao atingir o plano xz; a) a altura final do corpo Hf em relação ao solo;
b) a distância d percorrida ao longo da 15. (Ita 2010) Um pequeno bloco desliza sobre
superfície plana horizontal. uma rampa e logo em seguida por um “loop”
circular de raio R, onde há um rasgo de
Dados: comprimento de arco 2R, como ilustrado na
- aceleração da gravidade: g; figura. Sendo g a aceleração da gravidade e
- constante elástica da mola: k; desconsiderando qualquer atrito, obtenha a
- raio da rampa circular: h. expressão para a altura inicial em que o bloco
deve ser solto de forma a vencer o rasgo e
13. (Ita 2013) Uma pequena bola de massa m é continuar em contato com o restante da pista.
lançada de um ponto P contra uma parede
vertical lisa com uma certa velocidade v0, numa
direção de ângulo α em relação à horizontal.
Considere que após a colisão a bola retorna ao
seu ponto de lançamento, a uma distância d da
parede, como mostra a figura. Nestas
condições, o coeficiente de restituição deve ser

16. (Enem 2015) Um garoto foi à loja comprar


um estilingue e encontrou dois modelos: um
com borracha mais “dura” e outro com borracha
mais “mole”. O garoto concluiu que o mais
adequado seria o que proporcionasse maior
alcance horizontal, D, para as mesmas
a) e = gd ( v02 sen2α − gd) .
condições de arremesso, quando submetidos à
b) e = 2gd ( v02 cos2α − 2gd) . mesma força aplicada. Sabe-se que a constante
elástica k d (do estilingue mais “duro”) é o dobro
c) e = 3gd ( 2v02 sen2α − 2gd).
da constante elástica km (do estilingue mais
d) e = 4gd ( v02 cos2α − gd) . “mole”).
e) e = 2gd ( v02 tan2α − gd).
Dd
A razão entre os alcances , referentes aos
Dm
14. (Ita 2011) Duas partículas idênticas, de
mesma massa m, são projetadas de uma estilingues com borrachas “dura” e “mole”,
origem O comum, num plano vertical, com respectivamente, é igual a
velocidades iniciais de mesmo modulo v0 e 1
a) .
ângulos de lançamento respectivamente  e  4
1
em relação a horizontal. Considere T1 e T2 os b) .
respectivos tempos de alcance do ponto mais 2
alto de cada trajetória e t1 e t2 os respectivos c) 1.
tempos para as partículas alcançar um ponto d) 2.
comum de ambas as trajetórias. Assinale a e) 4.
opção com o valor da expressão t1T1 + t2T2.
a) 2v02 ( tg + tg) / g2 17. (Epcar (Afa) 2023) Uma partícula é lançada
obliquamente e descreve um movimento
b) 2v 02 / g2 parabólico, sem resistência do ar. No momento
c) 4v 02 sen / g2 do lançamento dessa partícula, o vetor
d) 4v 02 sen / g2 velocidade (v 0 ) faz o ângulo θ com a
e) 2v02 ( sen + sen) / g2 horizontal e, ao atingir a altura máxima de sua
trajetória, o vetor posição (H) da partícula faz
um ângulo α com essa mesma horizontal, primeiro degrau de altura h em relação ao
conforme ilustra figura a seguir: segundo degrau. A esfera atinge um ponto X na
superfície perfeitamente lisa do segundo
degrau, que tem um comprimento D, e,
imediatamente, começa a deslizar sem rolar,
também com velocidade horizontal V0
constante, até chegar na extremidade do
segundo degrau. Ela, então, percorre uma altura
2h na vertical e atinge o solo a uma distância L
da base do segundo degrau, conforme
representado no desenho abaixo. Podemos
afirmar que o intervalo de tempo que a esfera
leva, deslizando sem rolar, na superfície lisa do
segundo degrau é de:
Nessas condições, a razão entre as tangentes
tgθ
de θ e α, . vale
tgα
a) 1,5
b) 2,0
c) 2,5
d) 3,0

18. (Espcex (Aman) 2023) Uma granada de


massa M é lançada do solo plano e horizontal
com uma velocidade inicial V0 formando um
ângulo θ com o sentido positivo do eixo
horizontal X. Na altura máxima da sua trajetória
parabólica, ela explode em dois fragmentos F1
e F2. O fragmento F1 de massa M/4,
imediatamente após a explosão, adquire uma
velocidade V1, vertical e orientada para baixo Dados: despreze a força de resistência do ar e
considere o módulo da aceleração da gravidade
ao longo do sentido negativo do eixo Y. O igual a g.
intervalo de tempo entre o instante
imediatamente após a explosão da granada e o
a) [ g  (D + L) − 6h  V0 ] (V0  g)
instante em que o fragmento F2 toca o solo é
de: b) [ g  (D + L) + 6h  V0 ] (V0  g)
c) [ g  (D + L) + h  V0  ( 2 − 2)] (V0  g)
Dados: Despreze a resistência do ar, considere
que o módulo da aceleração da gravidade é d) [ g  (D + L) − h  V0  ( 2 + 2)] (V0  g)
igual a g e que as trajetórias da granada e dos e) [ h  V0  ( 2 + 2) − g  (D + L)] (V0  g)
fragmentos estão apenas no plano XY.
a)  V1 3 + V0 (1 9 − sen θ) / g 20. (Esc. Naval 2022) Um projétil de peso P é
lançado a partir de uma superfície horizontal
b)  V1 3 + V0 (1 9 − sen θ) / g
plana. Considere que durante todo o voo do
c)  V1 3 + V0 (1 9 + sen θ) / g projétil sopra um vento contrário, na horizontal,
aplicando ao projétil uma força horizontal
d)  V1 3 + (V12 9 + V02sen2θ)  / g constante de módulo P/4. Sabendo-se que não
 
 2 
há interferência do vento na direção vertical, de
e)  V1 3 + (V1 9 − V0 sen θ)  / g
2 2
que ângulo com a superfície horizontal o projétil
 
deve ser lançado de modo que a altura máxima
19. (Espcex (Aman) 2022) Em uma escada, alcançada pelo projétil seja igual ao seu alcance
uma esfera é lançada com velocidade ao retornar ao solo?
horizontal, de módulo V0, da extremidade do a) arctg (2)
b) arctg (1)
4 XB
c) arctg   Considerando RB = 4RA , a razão será
 
5 XA
2 igual a
d) arctg  
3 a)
1
 1 4
e) arctg   1
2 b)
2
c) 2
21. (Epcar (Afa) 2022) Na Figura 1, a seguir, d) 4
tem-se uma vista de cima de um movimento
circular uniforme descrito por duas partículas, A
e B, que percorrem trajetórias semicirculares, de 22. (Epcar (Afa) 2021) Uma partícula de massa
raios R A e RB , respectivamente, sobre uma M é lançada obliquamente com sua velocidade
mesa, mantendo-se sempre alinhadas com inicial v 0 fazendo um ângulo de 30 com a
centro C. direção horizontal, conforme indica figura a
seguir.

Ao atingir a altura máxima de sua trajetória


parabólica, essa partícula colide inelasticamente
com um bloco de massa 5M. Esse bloco, de
dimensões desprezíveis, está preso ao teto por
um fio ideal, de comprimento 1,2 m, formando
um pêndulo balístico. Inicialmente o fio do
Ao chegarem à borda da mesa, conforme ilustra pêndulo está na vertical. Após a colisão, o
a Figura 2, as partículas são lançadas pêndulo atinge uma altura máxima, na qual o fio
horizontalmente e descrevem trajetórias tem uma inclinação de 30 em relação à
parabólicas, livres de quaisquer forças de direção horizontal.
resistência, até chegarem ao piso, que é plano e Desprezando a resistência do ar, o módulo da
horizontal. Ao longo dessa queda, as partículas velocidade inicial da partícula, v 0 , em m s, é
A e B percorrem distâncias horizontais, X A e
igual a
XB , respectivamente. a) 5,0
b) 10
c) 15
d) 24

23. (Efomm 2018) Em uma mesa de 1,25


metros de altura, é colocada uma mola
comprimida e uma esfera, conforme a figura.
Sendo a esfera de massa igual a 50 g e a mola
comprimida em 10 cm, se ao ser liberada a
esfera atinge o solo a uma distância de 5
metros da mesa, com base nessas informações,
pode-se afirmar que a constante elástica da
mola é:
(Dados: considere a aceleração da gravidade
igual a 10 m s2 .)

a) 62,5 N m A figura acima mostra um pequeno bloco,


inicialmente em repouso, no ponto A,
b) 125 N m
correspondente ao topo de uma esfera
c) 250 N m perfeitamente lisa de raio R = 135 m. A esfera
d) 375 N m está presa ao chão no ponto B. O bloco
e) 500 N m começa a deslizar para baixo, sem atrito, com
uma velocidade inicial tão pequena que pode
24. (Epcar (Afa) 2017) Uma partícula de massa ser desprezada, e ao chegar ao ponto C, o
m, presa na extremidade de uma corda ideal, bloco perde contato com a esfera. Sabendo que
descreve um movimento circular acelerado, de a distância horizontal percorrida pelo bloco
raio R, contido em um plano vertical, conforme durante seu voo é d = 102 m, o tempo de voo
figura a seguir. do bloco, em segundos, ao cair do ponto C ao
ponto D vale

Dado: g = 10 m s2
a) 1,3
b) 5,1
c) 9,2
d) 13
e) 18

Quando essa partícula atinge determinado valor TEXTO PARA AS PRÓXIMAS 2 QUESTÕES:
de velocidade, a corda também atinge um valor Se necessário, use
máximo de tensão e se rompe. Nesse momento, aceleração da gravidade: g = 10 m / s2
a partícula é lançada horizontalmente, de uma
densidade da água: d = 1,0 kg / L
altura 2R, indo atingir uma distância horizontal
igual a 4R. Considerando a aceleração da calor específico da água: c = 1cal / g C
gravidade no local igual a g, a tensão máxima 1cal = 4 J
experimentada pela corda foi de constante eletrostática:
a) mg k = 9 ,0  109 N  m2 / C2
b) 2 mg constante universal dos gases perfeitos:
c) 3 mg R = 8 J / mol  K
d) 4 mg

26. (Epcar (Afa) 2016) Uma partícula de massa


25. (Esc. Naval 2016) Analise a figura abaixo.
m e carga elétrica −q é lançada com um
ângulo θ em relação ao eixo x, com
velocidade igual a v 0 , numa região onde atuam
um campo elétrico E e um campo gravitacional
g, ambos uniformes e constantes, conforme
indicado na figura abaixo.

Desprezando interações de quaisquer outras


naturezas com essa partícula, o gráfico que Num dado momento em que as partículas
melhor representa a variação de sua energia passam, simultaneamente, tangenciando o
potencial ( Ep ) em função da distância (d) plano horizontal α, elas são desacopladas dos
percorrida na direção do eixo x, é mecanismos de giro e, lançadas
horizontalmente, seguem as trajetórias 1 e 2
(figura 1) até se encontrarem no ponto P.
Os gráficos das energias cinéticas, em joule,
das partículas 1 e 2 durante os movimentos de
queda, até a colisão, são apresentados na
figura 2 em função de (h − y), em m, onde y
a) é a altura vertical das partículas num tempo
qualquer, medida a partir do solo perfeitamente
horizontal.

b)

c)
Desprezando qualquer forma de atrito, a razão
ω2
é
ω1
a) 1
b) 2
c) 3
d) d) 4

27. (Epcar (Afa) 2016) Dois mecanismos que 28. (Esc. Naval 2015) Analise a figura abaixo.
giram com velocidades angulares ω1 e ω2
constantes são usados para lançar
horizontalmente duas partículas de massas
m1 = 1kg e m2 = 2 kg de uma altura
h = 30 m, como mostra a figura 1 abaixo.
posição em função do tempo é descrita pela
equação
r(t) = (6,0t + 2,5)i + ( −5,0t 2 + 2,0t + 8,4) j,
com r em metros e t em segundos. Após 1,0
segundo, as medidas de sua altura do solo, em
metros, e do módulo da sua velocidade, em
m s, serão, respectivamente, iguais a
a) 3,4 e 10
b) 3,6 e 8,0
c) 3,6 e 10
d) 5,4 e 8,0
Conforme indica a figura acima, no instante
e) 5,4 e 10
t = 0, uma partícula é lançada no ar, e sua
Gabarito:

Resposta da questão 1:
a) Sejam A e B, respectivamente, pontos do topo do recipiente e da altura do furo. Aplicando a
equação de Bernoulli, obtemos a velocidade horizontal com a qual a água sai através do furo:
ρv A 2 ρ vB2 v 2
PA + ρ gH + = PB + ρ gh +  gH = gh + B  vB = 2g(H − h)
2 2 2
0

Equação da trajetória do filete de água:


 a t2  gt 2 2
 y = y0 + v 0y + y y = h − g x 
 2  2  y =h−  
2  2g(H − h) 
x = x + v t  x = 2g(H − h)t
 0 x 
x2
y = h−
4(H − h)

b) Da equação da trajetória, obtemos:


x2
y =h−  4yH − 4yh = 4Hh − 4h2 − x 2  4h2 − 4(y + H)h + 4yH + x 2 = 0
4(H − h)

Devemos ter Δ  0. Logo:


 −4(H + y)2 − 4  4  ( 4yH + x 2 )  0
( ) (
16 H2 + 2yH + y 2 − 16 4yH + x 2  0 )
H2 − 2yH + y 2 − x 2  0
( H − y )2 − x 2  0
H− y  x
y  H− x

Ou seja, o lugar geométrico procurado é dado pela região ilustrada abaixo:

Resposta da questão 2:
[B]

Temos as situações:
1) Colisão da bola de basquete com a parede:

2) Colisão entre as bolas:

Aplicando a conservação da quantidade de movimento e considerando a colisão elástica, obtemos:


Mv − mv = MV + mv ' ( 3M − m) v
  v' =
v '− V = 2v M+m

Tempo para a bolinha de gude sofrer uma queda de uma altura de h − r :


gt 2 2 (h − r )
h−r = t=
2 g

Para M  m, chegamos a:
( 3M − m) v 2 (h − r )
Δs = v ' t =
M+m g
2(h − r)
 Δs = 3v
g

Resposta da questão 3:
[C]

Sejam os pontos A e B destacados na figura:


Aplicando a equação de Bernoulli com v A  0, temos:
ρv A 2 ρv 2
PA + ρghA + = PB + ρghB + B
2 2
Mg ρv 2
Patm + + ρgH + 0 = Patm + ρgh + B
A 2
 M
vB = 2g  H − h + 
 ρA 

Tempo de queda:
gt q2 2h
h=  tq =
2 g

Portanto, o alcance é dado por:


 M  2h
D = vB  t q = 2g  H − h + 
 ρ A g

 m
D = 2 hH − h + 
 ρA 

Resposta da questão 4:
[C]

Após o lançamento, teremos:


Tempo de queda:
1 2 2h 2 1 1
h= gt  t = = t= s
2 g 10 5

Velocidade horizontal:
1
d = vt  5 = v  v=5 5 m s
5

Por conservação de energia para o lançamento, obtemos:

( )
2
kx 2 mv 2 k  0,12 0,2  5 5
= + mgh  = + 0,2  10  1 
2 2 2 2
 k  0,01 = 0,2  25  5 + 2  2  0,01k = 29
 k = 2900 N m
Resposta da questão 5:
No ponto mais baixo da trajetória, temos:
Fcp = Tmáx − P

mω2r = Tmáx − mg
1 ω2  1 = 46 − 1 10
 ω = 6 rad s

Velocidade horizontal:
v = ωr = 6  1  v = 6 m s

Tempo de queda após o lançamento horizontal:


gt 2 10t 2
Δy = v 0y +  6 −1=  t = 1s
2 2

Portanto, chegamos ao alcance:


A = v  t = 6 1
A = 6 m

Resposta da questão 6:
[A]

Sendo v a velocidade do projétil e v A a velocidade do conjunto corpo + projétil após a colisão, por
conservação da quantidade de movimento, temos:
v
mv = 10mv A  v A =
10

Por conservação de energia, a velocidade do conjunto no ponto B será:


EmA = EmB

10mv A 2 10mvB2 v2
= 10mgR +  vB = − 2gR
2 2 100

Para o lançamento horizontal, temos:


Em y :
1 2 2R
R= gt  t =
2 g

Em x :
v2 2R
R = vB t = − 2gR  
100 g
2v 2R 5gR
R2 = − 4R2  v 2 = 100 
100g 2
5gR
 v = 10
2

Resposta da questão 7:
[B]
Como o tempo de queda só depende do movimento vertical, tanto nas situações do ar em repouso
como na do ar em movimento, os tempos serão maiores do que t1 (para o vácuo), pois as partículas
sofrerão resistência ao caírem nessas circunstâncias, com t2 = t3 , já que o movimento do ar se dá
apenas horizontalmente.
Na primeira situação, o corpo irá adquirir velocidade máxima devido à ausência de resistência causada
pelo vácuo. Na segunda situação, o corpo irá sofrer atrito com o ar, tendo sua velocidade minimizada. E
na terceira situação, como o ar está em movimento, mas não se opondo à velocidade inicial, a partícula
terá velocidade intermediária.
Portanto, teremos que: t1  t 2 = t3 e v1  v3  v 2 .

Resposta da questão 8:
[C]

Do enunciado, temos o seguinte esquema:

Da figura, temos que:


1,8
tg30 =  a = 1,8 3 m
a

Tempo de queda até a superfície da água:


1 2 2h 2  1,8
h= gt  t = =  t = 0,6 s
2 g 10

Na horizontal, obtemos:
a + b = v x t  1,8 3 + b = 5 3  0,6  b = 1,2 3 m

Aplicando a lei de Snell, vem:


3 5 3 3 3
nar  sen60 = nágua  senr  1 =  senr  senr =  tgr =
2 6 5 4

Logo:
b 3 1,2 3
tgr =  =  h = 1,6 3 m
h 4 h
Resposta da questão 9:
[A]

Resposta da questão 10:


[A]

Tempo até alcançar a altura máxima:


v = v 0 + at
0 = v 0 sen α − gt
v sen α
t= 0
g

Tempo total:
2v sen α
t t = 2t = 0
g

Período do MCU descrito pela partícula devido ao campo magnético:


2 πm
T=
Bq

Logo:
t t = T + t1
2v 0 sen α 2πm
= + t1
g Bq
2 πm
B =
 2v sen α 
q 0 − t1 
 g 

Resposta da questão 11:


a) A trajetória em y não é alterada pelo campo magnético. Portanto:
gt 2 1 10t 2
y = h0 + v 0 sen θ t − = 280 + 100  t −
2 2 2
y = 280 + 50t − 5t 2

Altura máxima:
502 − 4  ( −5 )  280 
=− y
Δ
ymáx = − máx = 405 m
4a 4  ( −5 )

Instante de retorno a y = 0 :
280 + 50t f − 5t f 2 = 0  t f = 14 s

Alturas de A e C :
y A = 280 + 50  9 − 5  92  y A = 325 m
yC = 280 + 50  12 − 5  122  yC = 160 m
b) Raio da trajetória da partícula no plano xz :
mv x m  v 0 cos α 9v 0 cos α
R= = =
Bq mπ π
q
9q
9  100 3 450 3
R=  R= m
π 2 π

Período da trajetória:
2πm 2πm
T= =  T = 18 s
Bq mπ
q
9q

De A até C :
 T
Δt = 12 s − 9 s = 3 s  Δt = 
 6

Logo, o ângulo percorrido é de:


360
θ= = 60
6

Coordenada x A :
3
x A = v 0 cos α t A = 100   9  x A = 450 3 m
2

Desenhando a vista de cima, temos:


Sendo assim:
450 3 3
xC = x A + R sen60 = 450 3 + 
π 2
 xC =
225
π
(
2π 3 + 3 m)
225 3  450 3 450 3 1 
zC = z0 − (R − R cos 60 ) = − −  
π  π π 2 

 zC = 0 m

Resposta da questão 12:

 vBx = vB cos θ

Na figura: 
 vBy = vB sen θ

a) Usando a conservação da energia mecânica entre os pontos A e B, calculamos a velocidade em B:


A k x2 m vB2
Emec = EB
mec  + m g hA = + mg hB 
2 2

k x2 m vB2
+m gh = + m g (h − hcos θ ) 
2 2

k x2
vB2 = + 2 g hcos θ.
m

A partir do ponto B, temos um lançamento oblíquo com altura máxima hf em relação ao ponto de
lançamento.

( ) v 2 sen2θ
2
v 2y = vBy
2
− 2 g hf  0 = vB sen θ − 2 g hf  hf = B 
2g
 k x2  sen2θ  k x2 
hf =  + 2 g hcos θ   hf =  + hcos θ  sen2θ.
 m  2g  2mg 
   

Da figura:
 k x2 
Hf = hf + h (1 − cos θ)  Hf =  + hcos θ  sen2θ + h (1 − cos θ)
 2mg 
 
k x2
Hf = sen2θ + hcos θ sen2θ + h (1 − cos θ) 
2mg

Hf =
k x2
2mg  ( )
sen2θ + h cos θ 1 − cos2 θ + 1 − cos θ 


Hf =
k x 2 sen2θ
2mg
(
+ h 1 − cos3 θ . )
b) O movimento na superfície plana horizontal tem velocidade inicial vC = vBx = vB cosθ e velocidade
final vD = 0.
Entre os pontos C e D a força resultante é a força de atrito. Aplicando o teorema da energia cinética
entre esses pontos:

2 2
m vD m vC
WFat = ED C
cin − Ecin  − Fat d = − 
2 2
m ( vB cos θ )
2
1
−μ mg d = 0 −  d= vB2 cos2 θ 
2 2μ g

1  k x2 
d=  + 2 g hcos θ  cos2 θ 
2μ g  m 

 k x2 hcos θ 
d=  +  cos2 θ.
 2 μ mg μ 
 

Resposta da questão 13:


[A]

Num lançamento oblíquo sobre superfície plana e horizontal, sendo desprezível a resistência do ar, o
tempo total de movimento (tT) e a altura máxima atingida (H), dependem somente da componente
vertical de velocidade (v0y = v0 sen ). Como se pode demonstrar:

 2 v 0y 2 v 0 sen α
t T = = .
 g g


2
v 0y v 02 sen2α
H = = .
 2g 2g

Numa situação hipotética, se, ao longo do movimento, somente houvesse redução na componente
horizontal da velocidade (vx), seria alterado apenas o alcance horizontal (A), como indicado na figura.
No caso dessa questão, como a parede é lisa, não ocorre alteração na componente vertical da
velocidade, portanto o tempo total de movimento é igual ao tempo total que seria gasto se não
houvesse o choque. O tempo t1 do lançamento até o choque é maior que o tempo t2 do choque ao
retorno ao solo, pois o choque ocorre antes do ponto mais alto da trajetória.

 d
d = v x t1  t1 =
 vx

 ' d d d 2 v 0 sen α d 2 v 0 sen α d
d = v x t 2  t 2 = '  +
'
=  = − 
 vx vx vx g v 'x g v 0 cos α

t1 + t 2 = t T = 2 v 0 sen α

 g

d 2 v 02 senα cos α − g d d v 2 sen 2α − g d d g v 0 cos α


=  = 0  v 'x =
v 'x g v 0 cos α '
vx g v 0 cos α 2
v 0 sen 2α − g d

O coeficiente de restituição (e) é definido como:

g d v0 cos α
v afast v' v 02 sen 2 α − g d
e= = x  e= 
v aprox vx v0 cos α
gd
e= .
v 02 sen 2 α − g d
Resposta da questão 14:
[B]

A figura a seguir ilustra a situação.

– O tempo de subida (tS) é dado por:


 v sen
 T1 = 0 (I);
v 0y  g
tS =  
g T = v 0 sen (II).


2
g
– A função horária para eixo das abscissas é: x = vox t

Nos instantes t1 e t2 as abscissas são iguais. Então:


cos 
vo cos  t1 = v0 cos  t2  t 2 = t1 (III) .
cos 
g 2
– A função horária para eixo das ordenadas é: y = v oy t − t
2
Nos instantes t1 e t2 as ordenadas são iguais. Então:
g g
v0 sen t1 − t12 = v 0 sen t 2 − t 22 (IV)
2 2

– Substituindo (III) em (IV), vem:


g cos  g cos2  2
v 0 sen t1 − t12 = v 0 sen t1 − t1 
2 cos  2 cos2 
g  cos2    sen cos  
t1  −1 +  = v 0  − sen  
2   cos  
2
 cos  
g  cos2  − cos2    sen cos  − sen cos  
t1 
  = v 0   
2  cos 
2
  cos  

2v 0 cos   sen (  −  ) 
t1 =  2  (V) .
 cos  − cos  
g 2

– Fazendo (I)  (V), vem:


 v sen   2v 0 cos   sen (  −  )  
T1 t1 =  0    2  
 cos  − cos   
2
 g   g
2v 02 sen( −  ) sen cos 
T1 t1 = (VI).
g2 cos2  − cos2 
– Substituindo (V) em (III), temos:
 cos    2v 0 cos   sen (  −  )  
t2 =    2   
 cos     cos  − cos   
g 2

2v cos   sen (  −  ) 
t2 = 0  2 
 (VII)
.
 cos  − cos  
g 2

– Fazendo (II)  (VII), vem:


 v sen   2v 0 cos   sen (  −  )  
T2 t2 =  0    2  
 cos  − cos   
2
 g   g
2v 02 sen( −  ) sen cos 
T2 t2 = (VIII)
g2 cos2  − cos2 

– Fazendo (VI) + (VIII):


2v 2 sen (  −  )
T1 t1 + T2 t2 = 20 ( sen cos  + sen cos  ) 
g cos2  − cos2 
2v 02 sen cos  − sen cos 
T1 t1 + T2 t2 = ( sen cos  + sen cos  ) =
g2 cos2  − cos2 
2v 02 ( sen cos  sen cos  ) + sen2 cos2  − sen2  cos 2  − ( sen cos  sen cos  )

g2 cos2  − cos2 
2v 02 sen2 cos2  − sen2  cos2 
T1 t1 + T2 t2 = 
g2 cos2  − cos2 

T1 t1 + T2 t2 =
2v 02 (1− cos ) cos
2 2
(
 − 1 − cos2  cos2  ) 
g 2
cos  − cos 
2 2

T1 t1 + T2 t2 =
2v 02 (cos2
 − cos2  ) + cos2  cos2  − cos2  cos2 

g2 (cos 2
 − cos2  )
2v 02
T1 t1 + T2 t2 = .
g2

Resposta da questão 15:

O pequeno bloco parte do repouso do ponto A, na altura h, e atinge o ponto B com velocidade v, na
altura R + R cos φ.
Assim, pela conservação da energia mecânica:
mv 2
A
Emec = EB
mec  mgh = + mg (R + R cos φ ) 
2
v2
+ gR(1 + cos φ)
v 2 Rg(1 + cos φ)
h= 2 h= +  Para continuar em contato com o restante da
g 2g g
1 2
h= v + R(1 + cos φ) (equação 1)
2g
pista, o pequeno bloco deve realizar um lançamento oblíquo, descrevendo o arco de parábola BC. Como
mostra a figura acima, o alcance horizontal desse lançamento é:
D = 2 R sen φ (equação 2)

Mas o alcance horizontal de um lançamento oblíquo com velocidade de lançamento v é calculado por:
2v 2
D= sen φ cos φ (equação 3)
g

Igualando as equações (2) e (3), temos:


2v 2 Rg
sen φ cos φ = 2 R sen φ  v 2 = . Substituindo essa expressão na equação (1), vem:
g cos φ
1  Rg  R
h=   + R (1 + cos φ )  h = + R (1 + cos φ ) 
2g  cos φ  2cos φ
R + 2 R cos φ(1 + cos φ)
h= 
2 cos φ
R 1 + 2cos φ (1 + cos φ ) 
h= .
2cos φ

Resposta da questão 16:


[B]

Dados: k d = 2 km ; Fd = Fm.

Calculando a razão entre as deformações:


Fd = Fm  k d x d = k m x m  2 k m x d = k m x m  x m = 2 x d

Comparando as energias potenciais elásticas armazenadas nos dois estilingues:


 k x2 2
Epot = d d = 2 km x d = km x 2d
 d 2 2 pot
  Epot
m = 2 Ed
km (2 x d )
2 2
 pot km xm 4 k m x 2d
Em = = = = 2 km x 2d
 2 2 2

Considerando o sistema conservativo, toda essa energia potencial é transformada em cinética para o
objeto lançado. Assim:
cin
2
m vm m v 2d
Em = 2 Ecin
d  =2 2
 vm = 2v 2d
2 2

Supondo lançamentos oblíquos, sendo θ o ângulo com a direção horizontal, o alcance horizontal (D) é
dado pela expressão:
 v2
Dd = d sen (2 θ)
v2  g Dd 1
D = 0 sen (2 θ)    = .
g  2 v 2d Dm 2
D
 m = sen (2 θ)
 g

Resposta da questão 17:


[B]

Altura máxima:
v y 2 = v 0y 2 + 2aΔs y

0 = ( v 0 sen θ) − 2ghmáx
2

v 2 sen2 θ
hmáx = 0
2g

Tempo para que a altura máxima seja atingida:


v y = v 0y + at
0 = v 0 sen θ − gt
v 0 sen θ
t=
g

Distância horizontal percorrida até a partícula atingir a altura máxima:


Δs x = v x t
v 0 sen θ
d = v 0 cos θ 
g
v 02 sen θ cos θ
d=
g

Logo:
v 02 sen2 θ
h 2g
tg α = máx =
d v 02 sen θ cos θ
g
sen θ tg θ
tg α = =
2cos θ 2
tg θ
 =2
tg α

Resposta da questão 18:


[D]

Da situação representada abaixo, temos:


M
V1 V V
sen α = 4 = 1  V2 sen α = 1
3M 3V2 3
V2
4

Por conservação de energia, obtemos o valor da altura máxima:


M ( V0 sen θ)
2
V 2 sen2 θ
= M ghmáx  hmáx = 0
2 2g

Logo, o tempo procurado é dado por:


gt 2
hmáx = − V2 sen αt +
2
V02 sen2 θ V gt 2
= − 1t+
2g 3 2
gt 2 V1 V 2 sen2 θ
− t− 0 =0
2 3 2g

g  V sen θ 
2 2 2
V1  V 
 − 1  − 4 − 0 
3  3  2  2g 

t=
g
2
2
V1 V12
 + V02 sen2 θ
t= 3 9
g

Como o tempo deve ser positivo, chegamos a:


V V12 
t= 1+ + V02 sen2 θ  g
3 9 
 

Resposta da questão 19:


[D]
Tempo de queda do 1º para o 2º degrau:
1 2 2h
h= gt1  t1 =
2 g

Alcance horizontal no 2º degrau:


2h
x = v 0 t1 = v 0
g

Tempo até a esfera atingir a extremidade do 2º degrau após a sua queda:


2h
D − v0
D−x g D g − v 0 2h
v0 =  t2 = =
t2 v0 v0 g

Tempo de queda do 2º para o 3º degrau:


1 2 h
2h = gt3  t3 = 2
2 g

Alcance horizontal no 3º degrau:


h
L = v0 t3 = 2v0  L g − 2v0 h = 0
g

Como esta expressão é nula, podemos somá-la ao numerador da equação de t 2 , chegando a:


D g − v 0 2h + L g − 2v 0 h
t2 =
v0 g

 t2 =
g (D + L ) − v 0 h ( 2 +2 )
v0 g

Resposta da questão 20:


[A]

Sendo v 0 e θ, respectivamente, a velocidade e o ângulo de lançamento, temos:

Tempo de subida do projétil até a sua altura máxima:


v y = v 0y + at
0 = v 0 sen θ − gt s
v sen θ
ts = 0
g

Tempo total até o projétil retornar ao solo:


2v sen θ
t t = 2t s = 0
g

Altura máxima atingida pelo projétil:


v y 2 = v 0y 2 + 2aΔs

0 = ( v 0 sen θ) − 2gh
2

v 2 sen2 θ
h= 0
2g
Aceleração horizontal do projétil:
P
Fr = −
4
mg
ma = −
4
g
a=−
4

Alcance horizontal do projétil:


tt2
Δs x = v 0x t s + a
2
2
2v 0 sen θ g 1  2v 0 sen θ 
A = v 0 cos θ  −   
g 4 2 g 
2v 02 sen θ cos θ g 4v 02 sen2 θ
A= − 
g 8 g2
2v 02 sen θ cos θ v 02 sen2 θ
A= −
g 2g

Como h = A, chegamos a:
v 02 sen2 θ 2v 02 sen θ cos θ v 02 sen2 θ
= −
2g g 2g
v 02 sen 2 θ 2 v 02 senθ cos θ
=
g g
sen θ
= tg θ = 2
cos θ
 θ = arctg(2)

Resposta da questão 21:


[D]

Relação entre as velocidades de lançamento:


 2 πR A
v A = T

 2 v R R
  A = A = A  vB = 4v A
v = 2 π RB v B R B 4R A
 B T

 2

Como o tempo de queda é o mesmo, temos que:


 XA
v A = Δt v X v X
  A = A  A = A
v = XB vB XB 4v A XB
 B Δt
XB
 =4
XA

Resposta da questão 22:


[D]
Velocidade do sistema partícula + bloco após a colisão inelástica:
v 3
Qantes = Qdepois  M v 0 cos30 = 6 M v  v = 0
12

Altura atingida pelo sistema em relação ao ponto de colisão:

1
h = 1,2 − 1,2cos60º = 1,2 − 1,2   h = 0,6 m
2

Por conservação de energia, obtemos:


2
6M v 2 1 v 3 
Ec = Ep  = 6M gh   0  = 10  0,6 
2 2  12 

v 02  3
 = 2  6  v 0 = 4  144  v 0 = 2  12
144
 v 0 = 24 m s

Resposta da questão 23:


[E]

Após o lançamento horizontal, temos:


1 1
Em y : h = gt 2  1,25 =  10  t 2  t = 0,5 s (tempo de queda)
2 2

Em x : d = vt  5 = v  0,5  v = 10 m s (velocidade horizontal da esfera)

Desprezando o atrito com a mesa, por conservação da energia mecânica:


kx 2 mv 2
=
2 2
k  0,12 = 0,05  102
 k = 500 N
m

Resposta da questão 24:


[C]

Cálculo do tempo de queda:


gt 2 2h 2 ( 2R ) R
h= t= = t=2 .
2 g g g

Após a ruptura da corda, na direção horizontal o movimento é uniforme. A velocidade inicial do


lançamento é:
 R 2 2 R
D = v t  4R = v  2   16R = v 4  v 2 = 4R g.
 g g
 

Se a partícula é lançada horizontalmente, a corda se rompe no ponto mais alto. Imediatamente antes da
ruptura, a força resultante centrípeta tem intensidade igual à soma das intensidades do peso e da
tração.
mv 2 m ( 4R g)
T + P = Fcent  T + mg =  T= − mg  T = 3mg.
R R

Resposta da questão 25:


[B]

Pela figura, temos que:


Fcp = Pcosθ

mv 2
= mgcosθ  v 2 = Rgcosθ (I)
R

Por conservação de energia entre A e C :


EA = EC
mv 2
mgR(1 − cosθ) = (II)
2

Substituindo (I) em (II):


mRgcosθ
mgR(1 − cosθ) =
2
2(1 − cosθ) = cosθ  cosθ = 2 (III)
3

No lançamento oblíquo:
v x = v cosθ

v y = vsenθ

Em x : d = v x Δt = v cosθΔt (IV)

Substituindo (I) e (III) em (IV):


d = Rgcosθ  cosθ  Δt

102 = 135  10  2  2  Δt
3 3
 Δt = 5,1 s

Resposta da questão 26:


[B]

Nesta questão, temos uma composição de movimentos, pois se trata de um lançamento oblíquo em que
devido ao campo elétrico surge uma força elétrica de mesma direção e sentido da força gravitacional
atuando na vertical para baixo. Assim, temos uma aceleração resultante obtida pela soma da aceleração
da gravidade com a aceleração elétrica que aponta no mesmo sentido que a força elétrica, cujo módulo
é:
qE
a = g+
m

O movimento da partícula representa uma parábola com a concavidade voltada para baixo, mas
precisamos de uma função que relacione a variação da energia potencial gravitacional com o eixo x.

Decompondo a velocidade inicial v 0 nos eixos x e y:


v0x = v0  cosθ
v0y = v0  sen θ

Para o eixo x, temos um MRU, sendo a equação dada por:


x = x0 + v0x t  x = v0  t cosθ

Isolando t:
x
t= (1)
v 0  cosθ

Para o eixo y, temos um MRUV, sendo a equação da posição vertical com o tempo dada por:
a
y = y0 + v0y t − t 2
2
a
Δy = v0  sen θ  t − t 2 (2)
2

Substituindo a equação (1) na equação (2):


a x2
Δy = t g θ  x − (3)
2 v 02  cos2 θ

E, como a variação da energia potencial gravitacional é dada por:


qE
ΔEp = m  a  Δy onde a = g +
m

Substituindo Δy finalmente ficamos com:


 a x2 
ΔEp = m  a   t g θ  x − 
 2 2
 2 
θ
 v 0 cos 
m  a2 x2
ΔEp = m  a  t g θ  x −
2 v02  cos2 θ

Logo, temos uma equação representativa de uma parábola cuja concavidade está voltada para baixo.
Alternativa [B].

Resposta da questão 27:


[D]

Adotando A e B para os pontos iniciais de lançamento dos dois corpos:

Usando a expressão para a Energia cinética e a equação da velocidade linear em função da velocidade
angular
m
Ec = v 2 e v = ωR
2

Do gráfico, para o corpo 1 em A, temos:


Ec1(A) = 2 J
m1
( ω12R )2 = 2
2

Substituindo a massa do corpo 1, temos:


( ω1R )2 = 1
1
ω1 = (1)
R
Para o corpo 2, no ponto P, do gráfico retiramos:
Ec = 416 J
2(P)

Neste ponto, o corpo 2 desenvolveu uma velocidade vertical além de manter a velocidade horizontal do
lançamento, portanto temos uma composição de velocidades.

- Na horizontal: v x = v 2 = ω2R
- Na vertical: v y = 2g ( h − y ) da equação de Torricelli

Logo, para o corpo 2 no ponto P a sua energia cinética será:


m
(
Ec2(P) = 2 v x2 + v y2 = 416 J
2
)
Substituindo os dados e as componentes da velocidade no ponto P
2
( )
2
 ( ω2R ) + 2g ( h − y )  = 416 J
2
2 
( ω2R )2 = 416 − 400
ω2R = 16
4
ω2 = (2)
R

Finalmente, dividindo a equação (2) pela equação (1) chegamos a


4
ω2 R ω
=  2 =4
ω1 1 ω1
R

Resposta da questão 28:


[E]

( )
Na expressão dada, r(t) = ( 6,0 t + 2,5 ) i + −5,0 t 2 + 2,0 t + 8,4 j, tem-se:

 x(t) = ( 6,0 t + 2,5 ) i





 (
y(t) = −5,0 t 2 + 2,0 t + 8,4 j )
A altura (h) no instante t = 1s corresponde à ordenada (y) nesse instante:

( )
h = y(1) = −5,0 (1) + 2,0 (1) + 8,4 = −5 + 10,4 
2
h = 5,4m.

Calculando o módulo da velocidade:


dr
v(t) = = ( 6,0 ) i + ( −10,0 t + 2,0 ) ˆj 
dt
v(1) = ( 6,0 ) i + ( −10,0 (1) + 2,0 ) ˆj  v(1) = 6 ˆi − 8 ˆj.
2
v = 62 + 82  v = 10m s.
MOMENTO DE INÉRCIA OU INÉRCIA
ROTACIONAL

Momento de inércia de um corpo é uma medida de


sua capacidade de resistir a mudanças no estado de
rotação. Ela desempenha o mesmo papel no
movimento rotacional, qualquer que seja a inércia que
desempenha no movimento translacional.

Escolha um pequeno elemento de massa dm a uma


distância r do eixo, seu momento de inércia em
relação ao eixo de rotação (eixo z) dI = dm.r2 . O
momento de inércia de todo o corpo pode ser obtido
por

Matematicamente o momento de inércia de uma 𝐼 = ∫ 𝑑𝑚. 𝑟 2


partícula de massa m em torno de um eixo é definido
como:
Os limites de integração dependem da forma do
𝐼 = 𝑚. 𝑟 2 corpo. O momento de inércia de um corpo depende
dos seguintes fatores:
onde r é a distância da partícula ao eixo de rotação. É
uma grandeza escalar particularmente. Sua unidade (i) Massa do corpo.
SI é kg.m².
(ii) Tamanho e forma do corpo.
MOMENTO DE INÉRCIA DO SISTEMA DE
(iii) Posição e orientação do eixo de rotação.
PARTÍCULAS
1.EXEMPLO: Momento de inércia do anel.
Considere um sistema de n partículas como mostrado
na figura.

Considere um anel de massa M e raio R, tome um


pequeno elemento, a sua massa
Seu momento de inércia é dado por 𝑀
𝑑𝑚 = 𝑑𝑙
2𝜋. 𝑅
𝐼 = 𝑚1 𝑟12 + 𝑚2 𝑟22 + ⋯ + 𝑚𝑛 𝑟𝑛2 Momento de inércia deste elemento em relação ao
eixo mostrado na Figura
𝑛
𝑑𝐼 = 𝑑𝑚. 𝑟 2
∑ 𝑚𝑖 . 𝑟𝑖2
1

MOMENTO DE INÉRCIA DE UM CORPO RÍGIDO


Considere um corpo girando em torno de um eixo
conforme mostrado na Figura.
2.Exemplo: Momento de inércia do disco

Considerar um disco de massa M e raio R rodando


sobre o seu eixo geométrico.
Escolha um elemento de raio r e largura dr, a sua
massa Momento de inércia de uma haste ou cilindro
espesso
II)

Momento de inércia do elemento do anel sobre o


eixo mostrado
𝑑𝐼 = 𝑑𝑚. 𝑟 2

Escolher um disco de elemento de espessura dx a


uma distância x do eixo de rotação.
A massa do elemento

O M.I. do disco elementar em torno do eixo de


rotação dI = M.I. do disco sobre o diâmetro +
(dm)x2

3. Exemplo: Momento de inércia de uma haste


ou cilindro espesso
I)

Considere um cilindro circular sólido de massa M e


raio de secção transversal R. O comprimento do
cilindro é L.
Escolha um elemento do cilindro na forma de um
tubo de comprimento L e raio r e espessura dr.
A massa do elemento

4. Exemplo: Momento de inércia da esfera


Considerar uma esfera de massa M e raio R em
rotação sobre qualquer eixo do seu diâmetro.
Escolher um elemento na forma de um disco de
raio r e espessura dx. A massa do elemento Observação: Uma maneira de entender a razão
de o cilindro ter momento de inércia maior do que
a esfera, embora ambos tenham a mesma massa
e o mesmo raio, é prestando atenção na
distribuição de massa desses sólidos geométricos.
Observe a figura a seguir e note que a região
sombreada mostra a parcela da massa do cilindro
mais afastada do eixo de rotação em relação à
massa da esfera – essa parcela dá ao cilindro sua
maior inércia rotacional. O significado físico dessa
diferença de valores inerciais é fundamental para o
estudo da dinâmica das rotações.
Costuma-se dizer que a massa é a medida da
inércia de um corpo, mas, como se vê, essa é uma
afirmação válida apenas para movimentos de
translação. Em rotação; a medida da inércia de um
corpo é o seu momento de inércia.

Energia cinética de rotação e momento de inércia


A figura a seguir ilustra uma situação hipotética que
vai nos ajudar a entender algumas diferenças
importantes entre os movimentos de translação e
5. Exemplo: Uma esfera e um cilindro, maciços, rotação:
têm a mesma massa, m = 0,40 kg, e o mesmo raio,
r = 0,10 m. Determine o momento de inércia
desses corpos em relação ao eixo z. Veja a figura.

No trecho I a esfera rígida de massa m se desloca pelo


plano horizontal, inicialmente sem atrito (por isso não
gira), com velocidade v, representada no seu centro
de massa - ela tem apenas movimento de translação;
no trecho II, onde há atrito, a esfera passa também a
girar com velocidade angular de módulo ω - ela tem 𝑚 𝑣2
partícula, 𝐸𝑐𝑖 = 𝑖 𝑖 escrita em função do módulo da
movimento de rotação e de translação (agora com 2
velocidade v'). Em IlI, um obstáculo impede a esfera velocidade angular ω, é:
de avançar, mas, como nessa região não há atrito, ela 𝑚𝑖 𝛚 2 𝑟𝑖2
𝐸𝑐𝑖 =
se mantém girando com velocidade angular de 2
módulo ω’ - agora ela tem apenas movimento de O somatório de todas as energias cinéticas de
rotação. translação de todas as partículas é, portanto:
De início, vamos analisar o que ocorre com a energia 𝑚𝑖 𝛚 2 𝑟𝑖2
𝐸𝑐 = ∑
cinética da esfera. Em I, ela se deve apenas à velo- 2
cidade e, como sabemos, seu valor é dado por [∑ 𝑚𝑖 𝑟𝑖2 ]. 𝜔2
𝑚.𝑣 2 𝐸𝑐 =
𝐸𝑐 = (de agora em diante, vamos chamar essa 2
2 Para comparar essa expressão com a expressão da
energia cinética de energia cinética de translação). energia cinética de um corpo em translação, vamos
Em II, além da velocidade e da energia cinética de reescrevê-la destacando a massa m desse corpo:
translação correspondente, a esfera tem velocidade 𝑚𝑣 2
angular de módulo ω cinética de rotação-e passou a 𝐸𝑐 =
ter também energia cinética de rotação. 2
Como m, massa do corpo, mede a inércia desse corpo
A necessidade de se definir uma energia cinética
em translação, pode-se concluir que o termo ∑ 𝑚𝑖 𝑟𝑖2
especifica para a rotação de um corpo rígido fica
mede a inércia do sistema rígido em rotação.
evidente em III, pois, nessa situação, embora a esfera
Essas considerações nos levam a definir uma nova
não se desloque mais e, por consequência, não
grandeza, o momento de inércia, 1, do sistema rígido
apresente energia cinética de translação porque é um
em rotação, dada pela expressão:
corpo rígido único, todas as partículas que a
constituem (com exceção daquela que pode ser 𝐼 = ∑ 𝑚𝑖 𝑟𝑖2
considerada localizada no seu centro) têm velocidade As expressões da energia cinética de translação, ECT
e, portanto, energia cinética de translação. É a soma e de rotação, ECR, são conceitualmente idênticas, mas
das energias cinéticas de translação de todas as têm formas diferentes e se relacionam a grandezas
partículas que constituem a esfera que resultam na diferentes, por isso passaremos a representá-las na
sua energia cinética de rotação.
forma:
Podemos obter a expressão matemática da energia
cinética de rotação de um corpo rígido de forma 𝑚. 𝑣 2 𝐼. 𝜔2
𝐸𝑐𝑇 = 𝑒 𝐸𝑐𝑅 =
simplificada, considerando-o como um sistema rígido 2 2
de partículas em rotação, como mostra a figura a Tendo em vista que todo corpo rígido é, em última
seguir. análise, um sistema rígido de partículas, as definições
de momento de inércia e de energia cinética de
rotação aqui apresentadas valem também para todo
corpo rígido.

Teorema dos eixos paralelos


Este teorema é bem útil para simplificar o cálculo do
momento de inércia. Considere um corpo extenso cujo
momento de inércia em relação a certo eixo é 𝐼 e seja
𝐼𝑐𝑚 o momento de inércia em relação a um eixo
paralelo, que passa pelo centro de massa, conforme
figura abaixo:

Em relação a esse sistema, vamos considerar três


condições iniciais:
• ele é formado por partículas idênticas de
massa mi;
• por ser rígido, todas as suas partículas
giram com a mesma velocidade angular ω;
• cada partícula gira em torno de um eixo O
com movimento circular de raio ri e O teorema dos eixos paralelos diz que:
velocidade vi, de módulo vi = ω ri.
Como a energia cinética é uma grandeza escalar e
sempre positiva, podemos somar as energias Para provar este teorema, vamos utilizar o conceito
cinéticas de translação de cada uma das partículas de energia cinética. Podemos dizer que a
que compõem esse sistema em rotação. Assim, energia cinética do corpo é dada por:
sendo o módulo da velocidade de cada partícula dado
por vi = ω ri, a energia cinética de translação de cada
Em que 𝐼 é o momento de inércia em relação ao eixo
fixo. Por outro lado, podemos dizer que a energia
cinética é a energia cinética de translação(𝑀 ⋅ 𝑣2𝑐𝑚)/2
mais a energia cinética de rotação (𝐼𝑐𝑚 ⋅ 𝜔2)/2 em Note que 𝑟 ⋅ 𝐹𝑡 é o módulo do torque 𝜏 em relação ao
relação ao centro de massa. eixo de rotação associado à força. Ou seja:

Como já mencionamos anteriormente, a força


resultante que atua sobre um sistema de partículas é
igual à força externa resultante que age sobre o
sistema, já que as forças internas se anulam aos
pares.
O efeito dos torques internos exercidos entre as
partículas de um sistema conduz a um resultado
análogo, ou seja, o torque resultante que atua sobre
um sistema é igual ao torque externo resultante que
age sobre o sistema. Em outras palavras:
Note que o centro de massa se move ao longo de uma
trajetória circular de raio ℎ, de forma que é valido
𝑣𝑐𝑚=ℎ ⋅ 𝜔. Portanto:
Note que essa equação é o análogo rotacional da
segunda lei de Newton para o movimento de
translação,

CONSERVAÇÃO DO MOMENTO ANGULAR


Segunda lei de Newton para a rotação Se o momento (torque) das forças que atuam num
Considere uma partícula de massa 𝑚 presa a uma corpo em rotação é nulo, então o momento angular
das extremidades de uma haste rígida, sem massa, permanece constante.
de comprimento 𝑟. A haste pode girar livremente em Nessas condições, resulta em módulo:
torno de um eixo fixo que passa perpendicularmente L = lω = constante
por sua extremidade em 𝐴. Por construção, a Se o corpo for deformável, sendo L = lω constante,
partícula pode se movem apenas em um círculo de vem: se I aumenta, ω diminui e, se I diminui, ω
raio 𝑟. Se uma única força 𝐹 é aplicada sobre a aumenta.
partícula, como mostra a figura abaixo, então É o caso da bailarina girando em torno de seu eixo
podemos aplicar a segunda lei de Newton para a vertical de rotação r com os braços estendidos e com
partícula apenas nas componentes tangenciais: velocidade angular ω1, sendo l1 seu momento de
inércia em relação ao eixo r. Fechando os braços, o
momento de inércia diminui para l2 (l2 < l1) e sua
velocidade angular passa a ser ω2. Como l1ω1 = l2ω2,
resulta ω2 > ω1

𝐹𝑡 = 𝑚 ⋅ 𝑎𝑡
Em que 𝐹𝑡 = 𝐹 ⋅ 𝑠𝑒𝑛 𝜙 é a componente tangencial da
força 𝐹 e 𝑎𝑡 é a componente tangencial da
aceleração. Algebricamente, podemos reescrever a
segunda lei de Newton na direção tangencial para as
grandezas angulares:
Vejamos algumas situações envolvendo a
conservação do momento angular.

6. Exemplo: Cadeira giratória


O jovem da figura encontra-se sentado numa
cadeira giratória, sem encostar os pés no chão e
com os braços estendidos. Uma outra pessoa gira
a cadeira em torno do eixo vertical r. A seguir, o 2. Uma partícula de massa m é liberada do ponto P
jovem fecha os braços. O momento de inércia do em x = x0 no eixo x a partir da origem O e cai
sistema, em relação ao eixo r, diminui; verticalmente ao longo do eixo y, como mostrado na
consequentemente, ele passa a girar mais figura.
depressa, isto é, sua velocidade angular aumenta.
O efeito observado é mais acentuado quando o
jovem segura um par de halteres.

A) Determine o torque atuando sobre a partícula em


um momento t quando ela está no ponto Q com
respeito a O.

B) Encontre o momento angular L da partícula relativo


7. Exemplo: Cadeira giratória e roda de bicicleta a O no momento t.

C) Mostrar que

3. (Solved Problems in Physics) Um míssil é disparado


do solo com velocidade inicial v0, formando um ângulo
α com a vertical como mostra a figura abaixo. Se o
míssil deve atingir uma altitude máxima igual ao raio
Considere uma cadeira que pode girar em torno de da Terra, o professor Laerte Pereira pede que você
seu eixo vertical r, praticamente sem atrito. Uma demonstre que o ângulo necessário é definido pela
pessoa encontra-se sentada na cadeira, sem relação
encostar os pés no chão e segurando o eixo de
uma roda de bicicleta. A roda, com seu eixo
disposto horizontalmente, é colocada em rotação
(figura 8a). A componente vertical do momento onde é a velocidade de escape.
angular do sistema é nula. Como o torque externo
vertical é nulo, há conservação da componente 4. Uma nave espacial tripulada por marcianos chega
vertical do momento angular, isto é, a componente à vizinhança da Terra (de massa M) seguindo uma
vertical do momento angular permanece nula. órbita hiperbólica cuja assíntota dista b do centro da
Por outro lado, se a pessoa mantiver o eixo da roda Terra. Quando a nave se encontrava a uma distância
na vertical, com a roda girando num certo sentido, muito grande da Terra, sua velocidade era Vo. Qual a
a cadeira passa a girar em sentido oposto: os relação entre Vo, b e a distância de perigeu a?
momentos angulares ⃗⃗⃗⃗𝐿 e -𝐿⃗⃗ se anula

Exercícios
1. Considere um satélite de massa m em órbita em
torno do planeta de massa M como esquematizado na
figura abaixo, sujeito apenas a força de interação
gravitacional. Mostre que o momento angular será
constante durante o movimento do satélite.
5. Considere um disco de massa M e raio R está b) as tensões T e T' nos fios ligados a m e m'.
girando com velocidade angular 𝜔 sobre seu eixo
geométrico, como mostrado na figura. 8. Prende-se ao teto a ponta de uma fita métrica leve,
enrolada num estojo circular de massa m e raio r, e
solta-se o estojo em repouso (Figura), calcule a
aceleração linear do estojo.

Um pequeno objeto de massa m cai suavemente


sobre a borda do disco e aderi a ele. Determine então 9. Uma roda cilíndrica homogênea, de raio Re massa
a velocidade angular do disco. M, rola sem deslizar sobre um plano horizontal,
deslocando-se com velocidade v, e sobe sobre um
6. Uma corda flexível é enrolada várias vezes em plano inclinado de inclinação e, continuando a rolar
torno de um sólido cilindro de massa M e raio R, que sem deslizamento (Fig.). Até que altura h o centro da
gira sem fricção, sobre um eixo horizontal fixo, como roda subirá sobre o plano inclinado?
mostrado na Figura. A extremidade livre de a corda é 𝑀𝑅 2
amarrada a uma massa m que é solta do repouso a Considere 𝐼 = .
2
uma distância h acima do chão. Encontre a sua
velocidade tal como a massa m atinge o chão.
𝑀𝑅 2
Considere 𝐼 = .
2

10. Encontre a aceleração de m1 e m2 em uma


máquina de Atwood se houver atrito entre a superfície
da polia e a rosca e a rosca não deslizar sobre a
superfície da polia. O momento de inércia da polia é I.

7. (Moysés) Calcule o efeito da massa M da polia, de


raio R, sobre o sistema (Figura): a massa m, que
desliza sem atrito, está ligada a massa suspensa m'
pelo fio que passa sobre a polia.
𝑀𝑅 2
Considere 𝐼 = .
2
11. Um disco uniforme de raio R e massa M é
montado em um eixo apoiado em suporte fixo sem
atrito. O cordão leve (ideal) é enrolado ao redor do aro
do disco e suponha que penduramos um corpo de
massa m no cordão. Encontre a aceleração angular
do disco e a aceleração tangencial de um ponto na
borda.

Determine:
a) a aceleração a do sistema;
15. Um ioiô primitivo é feito enrolando uma corda
várias vezes em torno de um cilindro sólido com
12. Um cilindro uniforme de raio R é girado em torno massa M e raio R mostrado na figura. A extremidade
de seu eixo até a velocidade angular 𝜔0 e então da corda estacionária enquanto libera o cilindro sem
colocado em um canto conforme mostrado na figura. movimento inicial. A corda se desenrola, mas não
escorrega ou estica quando o cilindro cai e gira. Use
considerações de energia para encontrar a velocidade
v do centro de massa do cilindro sólido depois que ele
caiu de uma distância h.

O coeficiente de atrito entre a parede do canto, o piso


e o cilindro é igual a k. Quantas voltas o cilindro
realizará antes de parar?
𝑀𝑅 2
Considere 𝐼 = .
2

13. Um cilindro C está em repouso sobre uma


superfície horizontal. Uma pequena partícula de
massa m é mantida em equilíbrio conectada a uma
corda suspensa, conforme mostrado. A outra 16. Um cilindro sólido uniforme de massa M e raio 2R
extremidade da corda sem massa está sendo puxada repousa sobre o tampo de uma mesa horizontal. Um
horizontalmente por uma força F, como mostrado. fio preso a ele passa por uma polia (disco) de massa
Calcule o valor de F. M e raio R que é montada em um eixo sem atrito que
passa pelo seu centro. Um bloco de massa está
suspenso na extremidade livre da corda. A corda não
desliza sobre a superfície da polia e o cilindro rola sem
escorregar no tampo da mesa. Encontre o valor da
aceleração do bloco.

14. Uma massa pontual é amarrada a uma


extremidade de uma corda cuja outra extremidade
passa por um tubo oco vertical, preso em uma das
mãos. A massa pontual está sendo girada em um
círculo horizontal de raio 2 m com velocidade de 4 m/s.
A corda é então puxada para baixo de modo que o 17. Um ioiô de massa m tem um eixo de raio b e um
raio do círculo se reduza a 1 m. Calcule as novas carretel de raio R. O momento de inércia em relação
velocidades linear e angular da massa pontual e ao centro pode ser considerado Icm = (1/2) MR2 e a
calcule as energias cinéticas nos estados inicial e espessura do fio pode ser desprezada. O ioiô é
final. liberado do repouso. Você precisará assumir que o
centro de massa do ioiô desce verticalmente e que a
corda é vertical à medida que se desenrola.
do lutador em relação ao ponto O se o puxão a que
você aplica ao quimono

Qual é a tensão na corda enquanto o ioiô desce?


18. Derive uma expressão para o momento angular de
um projétil em relação ao ponto de lançamento, como
a função do tempo medido a partir do instante de
lançamento.

a) é desprezível

b) é horizontal, com um módulo de 300 N e aplicado a


uma altura h = 1,4 m?
Adote g = 9,8 m/s2

19. Uma fina esfera oca uniforme de massa m 22. (HALLIDAY & RESNICK) Uma roda de 32,0 kg,
permanece estacionária em uma superfície horizontal. que pode ser considerada um aro fino com 1,20 m de
A esfera oca está completamente preenchida com um raio, está girando a 280 rev/min. A roda precisa ser
líquido não viscoso de massa m. Vamos empurrar a parada em 15,0 s.
esfera para que ela comece a rolar sem deslizar na
superfície horizontal. Se a velocidade de CM da esfera a) Qual é o trabalho necessário para fazê-la parar?
oca for v, encontre o valor da energia cinética do
sistema que compreende a esfera oca e o líquido não b) Qual é a potência média necessária?
viscoso.
5.𝑚.𝑅 2 23. (HALLIDAY & RESNICK) Uma régua de um metro
Considere para a esfera oca 𝐼 = em relação
3 é mantida verticalmente com uma das extremidades
ao ponto de contato com o solo. apoiada no solo e depois liberada. Determine a
velocidade da outra extremidade pouco antes de tocar
o solo, supondo que a extremidade de apoio não
escorrega. (Sugestão: Considere a régua uma barra
fina e use a lei de conservação da energia.)
De acordo com o teorema dos eixos paralelos
𝑚𝑙 2
𝐼=
3

24. Uma casca esférica uniforme de massa M e raio R


gira em torno de um eixo vertical em um mancal sem
atrito, conforme mostrado na figura. Uma corda sem
20. (HALLIDAY & RESNICK) Em uma rasteira do massa passa em torno do equador da casca, sobre
judô, você tira o apoio do pé esquerdo do adversário uma polia de inércia rotacional I e raio r, e é presa a
e, ao mesmo tempo, puxa o quimono dele para o um pequeno objeto de massa m que, de outra forma,
mesmo lado. Em consequência, o lutador gira em estaria livre para cair sob a influência da gravidade.
torno do pé direito e cai no tatame. A figura mostra um Não há atrito do eixo da polia; a corda não escorrega
diagrama simplificado do lutador, já com o pé na polia. Qual é a velocidade do objeto depois que ele
esquerdo fora do chão. O eixo de rotação passa pelo caiu a uma distância h do repouso?
ponto O. A força gravitacional g age sobre o centro de
massa do lutador, que está a uma distância horizontal
d = 28 cm do ponto O. A massa do lutador é de 70 kg,
e o momento de inércia em relação ao ponto O é 65
kg · m2. Qual é o módulo da aceleração angular inicial
25. Uma bola uniforme de raio r rola sem escorregar
do topo de uma esfera de raio R. Encontre a
velocidade angular da bola no momento em que ela
se solta da esfera. A velocidade inicial da bola é
desprezível.

26. Um anel rola sobre uma superfície plana. A fração


de sua energia total associada à sua rotação é:

27. Uma esfera de massa M e raio R move-se sobre


uma superfície horizontal com velocidade v e depois
sobe um plano inclinado até uma altura h onde para.
A altura até a qual ele sobe será:

a) diretamente proporcional ao quadrado da


velocidade e inversamente proporcional ao ângulo de
inclinação

b) diretamente proporcional à velocidade e


inversamente proporcional à sua massa.

c) diretamente proporcional ao quadrado da


velocidade e independente da massa e do ângulo de
inclinação

d) diretamente proporcional à sua velocidade e


inversamente proporcional ao ângulo de inclinação.
Gabarito
1.

2
3
4
5
6
7
8

9
10
11

12
13

14

15
16

17
18
19

20
a)

b)

22

23
24

25

26

27
Letra C

Você também pode gostar